diff --git "a/exam/kaoyan.jsonl" "b/exam/kaoyan.jsonl" new file mode 100644--- /dev/null +++ "b/exam/kaoyan.jsonl" @@ -0,0 +1,377 @@ +{"instruction": "已知某温度时, 下列反应的标准平衡常数\n(1) $2 \\mathrm{CO}_2(\\mathrm{~g})=2 \\mathrm{CO}(\\mathrm{g})+\\mathrm{O}_2(\\mathrm{~g})$\n$$\nK_1^{\\ominus}=a\n$$\n(2) $\\mathrm{SnO}_2(\\mathrm{~s})=\\mathrm{Sn}(\\mathrm{s})+\\mathrm{O}_2(\\mathrm{~g})$\n$$\nK_2^{\\ominus}=b\n$$\n\n则在相同温度下, 反应 $\\mathrm{SnO}_2(\\mathrm{~s})+2 \\mathrm{CO}(\\mathrm{g})=\\mathrm{Sn}(\\mathrm{s})+2 \\mathrm{CO}_2(\\mathrm{~g})$ 的标准平衡常数 $K^{\\ominus}=$ __?", "input": "", "output": "本题考查多重平衡中, 总反应的标准平衡常数与各分反应标准平衡常数之间的关系。反应(2)减反应(1)可得总反应,所以, $K^{\\ominus}=K_{2}^{\\ominus} / K_{1}^{\\ominus}=b / a$ 。\n\n因此答案为:$b / a$", "task_type": {"major": ["试题"], "minor": ["考研", "填空题"]}, "domain": ["化学(农学类)"], "metadata": "year:2022, score:2", "answer_from": "human", "human_verified": false, "copyright": "暂无版权及作者信息"} +{"instruction": "电势分析中, 离子强度调节剂的作用是掩滶干扰离子、维持溶液的 $\\mathrm{pH}$ 和保持溶液的__?", "input": "", "output": "本题考查离子强度调节缓冲剂的定义和作用。\n\n因此答案为:离子强度", "task_type": {"major": ["试题"], "minor": ["考研", "填空题"]}, "domain": ["化学(农学类)"], "metadata": "year:2022, score:2", "answer_from": "human", "human_verified": false, "copyright": "暂无版权及作者信息"} +{"instruction": "下列三个反应(1) $\\mathrm{S}(\\mathrm{s})+\\mathrm{O}_2(\\mathrm{~g})=\\mathrm{SO}_2(\\mathrm{~g})$\n(2) $\\mathrm{H}_2(\\mathrm{~g})+\\mathrm{O}_2(\\mathrm{~g})=\\mathrm{H}_2 \\mathrm{O}_2$ (1)\n(3) $\\mathrm{C}(\\mathrm{s})+\\mathrm{H}_2 \\mathrm{O}(\\mathrm{g})=\\mathrm{CO}(\\mathrm{g})+\\mathrm{H}_2(\\mathrm{~g})$其中 $\\Delta_{\\mathrm{r}} S_{\\mathrm{m}}^{\\ominus}$ 由小到大的顺序为___。(填序号)", "input": "", "output": "本题考查物质的标准摩尔熵的变化规律, 通常微粒的运动自由程度总是气态大于液态、液态大于固态,故可根据方程式两侧气体分子数的变化判断反应的摘变的正负号。\n\n因此答案为:(2) (1) (3)", "task_type": {"major": ["试题"], "minor": ["考研", "填空题"]}, "domain": ["化学(农学类)"], "metadata": "year:2022, score:2", "answer_from": "human", "human_verified": false, "copyright": "暂无版权及作者信息"} +{"instruction": "在 $\\mathrm{C}_{2} \\mathrm{H}_{4} 、 \\mathrm{HCN} 、 \\mathrm{~N}_{2}$ 和 $\\mathrm{H}_{2} \\mathrm{O}$ 中, 只含单键的是___?", "input": "", "output": "本题考查共价键部分的相关知识。 $\\mathrm{C}_{2} \\mathrm{H}_{4}$ 分子中有 $\\mathrm{C}=\\mathrm{C}$ 双键, $\\mathrm{HCN}$ 分子中有 $\\mathrm{C} \\equiv \\mathrm{N}, \\mathrm{N}_{2}$分子中有 $\\mathrm{N} \\equiv \\mathrm{N}$, 分子中仅存在 $\\mathrm{O}-\\mathrm{H}$ 。\n\n因此答案为:$\\mathrm{H}_{2} \\mathrm{O}$", "task_type": {"major": ["试题"], "minor": ["考研", "填空题"]}, "domain": ["化学(农学类)"], "metadata": "year:2022, score:2", "answer_from": "human", "human_verified": false, "copyright": "暂无版权及作者信息"} +{"instruction": "用 $\\mathrm{KMnO}_{4}$ 溶液滴定 $\\mathrm{Na}_{2} \\mathrm{C}_{2} \\mathrm{O}_{4}$ 溶液之前, 加人几滴 $\\mathrm{MnSO}_{4}$ 溶液的作用是___?", "input": "", "output": "本题考查氧化还原滴定中, 提高反应速率可采取的措施。 $\\mathrm{KMnO}_{4}$ 和 $\\mathrm{Na}_{2} \\mathrm{C}_{2} \\mathrm{O}_{4}$ 反应后产生的 $\\mathrm{Mn}^{2+}$ 会作为催化剂加快该反应的进行,故可提前加人几滴 $\\mathrm{MnSO}_{4}$ 溶液。\n\n因此答案为:催化 (或加快反应)", "task_type": {"major": ["试题"], "minor": ["考研", "填空题"]}, "domain": ["化学(农学类)"], "metadata": "year:2022, score:2", "answer_from": "human", "human_verified": false, "copyright": "暂无版权及作者信息"} +{"instruction": "红色配合物的结构简式为 $\\mathrm{CoBr}\\left(\\mathrm{SO}_{4}\\right)\\left(\\mathrm{NH}_{3}\\right)_{5}$ 。向该配合物溶液中按下表分别滴加溶液,根据实验结果, 判断该配合物的内界为___?\n\\begin{center}\n\\begin{tabular}{c|c}\n\\hline\n滴加溶波 & 实验结果 \\\\\n\\hline\n$\\mathrm{AgNO}_{3}$ & 有 $\\mathrm{AgBr}^{2}$ 沉淀 \\\\\n\\hline\n$\\mathrm{BaCl}_{2}$ & 无 $\\mathrm{BaSO}_{4}$ 沉淀 \\\\\n\\hline\n$\\mathrm{NaOH}$ & 无氨气 \\\\\n\\hline\n\\end{tabular}\n\\end{center}", "input": "", "output": "本题考查配合物的组成。配合物的外界可完全离解, 而内界仅能部分离解。\n\n因此答案为:$\\left[\\mathrm{Co}\\left(\\mathrm{SO}_{4}\right)\\left(\\mathrm{NH}_{3}\right)_{5}\right]^{+}$", "task_type": {"major": ["试题"], "minor": ["考研", "填空题"]}, "domain": ["化学(农学类)"], "metadata": "year:2022, score:2", "answer_from": "human", "human_verified": false, "copyright": "暂无版权及作者信息"} +{"instruction": "已知电池 (- $) \\mathrm{Cu}^{\\mathrm{C}} \\mathrm{Cu}^{2+}\\left(0.1 \\mathrm{~mol} \\cdot \\mathrm{L}^{-1}\\right) \\| \\mathrm{Cu}^{2+}\\left(0.5 \\mathrm{~mol} \\cdot \\mathrm{L}^{-1}\\right) \\mid \\mathrm{Cu}(+), 25{ }^{\\circ} \\mathrm{C}$ 时, 该电池反应的平衡常数 $K^{\\ominus}=$____?", "input": "", "output": "本题考查 $\\Delta_{\\mathrm{r}} G_{\\mathrm{m}}^{\\ominus}$ 和 $K^{\\ominus}$ 的关系。该电池为浓差电池, 但标准状态下, $\\Delta_{\\mathrm{r}} G_{\\mathrm{m}}^{\\ominus}=0$, 根据 $\\Delta_{\\mathrm{r}} G_{\\mathrm{m}}^{\\ominus}$ $=-R T \\ln K^{\\ominus}$, 可得: $K^{\\ominus}=1$\n\n因此答案为:1", "task_type": {"major": ["试题"], "minor": ["考研", "填空题"]}, "domain": ["化学(农学类)"], "metadata": "year:2022, score:2", "answer_from": "human", "human_verified": false, "copyright": "暂无版权及作者信息"} +{"instruction": "$\\mathrm{PbI}_{2}$ 在纯水中的溶解度为 $1.28 \\times 10^{-3} \\mathrm{~mol} \\cdot \\mathrm{L}^{-1}$, 则 $K_{\\mathrm{sp}}^{\\ominus}\\left(\\mathrm{PbI}_{2}\\right)=$___?", "input": "", "output": "本题考查沉淀溶解平衡和溶度积常数。溶液中存在下列平衡:\n$$\n\\mathrm{PbI}_2(\\mathrm{~s})=\\mathrm{Pb}^{2+}(\\mathrm{aq})+2 \\mathrm{I}^{-}(\\mathrm{aq})\n$$\n\n设纯水中溶解度 $\\mathrm{PbI}_2$ 为 $S$, 则: $S$ 界\n$$\n\\begin{aligned}\n& K_{\\mathrm{sp}}^{\\ominus}\\left(\\mathrm{PbI}_2\\right)=\\frac{c\\left(\\mathrm{~Pb}^{2+}\\right)}{c^{\\ominus}} \\cdot\\left[\\frac{c\\left(\\mathrm{I}^{-}\\right.}{c^{\\ominus}}\\right]^2=\\frac{S}{c^{\\ominus}} \\cdot\\left(\\frac{2 S}{c}\\right)^2=4\\left(\\frac{S}{c^{\\ominus}}\\right)^3 \\\\\n& S=1.28 \\times 10^{-3} \\mathrm{~mol} \\cdot \\mathrm{L}^{-1} \\text { 时, } K_{\\mathrm{sp}}^{\\ominus}\\left(\\mathrm{PbI}_2\\right)=8.39 \\times 10^{-9}\n\\end{aligned}\n$$\n因此答案为:$8.39 \times 10^{-9}$", "task_type": {"major": ["试题"], "minor": ["考研", "填空题"]}, "domain": ["化学(农学类)"], "metadata": "year:2022, score:2", "answer_from": "human", "human_verified": false, "copyright": "暂无版权及作者信息"} +{"instruction": "向 $10 \\mathrm{~mL} 0.20 \\mathrm{~mol} \\cdot \\mathrm{L}^{-1} \\mathrm{NH}_{3} \\cdot \\mathrm{H}_{2} \\mathrm{O}\\left(\\mathrm{p} K_{\\mathrm{b}}^{\\ominus}=4.74\\right)$ 溶液中滴加 2 滴酚唒指示剂, 加人等体积的 $0.10 \\mathrm{~mol} \\cdot \\mathrm{L}^{-1} \\mathrm{HCl}$ 溶液后, 溶液颜色为___?", "input": "", "output": "本题考查酸碱平衡和弱酸弱碱水溶液中 $\\mathrm{pH}$ 的计算。\n据题意, 加入等体积的 $0.10 \\mathrm{~mol} \\cdot \\mathrm{L}^{-1}$ 的 $\\mathrm{HCl}$ 溶液后, $\\mathrm{NH}_{3}+\\mathrm{H}^{+} \rightarrow \\mathrm{NH}_{4}^{+}$\n达到平衡时: $c\\left(\\mathrm{NH}_{3}\right)=0.050 \\mathrm{~mol} \\cdot \\mathrm{L}^{-1}, c\\left(\\mathrm{NH}_{4}^{+}\right)=0.050 \\mathrm{~mol} \\cdot \\mathrm{L}^{-1}$\n可根据 $\\mathrm{pH}=\\mathrm{p} K_{a}^{\\ominus}\\left(\\mathrm{NH}_{4}^{+}\right)-\\lg \frac{c\\left(\\mathrm{NH}_{4}^{+}\right)}{c\\left(\\mathrm{NH}_{3}\right)}$ 或 $\\mathrm{pOH}=\\mathrm{p} K_{b}^{\\ominus}\\left(\\mathrm{NH}_{3}\right)-\\lg \frac{c\\left(\\mathrm{NH}_{3}\right)}{c\\left(\\mathrm{NH}_{4}^{+}\right)}$计算得溶液 $\\mathrm{pH}=14-4.74=9.26$, 接近酚酞指示剂的变色点的 $\\mathrm{pH}$, 此时酚唒指示剂呈浅红 (浅粉)。\n因此答案为:浅红 (浅粉)", "task_type": {"major": ["试题"], "minor": ["考研", "填空题"]}, "domain": ["化学(农学类)"], "metadata": "year:2022, score:2", "answer_from": "human", "human_verified": false, "copyright": "暂无版权及作者信息"} +{"instruction": "反-1-乙基-3-叔丁基环已烷优势构象中,乙基在____键上。(填“ $a$ ”或“ $e$ ”)", "input": "", "output": "反-1-乙基-3-叔丁基环已烷优势构象中,较大基团叔丁基在 $e$ 键上,乙基在$a$键上。\n\n因此答案为:$a$", "task_type": {"major": ["试题"], "minor": ["考研", "填空题"]}, "domain": ["化学(农学类)"], "metadata": "year:2022, score:2", "answer_from": "human", "human_verified": false, "copyright": "暂无版权及作者信息"} +{"instruction": "化合物 $\\mathrm{H}_{3} \\mathrm{C}-\\mathrm{H}-\\mathrm{COOH}$ 的系统命名是____。(标明“ $R$ ”或“ $S$ ”)", "input": "", "output": "化合物 $\\mathrm{H}_{3} \\mathrm{C}-\\stackrel{\\stackrel{\\mathrm{H}}{\\mathrm{C}_{2} \\mathrm{H}_{5}}}{\\mathrm{COOH}}$ 的系统命名是 $(R)$-2-甲基丁酸。\n\n因此答案为:$(R)$-2-甲基丁酸", "task_type": {"major": ["试题"], "minor": ["考研", "填空题"]}, "domain": ["化学(农学类)"], "metadata": "year:2022, score:2", "answer_from": "human", "human_verified": false, "copyright": "暂无版权及作者信息"} +{"instruction": "乙酸乙酯-水的二元体系中,水在___层。(填“上”或“下”)", "input": "", "output": "乙酸乙酯-水的二元体系中, 水比乙酸乙酯重, 在下层。\n\n因此答案为:下", "task_type": {"major": ["试题"], "minor": ["考研", "填空题"]}, "domain": ["化学(农学类)"], "metadata": "year:2022, score:2", "answer_from": "human", "human_verified": false, "copyright": "暂无版权及作者信息"} +{"instruction": "两组分混合物在硅胶薄层板上展开后, 比移值 $\\mathrm{R}_{\\mathrm{f}}$ 较大组分的极性较____。(填 “大”或“小”)", "input": "", "output": "两组分混合物在硅胶薄层板上展开后, 极性小的组分移动速率较快, 相对于溶剂的速率即比移值 $\\mathrm{R}_{\\mathrm{f}}$ 较大。故答案是极性较小\n\n因此答案为:小", "task_type": {"major": ["试题"], "minor": ["考研", "填空题"]}, "domain": ["化学(农学类)"], "metadata": "year:2022, score:2", "answer_from": "human", "human_verified": false, "copyright": "暂无版权及作者信息"} +{"instruction": "Directions:Read the following text. Choose the best word(s) for each numbered blank and mark A, B, C or D on the ANSWER SHEET. (10 points\nThe idea that plants have some degree of consciousness first took root in the early 2000s; the term “plant neurobiology” was 1 around the notion that some aspects of plant behavior could be 2 to intelligence in animals. 3 plants lack brains, the firing of electrical signals in their stems and leaves nonetheless triggered responses that 4 consciousness, researchers previously reported.\n But such an idea is bunk, according to the authors of the new article. Plant biology is complex and fascinating, but it 5 so greatly from that of animals that so-called 6 of plants’ intelligence is intriguing but inconclusive, the scientists wrote. In animals, neurobiology refers to the biological mechanisms through which a nervous system regulates behavior, according to Harvard University’s Mind Brain Behavior Interfaculty Initiative. Over millions of years, brains in diverse animal species have evolved to produce behaviors that experts identify as intelligent: Among them are reasoning and problem-solving, tool use and self-recognition.\n Beginning in 2006, some scientists have 7 that plants possess neuron-like cells that interact with hormones and neurotransmitters, 8 “a plant nervous system, 9 to that in animals,” said lead study author Lincoln Taiz, a professor emeritus of molecular, cell and developmental biology at the University of California Santa Cruz.\n “They 10 claimed that plants have ‘brain-like command centers’ at their root tips,” Taiz told Live Science in an email.\n This 11 makes sense if you simplify the workings of a complex brain, 12 it to an array of electrical pulses; cells in plants also communicate through electrical signals, according to the article. 13 , the signaling in a plant is only 14 similar to the billions of synapses firing in a complex animal brain, which is more than “a mass of cells that communicate by electricity,” Taiz said.\n “For consciousness to evolve, a brain with a threshold 15 of complexity and capacity is required,” he 16 .\n Other researchers who recently investigated the neuroscience of consciousness—awareness of one’s world and a sense of self—found that in animals, only vertebrates, arthropods and cephalopods had brains complex enough to enable them to be conscious.\n “If the lower animals—which have nervous systems—lack consciousness, the 17 that plants without nervous systems have consciousness are effectively nil,” Taiz said\n.And what’s so great about consciousness, anyway? Plants can’t run away from 18 , so investing energy in a body system that 19 a threat and can feel pain would be a very 20 evolutionary strategy, according to the article.\n1. \n[A] coined \n[B] discovered \n[C] collected \n[D] issued\n2. \n[A] attributed \n[B] directed \n[C] compared \n[D] confined\n3. \n[A] Unless \n[B] When \n[C] Once \n[D]Though\n4. \n[A] coped with \n[B] consisted of \n[C] hinted at \n[D] extended to\n5. \n[A] suffers \n[B] benefits \n[C] develops \n[D] differs\n6. \n[A] acceptance \n[B] evidence \n[C] cultivation \n[D] creation\n7. \n[A] doubted \n[B] denied \n[C] argued \n[D] requested\n8. \n[A] adapting \n[B] forming \n[C] repairing \n[D] testing\n9. \n[A] analogous \n[B] essential \n[C] suitable \n[D] sensitive\n10. \n[A] just \n[B] ever \n[C] still \n[D] even\n11. \n[A] restriction \n[B] experiment \n[C] perspective \n[D] demand\n12. \n[A] attaching \n[B] reducing \n[C] returning \n[D] exposing\n13. \n[A] However \n[B] Moreover \n[C] Therefore \n[D] Otherwise\n14. \n[A] temporarily \n[B] literally \n[C] superficially \n[D] imaginarily\n15. \n[A] list \n[B] level \n[C] label \n[D] load\n16. \n[A] recalled \n[B] agreed \n[C] questioned \n[D] added\n17. \n[A] chances \n[B] risks \n[C] excuses \n[D] assumptions\n18. \n[A] danger \n[B] failure \n[C] warning \n[D] control\n19. \n[A] represents \n[B] includes \n[C] reveals \n[D] recognizes\n20. \n[A] humble \n[B] poor \n[C] practical \n[D] easy", "input": "", "output": "1、【解题思路】该空分号前半句讲植物具有某种程度的意识的想法最早在21世纪初生根 。可以 看出,动物有意识是常识,这种认为植物有意识的想法是比较新颖的 。本空所��句要表达的含义是the term“plant neurobiology ” was ___around the notion that...(“植物神经生物学”一词是围绕 着这种观念被___) 。纵览四个选项,这一术语被创造出来符合文意 。故答案为【A】 。需要注意的 是,coin名词是硬币,本空取动词含义“创造(新词语)”之意。【干扰排除】 【B】 discovered (发现), 指自然界原本就有的东西,这一术语并非自然界中原有的东西,是近期人们才提出来的,故排除 。【C】 collected(收集),收集术语搭配不当,排除 。【D】issued(发出,公布)多指期刊、杂志等的发行、发布或公开正式地宣布,不符合文意,故排除。\n2、【解题思路】前文讲植物有意识的想法最近被提出,本空所在句提到动物智商的问题,显然 是植物和动物了比较 。该空要表达的意思为the notion that some aspects of plant behavior could be___ to intelligence in animals (有一种观点认为植物行为的某些方面可以与动物的智力相提并 论 。)故【C】compared(比较)为本题答案。【干扰排除】 【A】attributed代入构成搭配be attributed to(被归因于 ... ...),意思是植物的行为被归因于动物的智商, 明显逻辑错误,故排除 。【B】directed to (把 ... ...对准某物; 为 ... ... 引路), 不合文意 。【D】confined (be confined to局限, 限制),代入意思是植物行为的某些方面可能仅 限于动物的智力,逻辑不通,故排除。\n3、【解题思路】本题考查逻辑关系 。空格前半句提到植物没有大脑,后半句说茎叶中发出的电 信号仍会引发反应,前后构成逻辑上的转折让步关系 。故答案为【D】Though(虽然) 。【干扰排除】 【A】Unless(除非),意思为除非植物没有大脑,而事实情况是植物本来就没有大脑,构不成反条件逻辑关系 。【B】When(当 ... ... 时候),【C】Once(一旦)代入皆不符合逻辑,均排除\n4、【解题思路】该空为定语从句部分,that指代responses (反应),结合第三题,本句要表达 的含义应为研究人员此前曾报道,虽然植物没有大脑,但它们的茎和叶发出的电信号仍然引发 了暗示了植物有意识的反应 。故答案为【C】hinted at(暗示)【干扰排除】 【A】 coped with (应对, 处理),应对意识搭配不当,排除; 【B】consisted of(由 ... 构成),有意识构成的反应,逻辑不通,故排除; 【D】 extended to(extend延伸),extend为及物 动词,没有extend to 的用法,故而排除。\n5、【解题思路】本题空格位于but转折词后 。整句话的句意为:植物生物学复杂而迷人,但它与 动 物 的 相比 _____ 巨 大 (Plant biology is complex and fascinating, but it __5__ so greatly from that...)。上文提到植物似乎像动物一样拥有自主意识,而本段第一句话but转折暗示内容与上文 相反, 即作者认为植物生物学和动物的有差别,且后面from..., differ from...表示“与 ... ...不同”, 因而选【D】。【干扰排除】题干部分...from that of animals说明前后有比较概念,选项【A】suffers (遭受), 【B】benefits (受益), 【C】develops(发展)都不能体现比较的概念, 因此均排除\n6、【解题思路】空格词位于由so...that引导的结果状语从句中,所在部分的句意为“植物生物学 与动物的相比差异巨大以至于那些所谓的_____并没有说服力 。”上一段提到之前关于植物的研 究发现来证明植物像动物一样有自主意识, 而本段在反驳上文内容, 即上文的这些作为“证明” 的发现尚无定论, 只有选项【B】evidence符合文意。【干扰排除】所在句后为形容词inconclusive,形容某物是非结论的,不能让人信服的,说明空格 需要填一个偏向研究结果 、证据的词,选择【A】acceptance (接受), 【C】cultivation (培养)或 【D】creation(创造)均不符合\n7、【解题思路】空格位于some scientists have _____ that...,所在部分为句子的主句,需要填写一 个动词的过去分词形式,句意为“自2006开始,一些科学家_____植物拥有与激素和神经递质相 互作用的神经元样细胞 ... ...” 。that引导的内容是在说明植物拥有类似的精神细胞,该发现还是 支持植物拥有自主意识的内容,且Beginning in 2006呼应文章第一段第一句话的early 2000s,说 明这些科学家的态度意见与第一段相同, 即这些科学家也主张植物有神经细胞的观点, 因此选 择选项【C】argued(主张) 。【干扰排除】选项【A】doubted (怀疑)和【B】denied (否认)都体现是否定植物拥有神经细胞的观 点,与原文意思相反,故排除。选项【D】requested, 而that后面的内容是一个结论, 无法与requested 搭配,故排除。\n8、【解题思路】空格处位于a plant nervous system,需要填入一个现在分词,构成前面的内容的结 果。前面在表述有这样的神经细胞,后面的结果是与植物神经系统相关,将四个词代入句子中, 只有【B】forming符合逻辑, 即有了这些神经细胞从而形成了植物神经系统,故正确答案为选项 【B】。【干扰排除】根据前文的表述,描述植物拥有类似的神经细胞想要论证的植物有意识,有这样的 神经系统来体现 。而把选项【A】adapting(适应),选项【C】repairing(修复)和选项【D】testing(测 试)分别代入句中,都是已经在具体描述这个神经细胞的作用,不符合前后的逻辑,故排除。\n9、【解题思路】 空格处位于__9___to that in animals,意为“____动物的神经系统”,本题为形容 词辨析,需要一个形容词连接植物神经系统与动物神系统的关系 。前一句提到植物有与激素及 神经传到物质相互作用的神经样细胞,后面一句话也提到植物有类似大脑的指挥中心,因而选【A】 类似的。【干扰排除】【B】 essential (重要的), 【C】 suitable (合适的), 【D】 sensitive (敏感的),三个选项 均不能与前后句进行衔接与连贯,不符合上下文逻辑,故而排除。\n10、【解题思路】空格处位于They10 claimed that plants have 'brain-like command centers’ at theirroot tips,意为“他们______声称,植物的根尖上有‘类似大脑的指挥中心’”, 此处同样表明的是上文提及的科学家的相同观点并表达了递进逻辑, 因而选【D】even (甚至) 。【干扰排除】 【A】 just(仅仅, 刚刚)和【B】ever(曾经)表时间逻辑,与原文表达逻辑不符, 因此 排除; 【C】 still(仍然),虽然同样可以表达了与前文观点的逻辑一致性,但语义不符, 因此排除。\n11、【解题思路】 空格处位于This11makes sense,意为“这个____说得通”,代词this后要选 的这个名词应该是对上 一 段有些科学家的观点 They even claimed that plants have ‘brain-like command centers’ at their root tips... (他 们 甚 至 宣 称 植 物 在 其 根 尖 也 有 类 似 大 脑 的 控 制 中 心 ... ...), 因而选【C】perspective (观点,看法) 。【干扰排除】 【A】 restriction(限制), 【B】 experiment(实验),和【D】 demand(需求)三个选项的含 义与本文无关,上文并未出现过,故排除。\n12、【解题思路】空格处位于12 it to an array of electrical pulses,整个句子意为“如果你简化 一个复杂大脑的工作方式,将其______一系列电脉冲,这个观点就说得通了”,因为空格前文提 及simplify简化,后半句对前半句的解释,空格处的单词应该与前半句中的simplify为同意替换, 因而此处选【B】 reducing(减少,简化) 。【干扰排除】 【A】 attaching(使附着), 【C】returning(返回,归还)和【D】exposing(暴露)三个选 项的含义都不能和simplify构成语义上的同意替换,故而排除。\n13、【解题思路】空格处位于句首,选项均为逻辑关系词 。可以根据本句话和前文的意思来确定逻辑关系。前文提到“植物有一个像大脑一样的指挥中心,并且这种观点也是有科学道理的。” 再根据本句话中的only...similar...可知“植物与动物仅仅是相似”暗含了对于前文植物与动物一样存在意识这种观点的否定,故选择表示转折的逻辑关系词 。【A】选项“However(然而)”符合句意。【干扰排除】 【B】 Moreover表示“并且”,属于并列或递进的逻辑关系, 与后文否定前文内容的转折关系不符,故排除【B】选项; 【C】Therefore表示“因此”,属于因果逻辑关系,前文肯定了“植物 存在意识”,后文否定了这种观点,表示“只是类似于动物而已”,并不存在因果关系,故排除【C】选项;【D】otherwise表示“否则”, 虽然有转折含义,但是otherwise更多的是否定前文内容后会产 生的结果,而原文只是表示对于前文的否定,没有针对后续结果进行阐述,故排除【D】选项。\n14、【解题思路】空格处位于the signaling in a plant is only14 similar to...,纵览四个选项,可 知本题要填入副词修饰“植物只是 ... ...类似于动物”,根据前文内容可知“植物像动物一样拥有意 识 ... ...”本句进行转折,应该填入一个副词表示与动物的这种类似程度应该非常轻,才可以表达 出否定前文观点的含义,对比选项发现【C】superficially“表面上地、浅显地”代入题目后句意为“然 而,植物只是表面上跟动物类似”,暗含对于这种观点的否定,故选【C】。【干扰排除】 【A】 temporarily表示“暂时地”,表示时间上存在先后,但原文并未表明“目前植���与动 物相似,之后便不再相似”的含义,因此排除选项【A】。【B】 literally表示“实际上”,指对于目前情况的肯定,由于本句话有however,表示对于前文内容的否定,与句意不符,故排除选项【B】。【D】imaginarily表示“在想象中地”,植物类似于动物这样的观点是经过科学研究得到的结论,并不是 主观想象,故排除选项【D】。\n15、【解题思路】本题位于本句话的主语的位置,意为“意识的进化需要复杂程度和能力的门 槛 ... ...”,纵览四个选项, 只有填入【B】选项level“水平”可保持句意通顺,意为“意识的进化意味 着复杂程度和能力要求都很高”,以此来呼应上文“植物并不像动物一样有大脑控制中心”这样的 观点,作为对于“植物存在意识”这种观点的一种较轻程度的否定,故正确答案为【B】。【干扰排除】 【A】 list(清单)表示一件事情含有多种要素,原文并未列举意识出现的各个要素, 故排除【A】;【C】label(标签),原文中未提及意识出现的标志或标签,故排除【C】;【D】 load(负载 、工作量),原文中并未提及意识的出现是一种负担,故排除【D】。\n16、【解题思路】 空格位于句末,作为he(指前段中出现的学者Taiz)所发出的动作,结合句意 可知,双引号的内容为这位学者所提出的观点,结合前段这位学者已经提出了一种观点,纵览 四个选项, 【D】选项added(补充)符合句意,表示本句话是承接上一段这位学者所提出的第二 个观点,表示并列或递进关系,故选【D】。【干扰排除】 【A】选项 recalled表示“回忆”, 由本句句意可知这是学者提出的观点, 并非对于过去的事情的回忆,故排除【A】;【B】 agreed(同意),一般用在与前文所提到的某种观点一致时使用, 但本句话是这位学者提出的观点, 并未与其他观点相同,故排除【B】; 【D】 questioned(质疑), 根据句意可知,本句话是该学者的观点, 而非其质疑的内容,故排除【D】。\n17、【解题思路】空格所在部分后面的that从句是一个同位语从句, 整句话的主干是: the are...zero, ”根据语境, 空格前文提到If the lower animals — which have nervous systems — lack consciousness (如果具有神经系统的低级动物缺乏意识的话),那么“没有圣经系统的植物拥有 意识的可能性为零” 。表示可能性“chances”,故而选【A】。【干扰排除】 【B】risks表示“风险”与原文意思不符,前文提到的是如果动物有意思, 与下文植物是否有意识进行联系,故而排除;【C】excuses表示“借口”,“借口为零”搭配并不通顺,【D】assumptions 表示“假设”,“假设为零”搭配也不合适, 因而不管是搭配还是从原文意思出发,都可以排除。\n18、【解题思路】空格处位于Plants can’t run away from 18,本句可通过短语意思搭配解题 。 空格前面run away from表示“逃离”, 前文问道“意识到底有什么厉害之处”可以看出,所想要表 达的隐含意是:有意识的动物能逃离危险,而植物不能逃离危险 。且后文还有threat“威胁”做呼 应, 因而本题应选【A】。【干扰排除】 【B】 failure表示“失败”,“逃离失败”的话,更像是动物干的事, 而文中主语是植物, 搭配不符; 【C】 warning表示“警告”,该词本身就与run away from不搭配,警告应该是听从或者 不听,而不是逃离; 【D】 control表示“控制”,“run away from control”搭配可以,但就原文意思来 看, 并没有提到过谁的控制,有时候动物也可能逃离不了人的控制,所以这一点对于普遍的动 植物在意识的区分并不成立,故而排除。\n19、【解题思路】 根据后面and搭配的短语含义解题 。19 a threat and can feel pain,本空谓语 定从修饰前面body system的后面“ ... ...威胁并能感觉到疼痛”再根据前文中“逃离危险”可知,这 里应该填入“感知威胁/意识到威胁”之类的词,【D】recognizes可以表达这层意思, “认出; 识别”, 故而本题选【D】。【干扰排除】 【A】 represents(代表),代表威胁并感知疼痛,搭配不合适,排除;【B】 includes(包含),包含一个威胁并感到疼痛,搭配也不合适,排除;【C】reveals表示“揭露;展现”,如果是 某种现象揭露出一种威胁可以,但后面还搭配着feel pain,且前面修饰的是a body system,也就 是说前面修饰的相当于是一个动物/植物,它还能感觉到疼痛,所以用揭露不合适,故而排除。\n20、【解题思路】 本空位于a very20 evolutionary strategy,“这是一个什么样的进化策略”,根 据本句语气解题。从前文得知信息,���文说到“那么,意识到底有什么了不起的呢?植物无法逃 避危险,因此将精力投入到一个能够识别威胁并能感受到疼痛的身体系统中将是一种非常 进 化策略”根据意思可得知,本空应该是一个贬义词,类似于做无用功的意思,因此选项中带贬义 的只有【B】poor在文中不表示“贫穷的”,而表示“糟糕的”,选【B】。【干扰排除】 【A】 humble表示 “谦虚的;卑微的”是一个正向意义的词,通常用于形容人的品质;【C】 practical表示“实际的;切实可的”也是一个褒义词, 与文中语气不符,故排除; 【D】easy表 示“容易的”,与策略搭配不合适,且也与语气不符,故而排除。", "task_type": {"major": ["试题"], "minor": ["考研", "填空题"]}, "domain": ["英语"], "metadata": "year:2022, score:0.5", "answer_from": "human", "human_verified": false, "copyright": "暂无版权及作者信息"} +{"instruction": "$\\int_{0}^{+\\infty} \\frac{d x}{x^{2}+2 x+2}=$", "input": "", "output": "$\\int_{0}^{+\\infty} \\frac{d x}{x^{2}+2 x+2}=\\int_{0}^{+\\infty} \\frac{d x}{(x+1)^{2}+1}=\\left.\\arctan (x+1)\\right|_{0} ^{+\\infty}=\\frac{\\pi}{2}-\\frac{\\pi}{4}=\\frac{\\pi}{4}$\n因此答案为: $\\frac{\\pi}{4}$", "task_type": {"major": ["试题"], "minor": ["考研", "填空题"]}, "domain": ["数学"], "metadata": "year:2021, score:5", "answer_from": "human", "human_verified": false, "copyright": "暂无版权及作者信息"} +{"instruction": "设函数 $y=y(x)$ 由参数方程 $\\left\\{\\begin{array}{c}x=2 e^{t}+t+1, x<0 \\\\ y=4(t-1) e^{t}+t^{2}, x \\geq 0\\end{array}\\right.$ 确定, 则 $\\left.\\frac{d^{2} y}{d x^{2}}\\right|_{t=0}=$", "input": "", "output": "由 $\\frac{d y}{d x}=\\frac{4 t e^{t}+2 t}{2 e^{t}+1}$, 得 $\\frac{d^{2} y}{d x^{2}}=\\frac{\\left(4 e^{t}+4 t e^{t}+2\\right)\\left(2 e^{t}+1\\right)-\\left(4 t e^{t}+2 t\\right) 2 e^{t}}{\\left(2 e^{t}+1\\right)^{3}}$,将 $t=0$ 带入得 $\\left.\\frac{d^{2} y}{d x^{2}}\\right|_{t=0}=\\frac{2}{3}$.\n因此答案为: $\\frac{2}{3}$", "task_type": {"major": ["试题"], "minor": ["考研", "填空题"]}, "domain": ["数学"], "metadata": "year:2021, score:5", "answer_from": "human", "human_verified": false, "copyright": "暂无版权及作者信息"} +{"instruction": "欧拉方程 $x^{2} y^{\\prime \\prime}+x y^{\\prime}-4 y=0$ 满足条件 $y(1)=1, y^{\\prime}(1)=2$ 得解为 $y=$", "input": "", "output": "令 $x=e^{t}$, 则 $x y^{\\prime}=\\frac{d y}{d t}, x^{2} y^{\\prime \\prime}=\\frac{d^{2} y}{d x^{2}}-\\frac{d y}{d x}$, 原方程化为 $\\frac{d^{2} y}{d x^{2}}-4 y=0$, 特征方程为 $\\lambda^{2}-4=0$, 特征根为 $\\lambda_{1}=2, \\lambda_{2}=-2$, 通解为 $y=C_{1} e^{2 t}+C_{2} e^{-2 t}=C_{1} x^{2}+C_{2} x^{-2}$, 将初始条件 $y(1)=1, y^{\\prime}(1)=2$ 带入得 $C_{1}=1, C_{2}=0$, 故满足初始条件的解为 $y=x^{2}$.\n因此答案为: $x^{2}$", "task_type": {"major": ["试题"], "minor": ["考研", "填空题"]}, "domain": ["数学"], "metadata": "year:2021, score:5", "answer_from": "human", "human_verified": false, "copyright": "暂无版权及作者信息"} +{"instruction": "设 $\\Sigma$ 为空间区域 $\\left\\{(x, y, z) \\mid x^{2}+4 y^{2} \\leq 4,0 \\leq z \\leq 2\\right\\}$ 表面的外侧, 则曲面积分 $\\iint_{\\Sigma} x^{2} d y d z+y^{2} d z d x+z d x d y=$", "input": "", "output": "由高斯公式得原式 $=\\iiint_{\\Omega}(2 x+2 y+1) d V=\\int_{0}^{2} d z \\iint_{D} d x d y=4 \\pi$.\n因此答案为: $4 \\pi$", "task_type": {"major": ["试题"], "minor": ["考研", "填空题"]}, "domain": ["数学"], "metadata": "year:2021, score:5", "answer_from": "human", "human_verified": false, "copyright": "暂无版权及作者信息"} +{"instruction": "设 $A=a_{i j}$ 为 3 阶矩阵, $A_{i j}$ 为代数余子式, 若 $A$ 的每行元素之和均为 2 , 且 $|A|=3$, $A_{11}+A_{21}+A_{31}=$", "input": "", "output": "$A\\left(\\begin{array}{l}1 \\\\ 1 \\\\ 1\\end{array}\\right)=2\\left(\\begin{array}{l}1 \\\\ 1 \\\\ 1\\end{array}\\right), A \\alpha=\\lambda \\alpha, \\lambda=2, \\alpha=\\left(\\begin{array}{l}1 \\\\ 1 \\\\ 1\\end{array}\\right)$, 则 $A^*$ 的特征值为 $\\frac{|A|}{\\lambda}$, 对应的特征向量为\n$$\n\\begin{gathered}\n\\alpha=\\left(\\begin{array}{l}\n1 \\\\\n1 \\\\\n1\n\\end{array}\\right), A^* \\alpha=\\frac{|A|}{\\lambda} \\alpha \\text { 而 } A^*=\\left(\\begin{array}{ccc}\nA_{11} & A_{21} & A_{31} \\\\\nA_{12} & A_{22} & A_{32} \\\\\nA_{13} & A_{23} & A_{33}\n\\end{array}\\right), A^*\\left(\\begin{array}{l}\n1 \\\\\n1 \\\\\n1\n\\end{array}\\right)=\\left(\\begin{array}{c}\nA_{11}+A_{21}+A_{31} \\\\\nA_{12}+A_{22}+A_{32} \\\\\nA_{13}+A_{23}+A_{33}\n\\end{array}\\right)=\\frac{|A|}{\\lambda}\\left(\\begin{array}{l}\n1 \\\\\n1 \\\\\n1\n\\end{array}\\right) \\text {, 即 } \\\\\nA_{11}+A_{21}+A_{31}=\\frac{3}{2} .\n\\end{gathered}\n$$\n因此答案为: $\\frac{3}{2}$", "task_type": {"major": ["试题"], "minor": ["考研", "填空题"]}, "domain": ["数学"], "metadata": "year:2021, score:5", "answer_from": "human", "human_verified": false, "copyright": "暂无版权及作者信息"} +{"instruction": "甲乙两个盒子中各装有 2 个红球和 2 个白球, 先从甲盒中任取一球, 观察颜色后放入乙盒中,再从乙盒中任取一球. 令 $X, Y$ 分别表示从甲盒和乙盒中取到的红球个数, 则 $X$ 与 $Y$ 的相关系数", "input": "", "output": "联合分布率 $(X, Y) \\sim\\left(\\begin{array}{cccc}(0,0) & (0,1) & (1,0) & (1,1) \\\\ \\frac{3}{10} & \\frac{1}{5} & \\frac{1}{5} & \\frac{3}{10}\\end{array}\\right), X \\sim\\left(\\begin{array}{cc}0 & 1 \\\\ \\frac{1}{2} & \\frac{1}{2}\\end{array}\\right) Y \\sim\\left(\\begin{array}{cc}0 & 1 \\\\ \\frac{1}{2} & \\frac{1}{2}\\end{array}\\right)$ $\\operatorname{cov}(X, Y)=\\frac{1}{20}, D X=\\frac{1}{4}, D Y=\\frac{1}{4}$, 即 $\\rho_{X Y}=\\frac{1}{5}$\n因此答案为: $\\frac{1}{5}$", "task_type": {"major": ["试题"], "minor": ["考研", "填空题"]}, "domain": ["数学"], "metadata": "year:2021, score:5", "answer_from": "human", "human_verified": false, "copyright": "暂无版权及作者信息"} +{"instruction": "$f(x, y)=x^{2}+2 y^{2}$ 在 $(0,1)$ 处最大的方向导数为", "input": "", "output": "由已知可得 $\\frac{\\partial f}{\\partial x}=2 x, \\frac{\\partial f}{\\partial y}=4 y$, 故 $\\operatorname{grad}(0,1)=(0,4)$, 综上\n$\\left.\\frac{\\partial f}{\\partial l}\\right|_{\\max }=\\sqrt{0^{2}+4^{2}}=4$\n \n因此答案为:4", "task_type": {"major": ["试题"], "minor": ["考研", "填空题"]}, "domain": ["数学"], "metadata": "year:2022, score:5", "answer_from": "human", "human_verified": false, "copyright": "暂无版权及作者信息"} +{"instruction": "$\\int_{1}^{\\mathrm{e}^{2}} \\frac{\\ln x}{\\sqrt{x}} \\mathrm{~d} x=$", "input": "", "output": "$\\int_{1}^{\\mathrm{e}^{2}} \\frac{\\ln x}{\\sqrt{x}} \\mathrm{~d} x=2 \\int_{1}^{\\mathrm{e}^{2}} \\ln x \\mathrm{~d} \\sqrt{x}=2\\left[\\left.\\sqrt{x} \\ln x\\right|_{1} ^{\\mathrm{e}^{2}}-\\int_{1}^{\\mathrm{e}^{2}} \\sqrt{x} \\mathrm{~d}(\\ln x)\\right]$\n$=2\\left[\\left.\\sqrt{x} \\ln x\\right|_{1} ^{\\mathrm{e}^{2}}-\\int_{1}^{\\mathrm{e}^{2}} \\sqrt{x} \\mathrm{~d}(\\ln x)\\right]=2\\left(\\sqrt{\\mathrm{e}^{2}} \\ln \\mathrm{e}^{2}-\\int_{1}^{\\mathrm{e}^{2}} \\frac{1}{\\sqrt{x}} \\mathrm{~d} x\\right)$\n$=2\\left[\\left.\\sqrt{x} \\ln x\\right|_{1} ^{\\mathrm{e}^{2}}-\\int_{1}^{\\mathrm{e}^{2}} \\sqrt{x} \\mathrm{~d}(\\ln x)\\right]=2\\left(2 \\mathrm{e}-\\left.2 \\sqrt{x}\\right|_{1} ^{\\mathrm{e}^{2}}\\right)=4 \\quad\n因此答案为:4", "task_type": {"major": ["试题"], "minor": ["考研", "填空题"]}, "domain": ["数学"], "metadata": "year:2022, score:5", "answer_from": "human", "human_verified": false, "copyright": "暂无版权及作者信息"} +{"instruction": "当 $x \\rightarrow 0$ 时, 函数 $f(x)=a x+b x^{2}+\\ln (1+x)$ 与 $g(x)=e^{x^{2}}-\\cos x$ 是等价无穷小,则 $a b=$", "input": "", "output": "$\\lim _{x \\rightarrow 0} \\frac{f(x)}{g(x)}=\\lim _{x \\rightarrow 0} \\frac{a x+b x^{2}+\\ln (1+x)}{e^{x^{2}}-\\cos x}=\\lim _{x \\rightarrow 0} \\frac{a x+b x^{2}+x-\\frac{1}{2} x^{2}+o\\left(x^{2}\\right)}{1+x^{2}+o\\left(x^{2}\\right)-\\left[1-\\frac{1}{2} x^{2}+o\\left(x^{2}\\right)\\right]}=1$ ,\n可得 $a+1=0, b-\\frac{1}{2}=\\frac{3}{2}$, 即 $a=-1, b=2$, 故 $a b=-2$.\n\n因此答案为:-2", "task_type": {"major": ["试题"], "minor": ["考研", "填空题"]}, "domain": ["数学"], "metadata": "year:2023, score:5", "answer_from": "human", "human_verified": false, "copyright": "暂无版权及作者信息"} +{"instruction": "曲面 $z=x+2 y+\\ln \\left(1+x^{2}+y^{2}\\right)$ 在点 $(0,0,0)$ 处的切平面方程为", "input": "", "output": "$F(x, y, z)=x+2 y+\\ln \\left(1+x^{2}+y^{2}\\right)-z$ ,\n$\\mathbf{n}=\\left(F_{x}^{\\prime}, F_{y}^{\\prime}, F_{z}^{\\prime}\\right)=\\left(1+\\frac{2 x}{1+x^{2}+y^{2}}, 2+\\frac{2 y}{1+x^{2}+y^{2}},-1\\right)$,\n即在点 $(0,0,0)$ 处的法向量为 $(1,2,-1)$, 即切平面方程为 $x+2 y-z=0$.\n\n因此答案为:$x+2 y-z=0$", "task_type": {"major": ["试题"], "minor": ["考研", "填空题"]}, "domain": ["数学"], "metadata": "year:2023, score:5", "answer_from": "human", "human_verified": false, "copyright": "暂无版权及作者信息"} +{"instruction": "设 $f(x)$ 为周期为 2 的周期函数, 且 $f(x)=1-x, x \\in[0,1]$, 若\n$f(x)=\\frac{a_{0}}{2}+\\sum_{n=1}^{\\infty} a_{n} \\cos n \\pi x$, 则 $\\sum_{n=1}^{\\infty} a_{2 n}=__$?", "input": "", "output": "由 $f(x)$ 展开为余弦级数知, $f(x)$ 为偶函数. 由傅里叶系数计算公式有\n$$\n\\begin{aligned}\na_n= & 2 \\int_0^1(1-x) \\cos n \\pi x d x \\\\\n& =2\\left(\\int_0^1 \\cos n \\pi x d x-\\int_0^1 x \\cos n \\pi x d x\\right) \\\\\n& =2\\left(\\left.\\frac{1}{n \\pi} \\sin n \\pi x\\right|_0 ^1-\\frac{1}{n \\pi} \\int_0^1 x d \\sin n \\pi x\\right) \\\\\n& =-\\frac{2}{n \\pi} \\int_0^1 x d \\sin n \\pi x \\\\\n& =\\frac{-2}{n \\pi}\\left(\\left.x \\sin n \\pi x\\right|_0 ^1-\\int_0^1 \\sin n \\pi x d x\\right) \\\\\n& =\\frac{2}{n \\pi} \\int_0^1 \\sin n \\pi x d x \\\\\n& =\\left.\\frac{-2}{n^2 \\pi^2} \\cos n \\pi x\\right|_0 ^1 \\\\\n& =\\frac{-2}{n^2 \\pi^2}(\\cos n \\pi-1) .\n\\end{aligned}\n$$\n\n故 $a_{2 n}=\\frac{-1}{2 n^2 \\pi^2}(\\cos 2 n \\pi-1)=\\frac{-1}{2 n^2 \\pi^2}(1-1)=0$\n因此答案为:0", "task_type": {"major": ["试题"], "minor": ["考研", "填空题"]}, "domain": ["数学"], "metadata": "year:2023, score:5", "answer_from": "human", "human_verified": false, "copyright": "暂无版权及作者信息"} +{"instruction": "设连续函数 $f(x)$ 满足 $f(x+2)-f(x)=x, \\int_{0}^{2} f(x) d x=0$, 则 $\\int_{1}^{3} f(x) d x=$__?", "input": "", "output": "$\\begin{aligned} & \\int_1^3 f(x) d x=\\int_1^2 f(x) d x+\\int_2^3 f(x) d x \\\\ & =\\int_1^2 f(x) d x+\\int_0^1 f(t+2) d t(\\text { 令 } x=t+2) \\\\ & =\\int_1^2 f(x) d x+\\int_0^1 f(x+2) d x \\\\ & =\\int_1^2 f(x) d x+\\int_0^1[f(x)+x] d x \\\\ & =\\int_1^2 f(x) d x+\\int_0^1 f(x) d x+\\int_0^1 x d x \\\\ & =\\int_0^2 f(x) d x+\\int_0^1 x d x \\\\ & =\\int_0^1 x d x \\\\ & =\\frac{1}{2} \\\\ & \\end{aligned}$\n因此答案为:$\\frac{1}{2}$", "task_type": {"major": ["试题"], "minor": ["考研", "填空题"]}, "domain": ["数学"], "metadata": "year:2023, score:5", "answer_from": "human", "human_verified": false, "copyright": "暂无版权及作者信息"} +{"instruction": "已知向量 $\\alpha_{1}=\\left(\\begin{array}{l}1 \\\\ 0 \\\\ 1 \\\\ 1\\end{array}\\right), \\alpha_{2}=\\left(\\begin{array}{l}-1 \\\\ -1 \\\\ 0 \\\\ 1\\end{array}\\right), \\alpha_{3}=\\left(\\begin{array}{l}0 \\\\ 1 \\\\ -1 \\\\ 1\\end{array}\\right), \\beta=\\left(\\begin{array}{l}1 \\\\ 1 \\\\ 1 \\\\ -1\\end{array}\\right), \\gamma=k_{1} \\alpha_{1}+k_{2} \\alpha_{2}+k_{3} \\alpha_{3}$,\n若 $\\gamma^{T} \\alpha_{i}=\\beta^{T} \\alpha_{i}(i=1,2,3)$, 则 $k_{1}^{2}+k_{2}^{2}+k_{3}^{2}=$", "input": "", "output": "$\\gamma^{T} \\alpha_{1}=\\beta^{T} \\alpha_{1}=1 \\Rightarrow k_{1} \\alpha_{1}^{T} \\alpha_{1}+k_{2} \\alpha_{2}^{T} \\alpha_{1}+k_{3} \\alpha_{3}^{T} \\alpha_{1}=1 \\Rightarrow k_{1} \\cdot 3+k_{2} \\cdot 0+k_{3} \\cdot 0=1 \\Rightarrow k_{1}=\\frac{1}{3}$.\n同理 $k_{2}=-1, k_{3}=-\\frac{1}{3}$.\n所以, $k_{1}^{2}+k_{2}^{2}+k_{3}^{2}=\\frac{11}{9}$.\n\n因此答案为:$\\frac{11}{9}$", "task_type": {"major": ["试题"], "minor": ["考研", "填空题"]}, "domain": ["数学"], "metadata": "year:2023, score:5", "answer_from": "human", "human_verified": false, "copyright": "暂无版权及作者信息"} +{"instruction": "设随机变量 $X$ 与 $Y$ 相互独立, 且 $X \\sim B\\left(1, \\frac{1}{3}\\right), Y \\sim B\\left(2, \\frac{1}{2}\\right)$ 则 $P\\{X=Y\\}=$", "input": "", "output": "因为 $X \\sim B\\left(1, \\frac{1}{3}\\right)$, 所以 $X=0,1$;\n$Y \\sim B\\left(2, \\frac{1}{2}\\right)$, 所以 $Y=0,1,2$.\n\n又因为 $X$ 与 $Y$ 相互独立, 所以\n$$\n\\begin{aligned}\nP\\{X=Y\\}=P\\{X & =0, Y=0\\}+P\\{X=1, Y=1\\} \\\\\n& =P\\{X=0\\} P\\{Y=0\\}+P\\{X=1\\} P\\{Y=1\\} \\\\\n& =\\frac{2}{3} C_2^0\\left(\\frac{1}{2}\\right)^2+\\frac{1}{3} C_2^1\\left(\\frac{1}{2}\\right)^2=\\frac{1}{3} .\n\\end{aligned}\n$$\n因此答案为:$\\frac{1}{3}$", "task_type": {"major": ["试题"], "minor": ["考研", "填空题"]}, "domain": ["数学"], "metadata": "year:2023, score:5", "answer_from": "human", "human_verified": false, "copyright": "暂无版权及作者信息"} +{"instruction": "1.下列 $0.10 \\mathrm{~mol} \\cdot \\mathrm{kg}^{-1}$ 溶液性质的比较, 正确的是 ()。\nA. 蒸气压: $\\mathrm{NaCl}>\\mathrm{C}_{6} \\mathrm{H}_{12} \\mathrm{O}_{6}$ (葡萄糖)\nB. 沸点: $\\mathrm{K}_{2} \\mathrm{SO}_{4}>\\mathrm{C}_{6} \\mathrm{H}_{12} \\mathrm{O}_{6}$ (葡萄糖)\nC. 凝固点: $\\mathrm{CH}_{4} \\mathrm{~N}_{2} \\mathrm{O}$ (尿素) $>\\mathrm{C}_{6} \\mathrm{H}_{12} \\mathrm{O}_{6}$ (葡萄糖)\nD. 渗透压: $\\mathrm{NaCl}=\\mathrm{C}_{6} \\mathrm{H}_{12} \\mathrm{O}_{6}$ (葡菊糖)", "input": "", "output": "本题考查稀溶液的依数性, 包括蒸气压下降、沸点㺪高、凝固点下降和渗透压。 $\\mathrm{NaCl}$ 和 $\\mathrm{K}_{2} \\mathrm{SO}_{4}$ 均为强电解质, 溶液中离子总浓度分别约为葡葏糖浓度的 2 倍和 3 倍, 故答案 $\\mathrm{A}$ 中 $\\mathrm{NaCl}$ 溶液的蒸气压下降更多; 答案 $\\mathrm{C}$ 中尿素和葡萄糖均为非电解质, 二者浓度相同时, 凝固点的下降程度相同, 即凝固点相同; 答案 $\\mathrm{D}$ 中 $\\mathrm{NaCl}$ 溶液的渗透压更高。\n\n因此答案为:B", "task_type": {"major": ["试题"], "minor": ["考研", "选择题"]}, "domain": ["化学(农学类)"], "metadata": "year:2022, score:2", "answer_from": "human", "human_verified": false, "copyright": "暂无版权及作者信息"} +{"instruction": "已知反应 $2 \\mathrm{X}(\\mathrm{g})+\\mathrm{Y}(\\mathrm{g})=3 \\mathrm{Z}(\\mathrm{g})$, 在一定温度下 $\\mathrm{X} 、 \\mathrm{Y}$ 的初始浓度和初始反应速率的数据如下:\n\\begin{center}\n\\begin{tabular}{c|c|c|c}\n\\hline\n & $c(\\mathrm{X}) /\\left(\\mathrm{mol} \\cdot \\mathrm{L}^{-1}\\right)$ & $c(\\mathrm{Y}) /\\left(\\mathrm{mol} \\cdot \\mathrm{L}^{-1}\\right)$ & $v /\\left(\\mathrm{mol} \\cdot \\mathrm{L}^{-1} \\cdot \\mathrm{s}^{-1}\\right)$ \\\\\n\\hline\n1 & 0.20 & 0.30 & $2.0 \\times 10^{-4}$ \\\n2 & 0.20 & 0.60 & $8.0 \\times 10^{-4}$ \\\n3 & 0.30 & 0.60 & $8.0 \\times 10^{-4}$ \\\\\n\\hline\n\\end{tabular}\n\\end{center}\n则该反应的速率方程为 ( )。\nA. $v=k c^{2}(\\mathrm{X})$\nB. $v=k c^{2}(\\mathrm{Y})$\nC. $v=k c(\\mathrm{X}) \\cdot c^{2}(\\mathrm{Y})$\nD. $v=k c^{2}(\\mathrm{X}) \\cdot c(\\mathrm{Y})$", "input": "", "output": "对于基元反应, 可根据反应方程式写出反应速率方程式。而非基元反应中各反应物的反应级数只能通过实验进行测定。\n对比第 1 和第 2 组数据, 反应物 $\\mathrm{X}$ 浓度相等, 当反应物 $\\mathrm{Y}$ 的浓度增大为原来的 2 倍时, 反应速率增大为原来的 4 倍, 故反应物 $Y$ 的反应级数为 2 ;\n对比第 2 和第 3 组数据, 反应物 $\\mathrm{Y}$ 浓度相等, 当反应物 $\\mathrm{X}$ 的浓度增大为原来的 1.5 倍时, 反应速率保持不变, 故反应物 $\\mathrm{X}$ 的反应级数为 0\n\n因此答案为:B", "task_type": {"major": ["试题"], "minor": ["考研", "选择题"]}, "domain": ["化学(农学类)"], "metadata": "year:2022, score:2", "answer_from": "human", "human_verified": false, "copyright": "暂无版权及作者信息"} +{"instruction": "在 EDTA 配位滴定法中, 金属指示剂 (In) 应具备的条件是 ( )。\n(1) MIn 易溶于水\n(2) MIn 的稳定性大于 $\\mathrm{MY}$ 的稳定性\n(3) MIn 与 In 的颜色显著不同\n(4)显色反应灵敏、迅速,有良好的变色可逆性\nA. (1)(2) (3)\nB. (1)(3)(4)\nC. (2) (3) (4)\nD. (1)(2) (4)", "input": "", "output": "本题考察配位滴定中,金属指示剂应具备的条件。\n\n因此答案为:B", "task_type": {"major": ["试题"], "minor": ["考研", "选择题"]}, "domain": ["化学(农学类)"], "metadata": "year:2022, score:2", "answer_from": "human", "human_verified": false, "copyright": "暂无版权及作者信息"} +{"instruction": "在 $20{ }^{\\circ} \\mathrm{C}$ 和 $100{ }^{\\circ} \\mathrm{C}$ 时, $\\mathrm{PbCl}_{2}$ 的溶解度分别是 $0.99 \\mathrm{~g} / 100 \\mathrm{~g} \\mathrm{H}_{2} \\mathrm{O}$ 和 $3.34 \\mathrm{~g} / 100 \\mathrm{~g} \\mathrm{H}_{2} \\mathrm{O}$, 则反应 $\\mathrm{PbCl}_{2}(\\mathrm{~s})=\\mathrm{Pb}^{2+}(\\mathrm{aq})+2 \\mathrm{Cl}^{-}(\\mathrm{aq})$ 的 ()。\nA. $\\Delta_{\\mathrm{r}} H_{\\mathrm{m}}^{\\ominus}>0$\nB. $\\Delta_{\\mathrm{r}} H_{\\mathrm{m}}^{\\ominus}<0$\nC. $\\Delta_{\\mathrm{r}} S_{\\mathrm{m}}^{\\ominus}=0$\nD. $\\Delta_{\\mathrm{r}} S_{\\mathrm{m}}^{\\ominus}<0$", "input": "", "output": "本题考查影响化学平衡移动的因素问题。据题意, 随温度升高, $\\mathrm{PbCl}_{2}$ 溶解度增大, 说明标准平衡常数增大。温度升高, 平衡问吸热方向移动, 因此该反应为吸热反应。\n\n因此答案为:A", "task_type": {"major": ["试题"], "minor": ["考研", "选择题"]}, "domain": ["化学(农学类)"], "metadata": "year:2022, score:2", "answer_from": "human", "human_verified": false, "copyright": "暂无版权及作者信息"} +{"instruction": "若反应 $2 \\mathrm{~A}+2 \\mathrm{~B}=3 \\mathrm{D}$ 的 $E_{\\mathrm{n}}($ 正 $)=m \\mathrm{~kJ} \\cdot \\mathrm{mol}^{-1} 、 E_{\\mathrm{a}}$ (逆) $=n \\mathrm{~kJ} \\cdot \\mathrm{mol}^{-1}$, 则反应的 $\\Delta_{\\mathrm{r}} H_{\\mathrm{m}}$ 等于( )。\nA. $(2 m-3 n) \\mathrm{kJ} \\cdot \\mathrm{mol}^{-1}$\nB. $(3 n-2 m) \\mathrm{kJ} \\cdot \\mathrm{mol}^{-1}$\nC. $(m-n) \\mathrm{kJ} \\cdot \\mathrm{mol}^{-1}$\nD. $(n-m) \\mathrm{kJ} \\cdot \\mathrm{mol}^{-1}$", "input": "", "output": "本题考查正逆反应的活化能和反应热之间的关系。\n\n因此答案为:C", "task_type": {"major": ["试题"], "minor": ["考研", "选择题"]}, "domain": ["化学(农学类)"], "metadata": "year:2022, score:2", "answer_from": "human", "human_verified": false, "copyright": "暂无版权及作者信息"} +{"instruction": "既能标定 $\\mathrm{KMnO}_{4}$ 溶液又能标定 $\\mathrm{NaOH}$ 溶液的基准物是 ()。\nA. 硼砂\nB. $\\mathrm{Na}_{2} \\mathrm{C}_{2} \\mathrm{O}_{4}$\nC. 邻苯二甲酸氢钾\nD. $\\mathrm{H}_{2} \\mathrm{C}_{2} \\mathrm{O}_{4} \\cdot 2 \\mathrm{H}_{2} \\mathrm{O}$", "input": "", "output": "4 个物质都可以作为基准物质,草酸既是较强的酸又是还原剂, 因此选 $\\mathrm{D}$ 。", "task_type": {"major": ["试题"], "minor": ["考研", "选择题"]}, "domain": ["化学(农学类)"], "metadata": "year:2022, score:2", "answer_from": "human", "human_verified": false, "copyright": "暂无版权及作者信息"} +{"instruction": "按有效数字运算规则, $\\frac{0.0421 \\times 0.46 \\times 1.450 \\times 10^{2}}{1.0500}$ 的计算结果为 ()。\nA. 2.7\nB. 2.67\nC. 2.674\nD. 2.6744", "input": "", "output": "本题考查有效数字相关知识。根据乘除计算规则, 计算结果的有效数字位数应与算式中有效数字位数最少的数据相同,数据 0.46 中有 2 位有效数字,因此计算结果的有效数字应保留 2 位。\n\n因此答案为:A", "task_type": {"major": ["��题"], "minor": ["考研", "选择题"]}, "domain": ["化学(农学类)"], "metadata": "year:2022, score:2", "answer_from": "human", "human_verified": false, "copyright": "暂无版权及作者信息"} +{"instruction": "已知 $\\mathrm{Ag}_{2} \\mathrm{~S} 、 \\mathrm{CuS}$ 和 $\\mathrm{MnS}$ 的 $K_{\\mathrm{sp}}^{\\ominus}$ 分别为 $6.3 \\times 10^{-50} 、 6.3 \\times 10^{-36}$ 和 $2.5 \\times 10^{-13}$ 。向浓度均为 $0.010 \\mathrm{~mol} \\cdot \\mathrm{L}^{-1}$ 的 $\\mathrm{Ag}^{+} 、 \\mathrm{Cu}^{2+}$ 和 $\\mathrm{Mn}^{2+}$ 的混合溶液中滴加 $\\mathrm{Na}_{2} \\mathrm{~S}$ 溶液, 沉淀产生的先后顺序是 ( )。\nA. $\\mathrm{Ag}_{2} \\mathrm{~S}, \\mathrm{CuS}, \\mathrm{MnS}$\nB. $\\mathrm{Ag}_{2} \\mathrm{~S}, \\mathrm{MnS}, \\mathrm{CuS}$\nC. $\\mathrm{CuS}, \\mathrm{MnS} 、 \\mathrm{Ag}_{2} \\mathrm{~S}$\nD. $\\mathrm{CuS} 、 \\mathrm{Ag}_{2} \\mathrm{~S} 、 \\mathrm{MnS}$", "input": "", "output": "本题考查溶度积原理和分步沉淀等相关知识。不同类型的沉淀, 不能直接根据 $K_{s p}^{\\ominus}$ 判断沉淀的先后顺序。离子积 $Q$ 先达到其溶度积 $K_{s p}^{\\ominus}$ 者即可先沉淀, 或生成沉淀时, 所需沉淀剂更少的可先沉淀。\n$\\mathrm{Ag}_{2} \\mathrm{~S}$ 沉淀所需的最低的沉淀剂浓度:\n$c_{1}\\left(\\mathrm{~S}^{2-}\right) / c^{\\ominus}=\frac{K_{\\mathrm{sp}}^{\\ominus}\\left(\\mathrm{Ag}_{2} \\mathrm{~S}\right)}{\\left[c\\left(\\mathrm{Ag}^{1-}\right) / c^{\\ominus}\right]^{2}}=\frac{6.3 \times 10^{-50}}{0.010^{2}}=6.3 \times 10^{-16}$\n$\\operatorname{CuS}$ 沉淀: $c_{2}\\left(\\mathrm{~S}^{2-}\right) / c^{\\ominus}=\frac{K_{\\mathrm{sp}}^{\\ominus}(\\mathrm{CuS})}{c\\left(\\mathrm{Cu}^{2+}\right) / c^{\\ominus}}=\frac{6.3 \times 10^{-36}}{0.010}=6.3 \times 10^{-3.4}$\n$\\mathrm{MnS}$ 沉淀: $c_{3}\\left(\\mathrm{~S}^{2-}\right) / c^{\\ominus}=\frac{K_{\\mathrm{sp}}^{\\ominus}(\\mathrm{MnS})}{c\\left(\\mathrm{Mn}^{2+}\right) / c^{\\ominus}}=\frac{2.5 \times 10^{-13}}{0.010}=2.5 \times 10^{-11}$\n\n因此答案为:A", "task_type": {"major": ["试题"], "minor": ["考研", "选择题"]}, "domain": ["化学(农学类)"], "metadata": "year:2022, score:2", "answer_from": "human", "human_verified": false, "copyright": "暂无版权及作者信息"} +{"instruction": "将 $0.10 \\mathrm{~mol} \\cdot \\mathrm{L}^{-1}$ 的 $\\mathrm{HAc}\\left(\\mathrm{p} K_{\\mathrm{n}}^{\\ominus}=4.75\\right)$ 溶液稀释一倍, 则 $(\\quad)$ 。\nA. $\\alpha$ 和 $\\mathrm{pH}$ 都增大\nB. $\\alpha$ 和 $\\mathrm{pH}$ 都减少\nC. $\\alpha$ 增大, $\\mathrm{pH}$ 减小\nD. $\\alpha$ 减少, $\\mathrm{pH}$ 增大", "input": "", "output": "本题考查酸碱平衡的移动和稀释定律。\n$\\mathrm{HAc}$ 水溶液中, $\\mathrm{HAc}(\\mathrm{aq})+\\mathrm{H}_{2} \\mathrm{O}(1)=\\mathrm{H}_{3} \\mathrm{O}^{+}(\\mathrm{aq})+\\mathrm{Ac}^{\\cdots}(\\mathrm{aq})$\n稀释一倍后, 浓度下降为原来的 $1 / 2$,\n则反应商 $Q=\frac{\\left[c\\left(\\mathrm{H}^{+}\right) / c^{\\ominus}\right] \\cdot\\left[c\\left(\\mathrm{Ac}^{-}\right) / c^{\\ominus}\right]}{c(\\mathrm{HAc}) / c^{\\ominus}}=\frac{1}{2} K_{a}^{\\ominus}(\\mathrm{HAc})0$, 则反应正向不能自发, 逆向自发; 若 $\\Delta_{\\mathrm{r}} G_{\\mathrm{m}}=0$, 则反应处于化学平衡状态。\n\n因此答案为:B", "task_type": {"major": ["试题"], "minor": ["考研", "选择题"]}, "domain": ["化学(农学类)"], "metadata": "year:2022, score:2", "answer_from": "human", "human_verified": false, "copyright": "暂无版权及作者信息"} +{"instruction": "电池 $(-) \\mathrm{Pt}\\left|\\mathrm{H}_{2}\\left(p_{1}\\right)\\right| \\mathrm{H}^{+}\\left(c_{0}\\right) \\| \\mathrm{H}^{+}\\left(c_{0}\\right)\\left|\\mathrm{H}_{2}\\left(p_{2}\\right)\\right| \\mathrm{Pt}(+)$ 的电动势为 $E$, 下列关系正确的是 ( ) 。\nA. $E=0, p_{1}0, p_{1}0, p_{1}=p_{2}$\nC. $E>0, p_{1}>p_{2}$", "input": "", "output": "本题考查能斯特方程的书写和应用。\n正负极均为氢电极, 电动势 $E=\\varphi_{+}-\\varphi_{-}$\n电极反应均为: $2 \\mathrm{H}^{+}+2 \\mathrm{e} \rightarrow \\mathrm{H}_{2}$\n其能斯特方程: $\\varphi=\\varphi^{\\ominus}+\frac{2.303 R T}{2 F} \\lg \frac{\\left[c\\left(\\mathrm{H}^{+}\right) / c^{\\ominus}\right]^{2}}{p\\left(\\mathrm{H}_{2}\right) / p^{\\ominus}}$\n正负极中 $\\mathrm{H}^{+}$浓度相同, 若 $E=0$, 则 $p_{1}=p_{2}$; 若 $E>0$, 则 $p_{1}7$\nC. 有旋光性\nD. 只有一个氨基", "input": "", "output": "赖氨酸为碱性氨基酸, 含有 2 个氨基。\n\n因此答案为:D", "task_type": {"major": ["试题"], "minor": ["考研", "选择题"]}, "domain": ["化学(农学类)"], "metadata": "year:2022, score:2", "answer_from": "human", "human_verified": false, "copyright": "暂无版权及作者信息"} +{"instruction": "可区别 $\\mathrm{CH}_{3} \\mathrm{CH}_{2} \\mathrm{CH}_{2} \\mathrm{NH}_{2} 、\\left(\\mathrm{CH}_{3} \\mathrm{CH}_{2} \\mathrm{CH}_{2}\\right)_{2} \\mathrm{NH} 、\\left(\\mathrm{CH}_{3} \\mathrm{CH}_{2} \\mathrm{CH}_{2}\\right)_{3} \\mathrm{~N}$ 的试剂是 ( )。\nA. $\\mathrm{HCl}$\nB. $\\mathrm{H}_{2} \\mathrm{SO}_{4}$\nC. $\\mathrm{HNO}_{3}$\nD. $\\mathrm{HNO}_{2}$", "input": "", "output": "伯、仲、叔胺与盐酸、硫酸、硝酸的反应相近, 都是酸碱反应。亚硝酸与伯胺反应放出氮气,与仲胺反应形成 $N$-亚硝基化合物,与叔胺是酸碱反应,因而答案是 D。\n\n因此答案为:D", "task_type": {"major": ["试题"], "minor": ["考研", "选择题"]}, "domain": ["化学(农学类)"], "metadata": "year:2022, score:2", "answer_from": "human", "human_verified": false, "copyright": "暂无版权及作者信息"} +{"instruction": "化合物 (1)丙酸乙酯、(2)丙酸酐、(3)丙酰氯与苯胺反应的活性顺序是 () 。\nA. (3) $>$ (1) $>$ (2)\nB. (3) $>$ (2) $>$ (1)\nC. (1) $>$ (3) $>$ (2)\nD. (1) $>$ (2) $>$ (3)", "input": "", "output": "酰化试剂与苯胺反应的活性顺序一般为酰氯>酸酐>酯\n\n因此答案为:B", "task_type": {"major": ["试题"], "minor": ["考研", "选择题"]}, "domain": ["化学(农学类)"], "metadata": "year:2022, score:2", "answer_from": "human", "human_verified": false, "copyright": "暂无版权及作者信息"} +{"instruction": "下列化合物不含羧基官能团的是 ()。\nA. 苦味酸\nB. 酒石酸\nC. 草酸\nD. 水杨酸", "input": "", "output": "苦味酸是 $2,4,6$-三硝基苯酚, 没有羧基。\n\n因此答案为:A", "task_type": {"major": ["试题"], "minor": ["考研", "选择题"]}, "domain": ["化学(农学类)"], "metadata": "year:2022, score:2", "answer_from": "human", "human_verified": false, "copyright": "暂无版权及作者信息"} +{"instruction": "下列化合物沸点最低的是 ( )。\nA. 丁酮\nB. 丁胺\nC. 1-丁烯\nD. 丁酸", "input": "", "output": "同样是含有 4 个碳的有机物, 丁酸和丁胺都有分子间氢键, 沸点较高。丁酮有一定极性,分子间作用力也较大; 1-丁烯极性较小,分子间作用力最小,沸点最低。\n\n因此答案为:C", "task_type": {"major": ["试题"], "minor": ["考研", "选择题"]}, "domain": ["化学(农学类)"], "metadata": "year:2022, score:2", "answer_from": "human", "human_verified": false, "copyright": "暂无版权及作者信息"} +{"instruction": "果糖在稀碱溶液中发生差向异构化, 平衡混合物中有 ( )。\nA. 核糖\nB. 脱氧核糖\nC. 半乳糖\nD. 葡萄糖", "input": "", "output": "果糖在稀碱溶液中发生差向异构化,平衡混合物中有葡若糖和甘露糖。\n\n因此答案为:D", "task_type": {"major": ["试题"], "minor": ["考研", "选择题"]}, "domain": ["化学(农学类)"], "metadata": "year:2022, score:2", "answer_from": "human", "human_verified": false, "copyright": "暂无版权及作者信息"} +{"instruction": "下列溶剂可用于芐基溴与金属镁反应生成芐基溴化镁的是 ()。\nA. 无水乙醇\nB. 无水二乙胺\nC. 无水乙醚\nD. 无水乙酸乙酯", "input": "", "output": "格式试剂比较活泼,制备时需要使用不与之发生反应的惰性溶剂乙梄。\n\n因此答案为:C", "task_type": {"major": ["试题"], "minor": ["考研", "选择题"]}, "domain": ["化学(农学类)"], "metadata": "year:2022, score:2", "answer_from": "human", "human_verified": false, "copyright": "暂无版权及作者信息"} +{"instruction": "下列结构为油酸的是 ( )。\nA. $\\mathrm{CH}_{3}\\left(\\mathrm{CH}_{2}\\right)_{7} \\mathrm{CH}=\\mathrm{CH}\\left(\\mathrm{CH}_{2}\\right)_{7} \\mathrm{COOH}$\nB. $\\mathrm{CH}_{3}\\left(\\mathrm{CH}_{2}\\right)_{4} \\mathrm{CH}=\\mathrm{CHCH}_{2} \\mathrm{CH}=\\mathrm{CH}\\left(\\mathrm{CH}_{2}\\right)_{7} \\mathrm{COOH}$\nC. $\\mathrm{CH}_{3}\\left(\\mathrm{CH}_{2}\\right)_{16} \\mathrm{COOH}$\nD. $\\mathrm{CH}_{3}\\left(\\mathrm{CH}_{2}\\right)_{14} \\mathrm{COOH}$", "input": "", "output": "油酸是单不饱和十八碳脂肪酸。\n\n因此答案为:A", "task_type": {"major": ["试题"], "minor": ["考研", "选择题"]}, "domain": ["化学(农学类)"], "metadata": "year:2022, score:2", "answer_from": "human", "human_verified": false, "copyright": "暂无版权及作者信息"} +{"instruction": "下列程序段的时间复杂度是 ( ) .\nint sum $=\\theta$;\nfor (int $i=1 ; i2)$ 个字符的有限集 $S$, 用二叉树表示 $S$ 的哈夫曼编码集和定长编码集,分别得到二叉树 $T_{1}$ 和 $T_{2}$. 下列叙述中,正确的是().\nA. $T_{1}$ 与 $T_{2}$ 的结点数相同\nB. $T_{1}$ 的高度大于 $T_{2}$ 的高度\nC. 出现频次不同的字符在 $T_{1}$ 中处于不同的层\nD. 出现频次不同的字符在 $T_{2}$ 中处于相同的层", "input": "", "output": "哈夫曼编码属于不等长编码, 定长编码属于等长编码。就等长编码而言, 其编码长度相同, 且与字符出现频次无关, 因此, 等长编码在二叉树 $T_{2}$ 上必然出现在同一层上。\n\n因此答案为:D", "task_type": {"major": ["试题"], "minor": ["考研", "选择题"]}, "domain": ["计算机"], "metadata": "year:2022, score:2", "answer_from": "human", "human_verified": false, "copyright": "暂无版权及作者信息"} +{"instruction": "对于无向图 $G=(V, E)$, 下列选项中, 正确的是()。\nA. 当 $|V|>|E|$ 时, $G$ 一定是连通的\nB. 当 $|V|<|E|$ 时, $G$ 一定是连通的\nC. 当 $|V|=|E|-1$ 时, $G$ 一定是不连通的\nD. 当 $|V>| E \\mid+1$ 时, $G$ 一定是不连通的", "input": "", "output": "一个有 $|\\mathrm{V}|$ 个顶点的连通图应至少有 $|\\mathrm{V}|-1$ 条边, 当 $|E|<|\\mathrm{V}|-1$ 时, 即 $|\\mathrm{E}|+1<|\\mathrm{V}|$ 时, 一定是非连通图。所以 $\\mathrm{D}$ 正确。\n\n因此答案为:D", "task_type": {"major": ["试题"], "minor": ["考研", "选择题"]}, "domain": ["计算机"], "metadata": "year:2022, score:2", "answer_from": "human", "human_verified": false, "copyright": "暂无版权及作者信息"} +{"instruction": "下列因素中, 影响散列(哈希)方法平均查找长度是()。\nI. 装填因子\nII.散列函数\nIII.冲突解决策略\nA. 仅 I、II\nB. 仅 I、III\nC. 仅 II 、 III\nD. I、 II 、 III", "input": "", "output": "平均查找长度是查找算法在查找成功时平均查找长度和查找不成功时平均查找长度之和。查找成功的平均查找长度指的是为确定记录在查找表中的位置, 需和给定值进行比较的关键字个数的期望值。查找不成功平均查找长度是查找不成功时和给定值进行比较的关键字个数的期望值。而查找过程中需和给定值比较的关键字个数取决于:(1)哈希函数;(2)处理冲突的方法; (3)哈希表装填因子 $a=n / m$; $n$ :表中填入记录数; $m$ :哈希表长。\n\n因此答案为:D", "task_type": {"major": ["试题"], "minor": ["考研", "选择题"]}, "domain": ["计算机"], "metadata": "year:2022, score:2", "answer_from": "human", "human_verified": false, "copyright": "暂无版权及作者信息"} +{"instruction": "使用二路归并排序对含 $\\mathrm{n}$ 个元素的数组 $M$ 进行排序时, 二路归并操作的功能是()。\nA. 将两个有序表合并为一个新的有序表\nB. 将 $\\mathrm{M}$ 划分为两部分, 两部分的元素个数大致相等\n$C$. 将 $M$ 划分为 $n$ 个部分, 每个部分中仅含有一个元素\nD. 将 $M$ 划分为两部分, 一部分元素的值均小于另一部分元素的值", "input": "", "output": "归并排序算法的基本思想:将两个或以上的有序子序列归并为一个有序序列。\n\n因此答案为:A", "task_type": {"major": ["试题"], "minor": ["考研", "选择题"]}, "domain": ["计算机"], "metadata": "year:2022, score:2", "answer_from": "human", "human_verified": false, "copyright": "暂无版权及作者信息"} +{"instruction": "某计算机主频为 $1 \\mathrm{GHz}$, 程序 $\\mathrm{P}$ 运行过程中, 共执行了 10000 条指令, 其中, $80 \\%$ 的指令执行平均需 1 个时钟周期, $20 \\%$ 的指令执行平均需 10 个时钟周期. 程序 P 的平均 CPI 和 CPU 执行时间分别是 ( ) .\nA. $2.8,28 \\mu \\mathrm{s}$\nB. $28,28 \\mu \\mathrm{s}$\nC. $2.8,28 \\mathrm{~ms}$\nD. $28,28 \\mathrm{~ms}$", "input": "", "output": "P 程序执行需要的时钟周期数为 $10000^{*}\\left(0.8^{*} 1+0.2^{*} 10\\right)=28000, \\mathrm{CPU}$ 的执行时间 $=$ 时钟周期数 $/ \\mathrm{f}=28000 / 1 \\mathrm{G}=28^{*} 10^{-6} \\mathrm{~s}=28 \\mu \\mathrm{s} ; \\mathrm{P}$ 的平均 $\\mathrm{CPI}=$ 程序执行所需时钟周期数 $/$ 程序中所包含的指令条数 $=28000 / 10000=2.8$ 。\n\n因此答案为:A", "task_type": {"major": ["试题"], "minor": ["考研", "选择题"]}, "domain": ["计算机"], "metadata": "year:2022, score:2", "answer_from": "human", "human_verified": false, "copyright": "暂无版权及作者信息"} +{"instruction": "-0.4375 的 I EEE754 单精度浮点数表示为 ( ).\nA.BEE0 $0000 \\mathrm{H}$\nB.BF60 0000H\nC.BF70 0000H\nD.COE $00000 \\mathrm{H}$", "input": "", "output": "$(-0.4375)_{10}=(-1)^{1 *}(0.0111)_{2}=(-1)^{1 *}(1.11)_{2} * 2^{-2}$, 所以 $\\mathrm{S}=1, \\mathrm{M}=(11000 \\ldots 00)_{2}, \\mathrm{E}=-$ $2+127=125=(01111101)_{2},-0.4375$ 的 IEEE 754 单精度浮点数格式为\n$(10111110111000 \\ldots 00)_{2}=(\\text { BEE00000 })_{160}$\n\n因此答案为:A", "task_type": {"major": ["试题"], "minor": ["考研", "选择题"]}, "domain": ["计算机"], "metadata": "year:2022, score:2", "answer_from": "human", "human_verified": false, "copyright": "暂无版权及作者信息"} +{"instruction": "若计算机主存地址为 32 位, 按字节编址, 某 Cache 的数据区容量为 $32 \\mathrm{~KB}$, 主存块大小为 $64 \\mathrm{~B}$, 采用 8 路组相联映射方式, 该 Cache 中比较器的个数和位数分别为 () .\nA. 8,20\nB. 8,23\nC. 64,20\nD. 64,23", "input": "", "output": "组相联映射地址包括标记、组号和块内地址。cache 容量 $32 \\mathrm{~KB}$, 每块/行大小为 $64 \\mathrm{~B}$,所以 cache 共有 $32 \\mathrm{~KB} \\div 64 \\mathrm{~B}=512$ 块/行, 又因采用 8 路组相联映射, 所以 cache 的结构为 64 组 $(512 / 8)$, 组号位数为 $6\\left(2^{6}=64\\right)$, 每组 8 行。主存块大小为 $64 \\mathrm{~B}$, 所以块内地址为 6 位 $\\left(2^{6}=64\\right)$,地址线共 32 位, 标记位 $=32-6-6=20$ 。 8 路组相联映射, 所以需要 8 个比较器; 标记位为 20 ,所以比较器位数为 20 。\n\n因此答案为:A", "task_type": {"major": ["试题"], "minor": ["考研", "选择题"]}, "domain": ["计算机"], "metadata": "year:2022, score:2", "answer_from": "human", "human_verified": false, "copyright": "暂无版权及作者信息"} +{"instruction": "某内存条包含 8 个 $8192 \\times 8192 \\times 8$ 位的 DRAM 芯片, 按字节编址, 支持突发(b ust)传送方式, 对应存储器总线宽度为 64 位, 每个 DRAM 芯片内有一个行缓冲区(r owb uffer 。) 下列关于该内存条的柇述中,不正确的是()。\nA. 内存条的容量为 $512 \\mathrm{MB}$\nB. 采用多模块交叉编址方式\nC.芯片的地址引脚为 26 位\nD. 芯片内行缓冲有 $8192 \\times 8$ 位", "input": "", "output": "内存条的容量为 $8 * 22^{13 *} 2^{13 *} 8 \\mathrm{bits}=512 \\mathrm{MB}$, 采用多模块交叉编址方式。DRAM 的行、列地址分时传送, 容量为 $64 \\mathrm{MB}$ 需要 26 条地址线, 因此芯片的地址引脚为 13 。\n\n因此答案为:C", "task_type": {"major": ["试题"], "minor": ["考研", "选择题"]}, "domain": ["计算机"], "metadata": "year:2022, score:2", "answer_from": "human", "human_verified": false, "copyright": "暂无版权及作者信息"} +{"instruction": "下列选项中, 属于指令集体系结构(ISA)规定的内容是()。\nI. 指令字格式和指令类型\nIII. 通用寄存器个数和位数\nII .CPU 的时钟周期\nIV.加法器的进位方式\nA. 仅 I、II\nB. 仅 I、III\nC. 仅 II 、IV\nD. 仅 I、III、IV", "input": "", "output": "ISA 规定的内容包括数据类型及格式, 指令格式, 寻址方式和可访问地址空间的大小, 程序可访问的寄存器个数、位数和编号, 控制寄存器的定义, $\\mathrm{I} / \\mathrm{O}$ 空间的编制方式, 中断结构, 机器工作状态的定义和切换, 输入输出结构和数据传送方式, 存储保护方式等。\n\n因此答案为:B", "task_type": {"major": ["试题"], "minor": ["考研", "选择题"]}, "domain": ["计算机"], "metadata": "year:2022, score:2", "answer_from": "human", "human_verified": false, "copyright": "暂无版权及作者信息"} +{"instruction": "设计某指令系统时, 假设采用 16 位定长指令字格式, 操作码使用扩展编码方式, 地址码为 6 位, 包含零地址、一地址和二地址 3 种格式的指令。若二地址指令有 12 条, 一地址指令有 254 条, 则零地址指令的条数最多为 ( ) 。\nA.\nB. 2\nC. 64\nD. 128", "input": "", "output": "指令长度为 16 , 地址码为 6 位, 零地址、一地址和二地址指令对应的操作码位数分别为 $16,16-6=10,16-6^{*} 2=4$, 二地址指令编码剩余 $2^{4}-12=16-12=4$, 一地址指令最多 $4^{*} 2^{10-4}=256$条, 剩余编码 $256-254=2$, 零地址指令最多 $2 * 2^{16-10}=128$ 。\n\n因此答案为:D", "task_type": {"major": ["试题"], "minor": ["考研", "选择题"]}, "domain": ["计算机"], "metadata": "year:2022, score:2", "answer_from": "human", "human_verified": false, "copyright": "暂无版权及作者信息"} +{"instruction": "将高级语言源程序转换为可执行目标文件的主要过程是\nA. 预处理 $\\rightarrow$ 编译 $\\rightarrow$ 汇编 $\\rightarrow$ 链接\nB. 预处理 $\\rightarrow$ 汇编 $\\rightarrow$ 编译 $\\rightarrow$ 链接\nC. 预处理 $\\rightarrow$ 编译 $\\rightarrow$ 链接 $\\rightarrow$ 汇编\nD. 预处理 $\\rightarrow$ 汇编 $\\rightarrow$ 链接 $\\rightarrow$ 编译", "input": "", "output": "预处理程序对高级语言源程序进行文件包含, 宏替换等操作, 编译程序将预处理后的源程序转换为汇编语言代码, 汇编将汇编语言代码转换为目标代码, 链接程序将目标代码连接生成可执行程序。\n\n因此答案为:A", "task_type": {"major": ["试题"], "minor": ["考研", "选择题"]}, "domain": ["计算机"], "metadata": "year:2022, score:2", "answer_from": "human", "human_verified": false, "copyright": "暂无版权及作者信息"} +{"instruction": "下列关于中断 I/ 0 方式的叙述中, 不正确的是 ( ) 。\nA. 适用于键䑤、针式打印机等字符型设备\nB. 外设和主机之间的数据传送通过软件完成\nC. 外设准备数据的时间应小于中断处理时间\nD. 外设为某进程准备数据时 CPU 可运行其他进程", "input": "", "output": "键盘、针式打印机等字符型设备是外设, 与主机之间可以通过中断 $\\mathrm{I} / 0$ 方式进行数据交换, 所以 $\\mathrm{A}$ 是正确的。主机响应中断后转入中断处理程序, 在中断处理程序中进行数据传送, 所以 $\\mathrm{B}$ 是正确的。外设准备数据的同时, 主机在执行其他进程, 并在每条指令执行完后查看中断引脚, 因此, 外设准备数据和主机执行进程是同时进行的, $\\mathrm{D}$ 选项也是正确的。外设数据准备好后才会发出中断请求, 主机发现有中断请求后才转入中断处理程序进行中断处理, 因此外设准备数据的时间会影响整个系统效率, 但是与主机处理中断的时间没有关联, 故而选项 $\\mathrm{C}$ 错误。", "task_type": {"major": ["试题"], "minor": ["考研", "选择题"]}, "domain": ["计算机"], "metadata": "year:2022, score:2", "answer_from": "human", "human_verified": false, "copyright": "暂无版权及作者信息"} +{"instruction": "下列关于并行处理技术的叙述中,不正确的是()。\nA. 多核处理器属于 MIMD 结构\nB. 向量处理器属于 SIMD 结构\nC. 硬件多线程技术只可用于多核处理器\nD. SMP 中所有处理器共享单一物理地址空间", "input": "", "output": "多核处理器在一个 CPU 芯片中有多个处理器核, 他们可以并行计算。同一时刻在不同的核上可以运行不同的指令, 并分别处理不同的数据。同时有多个指令分别处理多个不同的数的并行系统叫做 MIMD 结构。因此选项 A 正确。一个指令流同时处理多个数据的并行结构叫做 SIMD, 由一个控制单元向多个处理单元提供单一指令流, 每个处理单元拥有局部存储器, 可以对不同的数据实行相同的操作, 适合处理向量。向量处理器是面向向量型数据的并行计算机, 可以在向量的各分量上按多种方式并行执行, 因此选项 B 正确。硬件多线程技术是一种共享单个处理器核内部功能部件的技术,也适用于提高单核处理器中功能部件的利用率。因\n此选项 C 错误。SMP 具有两个以上功能相似的处理器, 他们共享同一主存和外设, 故而选项 D 正确。", "task_type": {"major": ["试题"], "minor": ["考研", "选择题"]}, "domain": ["计算机"], "metadata": "year:2022, score:2", "answer_from": "human", "human_verified": false, "copyright": "暂无版权及作者信息"} +{"instruction": "下列关于多道程序系统的叙述中, 不正确的是 ( ) 。\nA 支持进程的并发执行\nB. 不必支持虚拟存储管理\nC. 需要实现对共享资源的管理\nD. 进程数越多 CPU 利用率��高", "input": "", "output": "并发是多道程序系统的一个基本特征, 进程的并发需要实现对资源的共享管理, 故 A、C 均为正确的。它同样也支持虚拟存储管理, 故 C 也正确。对多道程序系统而言, 进程数 (多道程序度) 并不是越多 CPU 利用率越高, 在初期提高进程数可以提高 CPU 的利用率, 但是当 CPU 利用率达到最大值后, 再提高并发进程数, 反而会导致 CPU 利用率的降低, 故 D 选项不正确。\n\n因此答案为:D", "task_type": {"major": ["试题"], "minor": ["考研", "选择题"]}, "domain": ["计算机"], "metadata": "year:2022, score:2", "answer_from": "human", "human_verified": false, "copyright": "暂无版权及作者信息"} +{"instruction": "下列选项中, 需要在操作系统进行初始化过程中创建的是 ( ) 。\nA. 中断向量表\nB. 文件系统的根目录\nC.硬盘分区表\nD. 文件系统的索引节点表", "input": "", "output": "文件系统的根目录是在操作系统初始化进行磁盘逻辑格式化时创建的, 故选项 B 正确。", "task_type": {"major": ["试题"], "minor": ["考研", "选择题"]}, "domain": ["计算机"], "metadata": "year:2022, score:2", "answer_from": "human", "human_verified": false, "copyright": "暂无版权及作者信息"} +{"instruction": "进程 P0、P1、P2 和 P3 进人就绪队列的时刻、优先级(值越小优先权越高)及 CPU 执行时间如下表所示.\n\\begin{center}\n\\begin{tabular}{|c|c|c|c|}\n\\hline\n进程 & 进入就绪队列的时刻 & 优先级 & CPU 执行时间 \\\\\n\\hline\n$\\mathrm{P} \\bullet$ & $0 \\mathrm{~ms}$ & 15 & $100 \\mathrm{~ms}$ \\\\\n\\hline\n$\\mathrm{P} 1$ & $1 \\mathrm{~ms}$ & $2 \\bullet$ & $60 \\mathrm{~ms}$ \\\\\n\\hline\n$\\mathrm{P} 2$ & $1 \\mathrm{~ms}$ & $1 \\bullet$ & $20 \\mathrm{~ms}$ \\\\\n\\hline\n$\\mathrm{P} 3$ & $15 \\mathrm{~ms}$ & $\\bullet$ & $10 \\mathrm{~ms}$ \\\\\n\\hline\n\\end{tabular}\n\\end{center}\n若系统采用基于优先权的抢占式进程调度算法, 则从 $0 \\mathrm{~ms}$ 时刻开始调度, 到 4 个进程都运行结束为止, 发生进程调度的总次数为 ( ) .\nA. 4\nB. 5\nC. 6\nD. 7", "input": "", "output": "$0 \\mathrm{~ms}$ 时, 只有 $\\mathrm{P} 0$ 进程就绪, 调度 $\\mathrm{P} 0$ 进程; $10 \\mathrm{~ms}$ 时 P1、 P2 就绪, P2 优先级高于 $\\mathrm{P} 0$, 进行 $\\mathrm{CPU}$ 抢占调度; $15 \\mathrm{~ms}$ 时, $\\mathrm{P} 3$ 进程就绪, $\\mathrm{P} 3$ 优先级高于 $\\mathrm{P} 2$, 进行 $\\mathrm{CPU}$ 抢占调度; $25 \\mathrm{~ms}$ 时, $\\mathrm{P} 3$ 进程执行完成, 再次发生进程调度, $\\mathrm{P} 2$ 获得 $\\mathrm{CPU} ; 40 \\mathrm{~ms}$ 时, $\\mathrm{P} 2$ 执行完成, 发生进程, 调度 P0 获得 CPU 继续执行: $130 \\mathrm{~ms}$, P0 进程执行完成, 发生进程调度, P1 获得 CPU,直至进程结束。在这个过程中共发生 6 次进程调度。\n\n因此答案为:C", "task_type": {"major": ["试题"], "minor": ["考研", "选择题"]}, "domain": ["计算机"], "metadata": "year:2022, score:2", "answer_from": "human", "human_verified": false, "copyright": "暂无版权及作者信息"} +{"instruction": "系统中有三个进程 P0、P1、P2 及三类资源 A、B、C. 若某时刻系统分配资源的情况如下表所示, 则此时系统中存在的安全序列的个数为\n\\begin{center}\n\\begin{tabular}{|r|r|r|r|r|r|r|r|r|r|}\n\\hline\n\\multirow{2}{*}{进程} & \\multicolumn{3}{|c|}{已分配资源数} & \\multicolumn{3}{|c|}{尚需资源数} & \\multicolumn{3}{|c|}{可用资源数} \\\\\n\\cline { 2 - 7 }\n & $\\mathrm{A}$ & $\\mathrm{B}$ & $\\mathrm{C}$ & $\\mathrm{A}$ & $\\mathrm{B}$ & $\\mathrm{C}$ & $\\mathrm{A}$ & $\\mathrm{B}$ & $\\mathrm{C}$ \\\\\n\\hline\n$\\mathrm{P} \\bullet$ & 2 & $\\bullet$ & 1 & $\\bullet$ & 2 & 1 & & & \\\\\n\\hline\n$\\mathrm{P} 1$ & $\\bullet$ & 2 & $\\bullet$ & 1 & 2 & 3 & 1 & 3 & \\\\\n\\hline\n$\\mathrm{P} 2$ & 1 & $\\bullet$ & 1 & $\\bullet$ & 1 & 3 & & & \\\\\n\\hline\n\\end{tabular}\n\\end{center}\nA. 1\nB. 2\nC. 3\nD. 4", "input": "", "output": "可用资源数初始状态 $(1,3,2)$, 只能满足 P0, 其存在两个安全序列 (P0, P1, P2)和(P0, P2, P1), 故选项 B 正确。", "task_type": {"major": ["试题"], "minor": ["考研", "选择题"]}, "domain": ["计算机"], "metadata": "year:2022, score:2", "answer_from": "human", "human_verified": false, "copyright": "暂无版权及作者信息"} +{"instruction": "下列关于 CPU 模式的叙述中,正确的是()。\nA.CPU 处于用户态时只能执行特权指令\nB.CPU处于内核态时只能执行特权指令\nC.CPU 处于用户态时只能执行非特权指令\nD.CPU 处于内核态时只能执行非特权指令", "input": "", "output": "CPU 处于内核态下可以执行一切指令, 包括特权指令和非特权指令; 用户态下只能执行非特权指令, 故选项 C 正确。", "task_type": {"major": ["试题"], "minor": ["考研", "选择题"]}, "domain": ["计算机"], "metadata": "year:2022, score:2", "answer_from": "human", "human_verified": false, "copyright": "暂无版权及作者信息"} +{"instruction": "下列事件或操作中, 可能导致进程 $\\mathrm{P}$ 由执行态变为阻塞态的是().\nI. 进程 P 读文件保进程 $\\mathrm{P}$ 的时间片用完\nIII. 进程 P 申请外设 IV. 进程 P 执行信号量的 wait()操作\nA. 仅 I、IV\nB. 仅 II 、III\nC.仅III、IV\nD. 仅 I、III、IV", "input": "", "output": "I、III、IV均有可能发生等待, 使得进程由执行态转换为阻塞态; II 中进程 P 时间片用完后将会转为就绪态, 故选项 D 正确。", "task_type": {"major": ["试题"], "minor": ["考研", "选择题"]}, "domain": ["计算机"], "metadata": "year:2022, score:2", "answer_from": "human", "human_verified": false, "copyright": "暂无版权及作者信息"} +{"instruction": "某进程访问的页 $\\mathrm{b}$ 不在内存中, 导致产生缺页异常, 该缺页异常处理过程中不一定包含的操作是 ( ) 。\nA. 淘汰内存中的页\nB. 建立页号与页框号的对应关系\n$\\mathrm{C}$. 将页 $\\mathrm{b}$ 从外存读入内存\nD. 修改页表中页 $\\mathrm{b}$ 对应的存在位", "input": "", "output": "缺页处理时, 如果内存空间够用, 就不需要淘汰内存中的页, 但是对新加载进内存中的页 $\\mathrm{b}$, 都需要从外存读入内存, 在页表中建立它与页框号的对应关系, 修改页表中页 $\\mathrm{b}$ 的存在位。\n\n因此答案为:A", "task_type": {"major": ["试题"], "minor": ["考研", "选择题"]}, "domain": ["计算机"], "metadata": "year:2022, score:2", "answer_from": "human", "human_verified": false, "copyright": "暂无版权及作者信息"} +{"instruction": "下列选项中, 不会影响系统缺页率的是()。\nA. 页置换算法\nB.工作集的大小\nC. 进程的数量\nD. 页缓冲队列的长度", "input": "", "output": "页面置换算法选择不当, 导致缺页次数变多, 影响缺页率; 工作集的大小, 进程的数量都会影响进程执行过程中的缺页次数; 页缓存则是为了提高文件读写效率设置的, 故选择 D 选项。", "task_type": {"major": ["试题"], "minor": ["考研", "选择题"]}, "domain": ["计算机"], "metadata": "year:2022, score:2", "answer_from": "human", "human_verified": false, "copyright": "暂无版权及作者信息"} +{"instruction": "执行系统调用的过程涉及下列操作, 其中由操作系统完成的是\nI. 保存断点和程序状态字\nII. 保存通用寄存器的内容\nIII. 执行系统调用服务例程\n$\\mathrm{V}$. 将 CPU 模式改为内核态\nA. 仅 I 、III\nB. 仅 II 、III\nC. 仅 II 、 N\nD. 仅 II 、 III 、 N", "input": "", "output": "I 和 IV 可由相应的硬件支撑完成, II 和III则需要由操作系统在进行系统调用时完成,故选项 B 正确。", "task_type": {"major": ["试题"], "minor": ["考研", "选择题"]}, "domain": ["计算机"], "metadata": "year:2022, score:2", "answer_from": "human", "human_verified": false, "copyright": "暂无版权及作者信息"} +{"instruction": "下列关于驱动程序的叙述中, 不正确的是 ( ) 。\nA. 驱动程序与 $I / O$ 控制方式无关\nB. 初始化设备是由驱动程序控制完成的\nC. 进程在执行驱动程序时可能进入阻塞态\nD. 读/写设备的操作是由驱动程序控制完成的", "input": "", "output": "驱动程序是 $I / O$ 进程与设备控制器之间的通信程序, 它与 $I / O$ 设备所采用的 I/O 控制方式紧密相关。初始化设备、读/写设备均由驱动程序控制完成, 进程在执行驱动程序时, 有可能设备处于 “忙” 状态, 从而使进程阻塞。\n\n因此答案为:A", "task_type": {"major": ["试题"], "minor": ["考研", "选择题"]}, "domain": ["计算机"], "metadata": "year:2022, score:2", "answer_from": "human", "human_verified": false, "copyright": "暂无版权及作者信息"} +{"instruction": "在 ISO/OS 参考模型中, 实现两个相邻结点间流量控制功能的是()。\nA. 物理层\nB. 数据链路层\nC. 网络层\nD. 传输层", "input": "", "output": "本题考查 OSI 模型中数据链路层的功能。实现两相邻节点间的流量控制是数据链路层的功能之一, 传输层提供应用进程间的逻辑通信, 即端到端的通信。网络层提供点到点的逻辑通信。\n\n因此答案为:B", "task_type": {"major": ["试题"], "minor": ["考研", "选择题"]}, "domain": ["计算机"], "metadata": "year:2022, score:2", "answer_from": "human", "human_verified": false, "copyright": "暂无版权及作者信息"} +{"instruction": "在一条带宽为 $200 \\mathrm{kHz}$ 的无噪音信道上, 若采用 4 个幅值的 ASK 调制, 则该信道的最大数据传输速率是 ( ) 。\nA. $200 \\mathrm{kh} / \\mathrm{s}$\nB. $400 \\mathrm{kh} / \\mathrm{s}$\nC. $800 \\mathrm{~kb} / \\mathrm{s}$\nD. $1600 \\mathrm{~kb} / \\mathrm{s}$", "input": "", "output": "本题考查奈氏准则的应用。最大数据传输率 $=2 \\mathrm{~W} \\log _{2} \\mathrm{~N}$, 其中带宽 $\\mathrm{W}$ 为 $200 \\mathrm{kHz}$, 采用 4 个幅值的 ASK 调制, 则码元状态数为 4 , 故最大数据传输率为 $2 \\times 200 \\times \\log _{2} 4=800 \\mathrm{~kb} / \\mathrm{s}$ 。\n\n因此答案为:C", "task_type": {"major": ["试题"], "minor": ["考研", "选择题"]}, "domain": ["计算机"], "metadata": "year:2022, score:2", "answer_from": "human", "human_verified": false, "copyright": "暂无版权及作者信息"} +{"instruction": "若某主机的 IP 地址是 183.8 .72.48, 子网掩码是 255.255.192. , 则该主机所在网络的网络地址是 ( ) 。\nA. $183.8 \\bullet .0$\nB. 183.84 .64 .\nC. $183.8 \\bullet .72$.\nD. 183.81 .192\\$.", "input": "", "output": "本题考查对 IP 地址、子网掩码概念的理解。用主机 IP 地址与子网掩码进行逻辑与 (And) 运算可以得出主机所在网络的网络地址, 在进行逻辑与运算时, 只需将 IP 地址与子网掩码中的第三个数转换成二进制进行运算, 即 $(0100$ 1000) And (1100 0000) $=01000000$ (64),答案为 $\\mathrm{B}$ 。\n\n因此答案为:B", "task_type": {"major": ["试题"], "minor": ["考研", "选择题"]}, "domain": ["计算机"], "metadata": "year:2022, score:2", "answer_from": "human", "human_verified": false, "copyright": "暂无版权及作者信息"} +{"instruction": "在 SDN 网络体系结构中, SDN 控制器向数据平面的 SDN 交换机下发流表时所使用的接口是 ( ) 。\nA.东向接口\nB. 南向接口\nC. 西向接口\nD. 北向接口", "input": "", "output": "SDN(软件定义网络)是一种新型的网络架构, 它实现了转发平面和控制平面的分离。在这种网络体系架构中, 从上到下依次被分为: 应用平面、控制平面和转发平面。其中控制器\n(Controler) 位于控制平面, SDN 交换机位于转发平面, 各种应用程序处于应用平面。控制平面和转发平面之间的网络设备状态、数据流表项和控制指令的传达都需要经由通信协议传达,实现控制器对网络设备的管控。目前业界比较看好的是 ONF 主张的 OpenFlow 协议(南向接口) 。在应用平面, 通过控制器提供的编程接口(北向接口)对底层设备进行编程, 把网络的控制器开放给用户, 开发各种业务应用, 实现多样化的业务创新。\n\n因此答案为:B", "task_type": {"major": ["试题"], "minor": ["考研", "选择题"]}, "domain": ["计算机"], "metadata": "year:2022, score:2", "answer_from": "human", "human_verified": false, "copyright": "暂无版权及作者信息"} +{"instruction": "假设主机甲和主机乙已建立一个 $\\mathrm{TCP}$ 连接, 最大段长 $\\mathrm{MSS}=1 \\mathrm{~KB}$, 甲一直有数据向乙发送, 当甲的拥塞窗口为 $16 \\mathrm{~KB}$ 时, 计时器发生了超时, 则甲的拥塞窗口再次增长到 $16 \\mathrm{~KB}$ 所需要的时间至少是 ( ) 。\nA.4 RTT\nB. 5 RTT\nC. 11 RTT\nD. 16 RTT", "input": "", "output": "本题考查 TCP 的拥塞控制机制。在拥塞窗口为 $16 \\mathrm{~KB}$ 时计时器超时, 此时判断网络出现拥塞, 要把慢开始门限值 ssthresh 设置为出现拥塞时的发送方窗口值的一半, 即 $8 \\mathrm{~KB}$ 。然后把拥塞窗口 cwnd 的值重新设置为 1 , 再执行慢开始算法, 每经过一个传输轮次 (RTT)后, 拥塞窗口的值分别为: $2,4,8,9,10,11,12,13,14,15,16$, 因此共需要 11 个RTT。\n\n因此答案为:C", "task_type": {"major": ["试题"], "minor": ["考研", "选择题"]}, "domain": ["计算机"], "metadata": "year:2022, score:2", "answer_from": "human", "human_verified": false, "copyright": "暂无版权及作者信息"} +{"instruction": "假设客户 $\\mathrm{C}$ 和服务器 $\\mathrm{S}$ 已建立一个 $\\mathrm{TCP}$ 连接、通信往返时间 $\\mathrm{RTT}=50 \\mathrm{~ms}$, 最长报文段寿命 $\\mathrm{MSL}=800 \\mathrm{~ms}$, 数据传输结束后, $\\mathrm{C}$ 主动请求断开连接。若从 $\\mathrm{C}$ 主动向 $\\mathrm{S}$ 发出 $\\mathrm{FIN}$ 段时刻算起, 则 $\\mathrm{C}$ 和 $\\mathrm{S}$ 进入 CLOSED 状态所需的时间至少分别是()。\nA. $850 \\mathrm{~ms}, 50 \\mathrm{~ms}$\nB. $1650 \\mathrm{~ms}, 50 \\mathrm{~ms}$\nC. $850 \\mathrm{~ms}, 75 \\mathrm{~ms}$\nD. $1650 \\mathrm{~ms}, 75 \\mathrm{~ms}$", "input": "", "output": "本题考查对 $\\mathrm{TCP}$ 连接释放 (四次挥手) 过程的理解。对客户 $\\mathrm{C}$ 来说, 从向 $\\mathrm{S}$ 发出 FIN 段时刻, 共经历了发送连接释放报文段(阶段 1)、接收服务器对连接释放报文段的确认 (阶段 2) 、服务器端的连接释放报文段(阶段 3) 、对服务器端连接释放报文段的确认(阶段 3) 和时间等待计时器设置的时间 2MSL (最长报文段寿命) 之后才进入 CLOSED 状态, 由于要求出最少时间, 因此阶段 2 和阶段 3 同时进行, 即 $\\mathrm{B}$ 也没有数据向 $\\mathrm{A}$ 发送, 无需等待,因此客户 $\\mathrm{C}$ 总共经历的时间为 $\\mathrm{RTT}+2 \\mathrm{MSL}=1650 \\mathrm{~ms}$ 。在此过程中, 服务器端 $\\mathrm{S}$ 从客户 $\\mathrm{C}$ 发出 FIN 时刻起到进入 CLOSED 状态共经历了 1.5 个 RTT, 即 $75 \\mathrm{~ms}$ 。该题请结合连接释放过程的图来理解。\n\n因此答案为:D", "task_type": {"major": ["试题"], "minor": ["考研", "选择题"]}, "domain": ["计算机"], "metadata": "year:2022, score:2", "answer_from": "human", "human_verified": false, "copyright": "暂无版权及作者信息"} +{"instruction": "根设主机 $\\mathrm{H}$ 通过 HTTP/1.1 请求浏览某 Web 服务器 $\\mathrm{S}$ 上的 Web 页 news408.hml, news408.hml 引用了同目录下 1 个图像, news408.hml 文件大小为 $1 \\mathrm{MSS}$ (最大段长), 图像文件大小为 $3 \\mathrm{MSS}, \\mathrm{H}$ 访问 $\\mathrm{S}$ 的往返时间 RTT $=10 \\mathrm{~ms}$, 忽略 HTTP 响应报文的首部开销和 TCP 段传输时延。若 $\\mathrm{H}$ 已完成域名解析, 则从 $\\mathrm{H}$ 请求与 $\\mathrm{S}$ 建立 TCP 连接时刻起, 到接收到全部内容止, 所需的时间至少是 ( ) 。\nA. $30 \\mathrm{~ms}$\nB. $40 \\mathrm{~ms}$\nC. $50 \\mathrm{~ms}$\nD. $60 \\mathrm{~ms}$", "input": "", "output": "本题考查对 HTTP/1.1 工作过程及持续非流水线工作方式的理解。HTTP/1.1 使用了面向连接的 TCP 作为传输层协议。在 HTTP 中, 当请求消息比较长, 超过了 MSS 的长度, TCP 就需要把 HTTP 的数据拆解成一块块的数据发送, 而不是一次性发送所有数据, 拆分出来的每一块数据都要加上 $\\mathrm{TCP}$ 头信息放进单独的网络包中, 然后交给 IP 模块来发送数据, 因此大小为 $3 \\mathrm{MSS}$ 的图像文件在实际传输时被拆分成 3 个对象, 需要 3 次请求, 每访问一次对象就用去一个 RTT, 共需要 3 个 RTT, 再加上第一次请求页面时的 1 个 RTT, 因此需计 4 个 RTT 才能收到全部内容。\n\n因此答案为:B", "task_type": {"major": ["试题"], "minor": ["考研", "选择题"]}, "domain": ["计算机"], "metadata": "year:2022, score:2", "answer_from": "human", "human_verified": false, "copyright": "暂无版权及作者信息"} +{"instruction": "People often grumble that plastics are too durable. Water bottles, shopping bags, and other trash litter the planet, from Mount Everest to the Mariana Trench, because plastics are ubiquitous and don't break down easily. But some plastic materials change over time. They crack and frizzle They “weep” out additives. They melt into sludge. All of which creates huge headaches for institutions, such as museums, trying to preserve culturally important objects. Until recently, museums only had to worry about traditional materials. The variety of plastic objects at risk is dizzying: early radios, avant-garde sculptures, celluloid animation stills from Disney films, David Bowie costumes, the first artificial heart.\nCertain artifacts are especially vulnerable because some pioneers in plastic art didn’t always know how to mix ingredients properly, says Thea van Oosten, a polymer chemist who, until retiring a few years ago, worked for decades at the Cultural Heritage Agency of the Netherlands (RCE). “It’s like baking a cake: If you don’t have exact amounts, it goes wrong,” she says. “The object you make is already a time bomb.”\nAnd sometimes, it’s not the artist’s fault. In the 1960s, the Italian artist Piero Gilardi began to create hundreds of bright, colorful foam pieces. Those pieces included small beds of roses and other items as well as a few dozen “nature carpet” — large rectangles decorated with foam pumpkins, cabbages, and watermelons. He wanted viewers to walk around on the carpets—which meant they had to be durable\nUnfortunately, the polyurethane foam he used is inherently unstable. It’s especially vulnerable to light damage, and by the mid-1990s, Gilardi’s pumpkins, roses, and other figures were splitting and crumbling. Museums locked some of them away in the dark.\n So van Oosten and colleagues at RCE began to study ways to protect polyurethane. First, they took foam samples similar to the nature carpets and infused some with stabilizing and consolidating chemicals that modern manufacturers often use. Van Oosten calls those chemicals “sunscreens” because their goal was to prevent further light damage and rebuild worn polymer fibers. Then the team used xenon lamps to artificially age both treated and untreated samples, and examined them under high-powered microscopes. The results were encouraging. Samples that lacked sunscreen had withered under the barrage of photons: The molecular “struts” shoring up the foam were 42% thinner and notably more brittle than before the lamp treatment. The struts in samples with sunscreen decreased by as little as 12.5%. Armed with that knowledge, conservators working with RCE infused several Gilardi sculptures, including two nature carpets, with the sunscreen to stabilize them. Van Oosten is proud that several have even gone on display again, albeit sometimes beneath protective cases. Long called the “queen of plastics,” in 2012, van Oosten was knighted in the Netherlands for her efforts to preserve plastic objects and spread knowledge to other institutes.\n Despite such success stories, preservation of plastics will likely get harder. Old objects continue to deteriorate. Worse, biodegradable plastics, designed to disintegrate, are increasingly common.\nAnd more is at stake here than individual objects. Ferreira notes that archaeologists first defined the great material ages of human history — Stone Age, Iron Age, and so on — after examining artifacts in museums. We now live in an age of plastic, she says, “and what we decide to collect today, what we decide to preserve...will have a strong impact on how in the future we'll be seen.”\n\n21. According to paragraph 1, museums are faced with difficulties in_______.\n[A] maintaining their plastic items\n[B] Obtaining durable plastic artifacts\n[C] Handling outdated plastic exhibits\n[D] Classifying their plastic collections\n22. Van Oosten holds certain plastic are_______.\n[A] immune to decay\n[B] improperly shaped\n[C] inherently flawed\n[D] complex in structure\n23. Museums stopped exhibiting artworks of G’s to ______.\n[A] keep them from hurting visitors\n[B] duplicate them for future display\n[C] have ingredients for future analyzed\n[D] prevent them from further damage\n24. The author thinks that preservation of plastics is______.\n[A] costly\n[B] unworthy\n[C] unpopular\n[D] challenging\n25. In Ferreisia’s opinion, the preservation of plastic artifacts______.\n[A] will inspire future scientific research\n[B] has profound historical significance\n[C] will help us separate the material ages\n[D] has the impact on today’s cultural life", "input": "", "output": "21、【解题思路】 题目问的是博物馆所面临的困难,可定位到第一段“All of which creates hugeheadaches for institutions, such as museums, trying to preserve culturally important objects.”即:让博 物馆之类的机构头疼的是如何保存这些有文化意义的物品,对应 A 选项保养他们的塑料品,原 文preserve 对应选项中的 maintain。【干扰排除】选项【B】 Obtaining durable plastic artifacts 获得耐用的塑料手工艺品在文中未提及;选 项【C】 Handling outdated plastic exhibits 处理过时的塑料展品有一定干扰性, 但“过时的”这一层意 思在文章中为体现;选项【D】 Classifying their plastic collections 分类塑料收藏品在文章中未提及。\n22、【解析思路】由题干关键词人名 Van Oosten 定位到第二段第一句话, 首先上来就点明 certainartifacts are vulnerable是有弱点的,同义词替换约等于选项【C】 flawed 即“有缺陷的”,“本质上是 有缺陷的” 。【干扰排除】 【B】干扰性极强,定位到原文中 artifacts有弱点的原因部分, 即艺术家们不知道如何 恰当地混合各种原材料,才致使塑料成品存在弱点,而非是【B】选项中不恰当的形状 shape 。【A】和【D】属于弱干扰, 第二段没有提及【A】中的“免于腐烂”, 虽然在第一段提到 don’t break down easily不易降解,但【A】选项仍未偷换概念 。【D】选项“结构复杂”在原文中未提及,结构不等于原 材料,更不等于原材料的配比情况。\n23、【解题思路】本题答案为【D】“防止这些塑料艺术品的进一步损坏” 。根据本道题所定位的 段落。题干出现在段落末尾。此段一开始谈到了一些塑料艺术品用的一些 poly 之类的化学物质, 会易受光源的影响,这就会导致艺术品的进一步损坏。【干扰排除】 【A】选项谈到的游客在全段话没有提及,属于无关定位的错误选项;【B】选项复制艺术品为了未来展示也未提及;而【C】选项的 ingredients 属于定位错误理解,作者提到成分是为了指 出一些成分易受光源影响而非是为了进一步分析。\n24、【解题思路】本题正确答案【D】。Challenging (有挑战的)。题干问“作者认为塑料的保存__”, 问的是作者观点,根据顺序原则,第 5 段讲的是 Oosten 的具体做法,第 7 段讲的是 25 题题干 中提到的Ferrira,可以确定该题答案位于第 6 段 。第 6 段第一句说 “Despite successful stories as Van Oosten,preserve plastics will likely get harder”(尽管有 Oosten这样成功的案例,保存塑料似 乎变得愈发困难)【干扰排除】 四个选项【A】 Costly(昂贵的),【B】Unworthy(不值得的),【C】Unpopular(不受欢迎的), 只有【D】 Challenging (有挑战性的)对应原文 get harder(更加困难的),其余 概念并未提及。\n25、【解题思路】 本题答案为【B】“有着深远的历史意义” 。根据本道题题干信息,细节题,定位 在文章最后 一 段 。 【F】这个 人的观 点中, 提到的 文章主 题词围 绕 the preservation of plastic artifacts“塑料人造艺术品的保留”会决定未来人们如何定义我们,也就是同义替换为在未来如何 看到过去的历史, 即为【B】选项中历史意义。【干扰排除】 【A】选项有诱惑原词 future,将鼓励未来科学研究“scientific research”为未提到信息的 错误选项; 【C】选项“帮助我们分隔材料的年限”;D 选项“影响我们当今的文化生活”均为最后一 段没有提到的信息,属于无中生有的错误选项。", "task_type": {"major": ["试题"], "minor": ["考研", "选择题"]}, "domain": ["英语"], "metadata": "year:2022, score:2", "answer_from": "human", "human_verified": false, "copyright": "暂无版权及作者信息"} +{"instruction": "As the latest crop of students pen their UCAS form and weigh up their options, it may be worth considering just how the point, purpose and value of a degree has changed and what Gen Z (and their parents) need to consider as they start the third stage of their educational journey. Millennials were told that if you did well in school, got a decent degree, you would be set up for life. But that promise has been found wanting. As degrees became universal, they became devalued (just as governments hiked up the cost of getting one). Education was no longer a secure route of social mobility. Today, 28 per cent of graduates in the UK are in non-graduate roles; a percentage which is double the average amongst the OECD.\n This is not to say that there is no point in getting a degree, but, rather stress that a degree is not for everyone, that the switch from classroom to lecture hall is not an inevitable one and that other options are available.\n Thankfully, there are signs that this is already happening, with Gen Z seeking to learn from their millennial predecessors, even if parents and teachers tend to be still set in the degree mindset. Employers have long seen the advantages of hiring school leavers who often prove themselves to be more committed and loyal employees than graduates. Many too are seeing the advantages of scrapping a degree requirement for certain roles—just as Penguin did in 2016.\n For those for whom a degree is the desired route, consider that this may well be the first of many. In this age of generalists, it pays to have specific knowledge or skills. Postgraduates now earn 40 per cent more than graduates. When more and more of us have a degree, it makes sense (albeit expensive) to have two.\n It is unlikely that Gen Z will be done with education at 18 or 21; they will need to be constantly up-skilling throughout their career to stay agile, relevant and employable. It has been estimated that this generation due to the pressures of technology, the wish for personal fulfilment and desire for diversity will work for 17 different employers over the course of their working life and have five different careers. Education, and not just knowledge gained on campus, will be a core part of Generation Z’s career trajectory.\n I have often heard older generations talk about their degree (even if it was gained decades ago) in the present and personal tense: “I am a geographer or ‘I am a classist’. Their sons or daughters would never say such a thing; it’s as if they already know that their degree won’t define them in the same way.\n26. The author suggests that generation Z should________.\n[A] Be careful in choosing college\n[B] Be diligent at each educational stage\n[C] Reassess the necessity of college education\n[D] Postpone their undergraduate application\n27. The percentage of UK graduates in non-graduate roles reflects_____.\n[A] millennials opinions about work\n[B] the shrinking value of degree\n[C] public discount with education\n[D] the desired route of social mobility\n28. The author considers it a good sign that______.\n[A] Generation Z are seeking to earn a decent degree\n[B] school leavers are willing to be skilled workers\n[C] employers are taking a realistic attitude to degrees\n[D] parents are changing their minds about education\n29. It is advised in paragraph 5 that those with one degree should_______.\n[A] make an early decision on their career\n[B] attend on-the-job training programs\n[C] team up with high-paid post graduates\n[D] further their studies in the specific field\n30. What can be concluded about Generation Z from last two paragraphs?\n[A] Lifelong learning will define them.\n[B] They will make qualified educators.\n[C] Degrees will no longer appeal to them.\n[D] They will have a limited choice of jobs.", "input": "", "output": "26、【解题思路】本题为第 1 题,并且结合题干关键词 generation Z 定位至文章第一段 。由第一 段可知 Z世代的人应该意识到大学的价值等是发生了变化的,可以推知作者认为 Z 世代的人需 要去考虑是否有必要去读大学 。这个也是符合这篇文章所讨论的中心主题的 。因此答案为选项 【C】reassess the necessity ofcollege education【干扰排除】选项【A】的意思是在挑选大学的时候要小心仔细,第一段开头说到学生在填大学申请 时,需要意识到现在大学的价值目的等变化, 并不是说挑选学校的时候要小心,故排除 。没有 提及延期申请,故排除选项【D】。在教育的不同阶段都要勤奋属于无中生有,故排除选项【B】\n27、【解题思路】根据题干关键词 The percentage of UK graduates 和 non-graduate roles 可定位 到第二段As degrees become universal, they become devalued.所以选【B】the shrinking value of degree【干扰排除】 【A】和【C】在文章中并没有提到; 【D】原文虽然有一句 education was no longer asecure route of social mobility,但与【D】所提的没有关联,容易混淆。\n28、【解题思路】本题根据题文同序原则定位于第四段 。因为 26 题位于 1 段,27 题位于 2 和 3 段。段首句的 thankfully 暗示作者正面评价,对应题干的 a good sign,但没有具体说明 good sign 是什么 。第二句说 Z 世代已经从他们的前辈千禧一代身上学到了一些教训 。第三句说雇主们了 解到一些离开学校的人(即没有拿到学历)更加的忠诚,第四句继续补充 。因此第三句对应于 C 选项内容:雇主们对学历采取一种现实的态度。【干扰排除】 【A】选项:Z 世代正在寻求获得一个体面的学历,本段并没有提及,而且和上文逻辑 (学历贬值)相反,故排除。【B】选项:离开学校的人愿意成为熟练工人。本段第 3 句确实说雇主愿意雇佣一些离开学校的人。 但并没有说这些人怎么想,熟练工人也未被提及,故排除。【D】选项:家长正在改变他们关于教育的思维定势 。家长和思维定势(mindset)是第一句中原词, 但是只是提到了家长仍然有思维定式,并没有说他们在改变,故排除。\n29、【解题思路】 由题干关键词 Paragraph5 “those with one degree should ...”可以定位至第 5 段末尾句 “When more and more of us have a degree, it makes sense to have two.”意味: 当越来越多的人 开始拥有一个学历,对于我们而言明智的选择是“拥有两个学位”,所以正确选项是 D,”further theirstudies in a specified field” .【干扰排除】 【C】选项 team up with high paid post graduates,而第 3 句中,“post graduatesearn more than 40% than graduates ”,只是提到硕士生收入高于本科毕业生 40%。没有提到,要和硕士生一 起工作。所以错误。【B】选项属于无中生有选项,第 5 段只在第 2 句中提到,“it pays to havespecified knowledge or skills”,毕业生要有专业知识或者技能,并未提到他们应该参加职业培训。【A】选项, “make an early decision on their career”,要毕业生尽早在职业上做决定,也是并未提到 。而第 5段首句“a degree is the desired route, consider that this may well be the first of many”, 意味毕业生依 然会首选获得一个学位,和【A】选项表意不符。\n30、【解题思路】 题干中的 last two paragraphs 直接告诉我们本道题的定位在文章最后两段,除 此之外还有一个关键词 Generation Z。但是由于本篇文章讲的全部都是 Generation Z 的内容,所 以这个关键词没有实质的定位作用,那么可以通过观察选项的方式进行反向排除 。A 选项中的 lifelong learning 意为“终身学习”,于文章中倒数第二段第一句分号之后的内容 they will need to be constantly up-skillinghroughout their career to stay employable 是同义替换,都表示要一直学习 来提高自己的技能, 因此答案为选项【A】。【干扰排除】选项【B】中的关键词 educators 在最后两段中没有原词和同义替换,而且“教育工作者” 这个内容和本文的主旨内容也不相关,故排除选项【B】。【C】选项中有一个文章最后一句出现的 原词degree,但是文章此处的 degree 只是说“学历不会以同样的形式来定义他们”, 并不能得出 选项中所说的“学历将不再吸引他们”, 故排除选项【C】。文章倒数第二段最后一句中有个动词 work 和名词 workinglife,可以理解为【D】选项中 job 的同义替换,但是文章中这句话讲的是他们 在职业生涯中会为 17 个不同的雇主工作,与选项中所说的“有限的工作机会”是相反的,故排除 选项【D】。", "task_type": {"major": ["试题"], "minor": ["考研", "选择题"]}, "domain": ["英语"], "metadata": "year:2022, score:2", "answer_from": "human", "human_verified": false, "copyright": "暂无版权及作者信息"} +{"instruction": "Enlightening, stimulating, inspiring, fun. These were some of the words that Nature readers used to describe their experiences of art-science collaborations in a series of articles on partnerships between artists and researchers. Nearly 40% of the roughly 350 people who responded to an accompanying poll said they had collaborated with artists; and almost all said they would consider doing so in future.\n Such an encouraging result is not surprising. Public engagement has become essential to many research projects. Scientists are increasingly seeking out visual artists and designers to help them to communicate their work to new audiences. “Artists help scientists reach a broader audience and make emotional connections that enhance learning,” one respondent said. “The experience is very liberating for me, as a scientist,” said another. “There’s often a visual aspect to my science that generating and publishing data does not convey.”\n One example of how artists and scientists have together rocked the senses came last month when the Sydney Symphony Orchestra in Australia performed a reworked version of Antonio Vivaldi’s The Four Seasons. They reimagined the 300-year-old score by injecting the latest climate prediction data for each season — provided by Monash University’s Climate Change Communication Research Hub in Melbourne. The work was entitled The (Uncertain) Four Seasons, and variations of the score containing local data were sent to every major orchestra in the world. The performance was a creative call to action ahead of November’s United Nations.\n But a genuine partnership must be a two-way street. Fewer artists than scientists responded to the Nature poll; however, several respondents noted that artists do not simply assist scientists with their communication requirements. Nor should their work be considered only as an object of study —even if these are reasons why scientists seek opportunities to work with artists. The alliances are most valuable when scientists and artists have a shared stake in a project, are able to jointly design it and can critique each other’s work. Such an approach can both prompt new research as well as result in powerful art.\n More than half a century ago, the Massachusetts Institute of Technology (MIT) opened its Center for Advanced Visual Studies (CAVS) in Cambridge to explore the role of technology in culture. The centre was established during the Vietnam War, when many scientists in the United States were being criticized for working on defence contracts. Its founders believed that artists and scientists could, together, create a vision for a more humane world. They deliberately focused their projects around light — hence the ‘visual studies’ in the name. Light was a something that both artists and scientists had an interest in, and therefore could form the basis of collaboration, says Seth Riskin, a visual-arts researcher at the MIT Museum who previously worked at CAVS. The reach of art-science tie-ups needs to go beyond the necessary purpose of research communication, and participants must not fall into the trap of stereotyping each other. Artists and scientists alike are immersed in discovery and invention, and challenge and critique are core to both, too.\n31. According to paragraph 1, art-science collaborations have ______.\n[A] caught the attention of critics\n[B] received favorable responses\n[C] promoted academic publishing\n[D] sparked heated public disputes\n32. The reworked version of The Year Season is mentioned to show that______.\n[A] art can offer audiences easy access to science\n[B] science can help with the expression of emotions\n[C] public participation in science has a promising future\n[D] art is effective in facilitating scientific innovation\n33. Some artists seem to worry that in the art-science partnership _______.\n[A] their roles may be underestimated\n[B] their reputation may be impaired\n[C] their creativity may be inhibited\n[D] their work may be misguided\n34. What does the author say about CAVS ?\n[A] It was headed alternately by artists and scientists.\n[B] It exemplified valuable art-science alliances.\n[C] Its projects aimed at advancing visual studies.\n[D] Its founders sought to raise the status of artists.\n35. In the last paragraph, the author holds art- science collaborations______.\n[A] are likely to go beyond public expectations\n[B] will intensify interdisciplinary competition\n[C] should do more than communicating science\n[D] are becoming more popular than before", "input": "", "output": "31、【解题思路】由题干关键词 art-science collaborations 定位至第一段 。根据第一段句①:启 发性的、挑战性的、激励性的、趣味性的。可知读者对合作的态度是积极的。第二段句①中 such an encouragingresult 指代第一段中的结果是鼓舞性的,句②中的 help 和句③中 enhance learning 等地方也能看出是积极的回应,对应选项【B】 received favourable responses。【干扰排除】选项【A】文中没有提到 critics,文中提到了读者对合作的评价, 而不是 critics 的评价,顾排除 。选项【C】没有提到与促进学术出版相关的信息,故排除 。选项【D】文中只有读者对合作 的评价,上下文的意思都是一致的,积极的,没有 dispute 争议,顾排除。\n32、【解题思路】 例证题。根据题干关键词 the Four Seasons 定位到第三段第句,这个例句是为 了证明论点,所以在例句前后找论点,第二段的第句和第④句是论点句,第句提到 scientists are increasinglyseeking out visual artists ... communicate their work to new audiences科学家正在寻找 视觉艺术家...与新��听众交流;第 4 句提到,artists help scientists reach a broader audiences ...艺 术家帮助科学家接到更广泛的听众,可知论点讨论的是艺术家可以帮助科学家接触到听众, 故 A 项正确。【干扰排除】选项【B】文中没有提到 expression,文中提到了 connection 联系,故排除。选项【C】没 有提到 a promising future 有前景的未来,故排除 。选项【D】文中未提到 scientific innovation 科学创新,故排除。\n33、【解题思路】由题干关键词 artists 和 worry 定位至第四段 。第四段首句 But a genuine partnershipmust be a two-way street 指出“真正的合作关系应该是双向的”, 后面第二句 however 接着讲到一些“artists do not simply assist scientists with their communication requirements”即“一些 艺术家不仅仅帮助科学家去交流”,再往后一句紧跟着讲到“nor should their work be considered only as...”即“艺术家的工作也不应该只被认为...”,说明这些艺术家们认为他们的工作没有得到 全面的认知, 与选项【A】their role may be underestimated“他们的角色/作用被低估了”一致,所以 答案选【A】。【干扰排除】选项【B】“艺术家的名声被破坏”在原文中没有涉及到,故排除;选项【C】“艺术家的创 新能力被压抑”与原文相反,因为上文中有提到艺术家们用生动别致的展现形式让观众了解了科 学,所以这个选项也排除掉;选项【D】“艺术家的工作被误导”与原文信息不一致,原文中对艺术 家与科学家的合作是肯定的,并没有指出他们的工作出现偏差,所以【D】也不是正确答案。\n34、【解题思路】由题干可定位到篇章第六段,作者提到 MIT 开设(opened)CAVS 目的是为 了探索技术在文化中的角色(to explore the role of technology in culture),接下来的内容也说到它 的创始人相信艺术家和科学家可以共同创造一个更加人性化的世界 。(Its founder believed that artists and scientistcould, together, create a vision for a more humane world.)所述均对应 B 选项的 art-science alliances(n.联合)【干扰排除】A 选项中的 alternately (adv.轮流地)。选项表示:它是由艺术家和科学家轮流执掌 的,文章中并未提及这一机构的领导问题,故排除;【C】选项表示:它的项目旨在推进视觉研究, 这是用文中的举例 visual studies 混淆视听,这里的举例只是艺术与科学联合的体现,作为选项 则以偏概全了,可排除;【D】选项:它的创始人试图提高艺术家的地位 。文章中并没有此类内容 的描述,故也可排除。\n35、【解题思路】由题干定位到最后一段,在最后一段中作者提到 art-science collaborations,作 者表示both sides, 即艺术和科学这两方, need to go beyond the necessary purpose of researchcommunication,表明他们需要不仅仅是研究交流方面,后文表明,他们要更深入发现及研究 。 所以,C选项表明需要的不仅仅是交流,应该是拓展更多的方面。【干扰排除】选项【A】超过公众期待,在这一段里并无出现相关细节,属于无中生有错误。选项【B】加强各学科之间的竞争。同样在本段内并无相关细节。属于无中生有。选项【D】比之前更受欢迎, 属于无中生有。", "task_type": {"major": ["试题"], "minor": ["考研", "选择题"]}, "domain": ["英语"], "metadata": "year:2022, score:2", "answer_from": "human", "human_verified": false, "copyright": "暂无版权及作者信息"} +{"instruction": "The personal grievance provisions of the Employment Relations Act 2000 (ERA) prevent an employer from firing an employee without good cause. Instead, dismissals must be justified. Employers must both show cause and act in a procedurally fair way.\n Personal grievance procedures were designed to guard the jobs of ordinary workers from “unjustified dismissals”. The premise was that the common law of contract lacked sufficient safeguards for workers against arbitrary conduct by management. Long gone are the days when a boss could simply give an employee contractual notice.\n But these provisions create difficulties for businesses when applied to highly paid managers and executives. As countless boards and business owners will attest, constraining firms from firing poorly performing, high-earning managers is a handbrake on boosting productivity and overall performance. The difference between C-grade and A-grade managers may very well be the difference between business success or failure. Between preserving the jobs of ordinary workers or losing them. Yet mediocrity is no longer enough to justify a dismissal.\n Consequently—and paradoxically —laws introduced to protect the jobs of ordinary workers may be placing those jobs at risk.\n If not placing jobs at risk, to the extent employment protection laws constrain business owners from dismissing underperforming managers, those laws act as a constraint on firm productivity and therefore on workers’ wages. Indeed, in “An International Perspective on New Zealand’s Productivity Paradox” (2014), the Productivity Commission singled out the low quality of managerial capabilities as a cause of the country’s poor productivity growth record. Nor are highly paid managers themselves immune from the harm caused by the ERA’s unjustified dismissal procedures. Because employment protection laws make it costlier to fire an employee, employers are more cautious about hiring new staff. This makes it harder for the marginal manager to gain employment. And firms pay staff less because firms carry the burden of the employment arrangement going wrong.\n Society also suffers from excessive employment protections. Stringent job dismissal regulations adversely affect productivity growth and hamper both prosperity and overall wellbeing.\n Across the Tasman, Australia deals with the unjustified dismissal paradox by excluding employees earning above a specified “high-income threshold” from the protection of its unfair dismissal laws.In New Zealand, a 2016 private members’ Bill tried to permit firms and high-income employees to contract out of the unjustified dismissal regime. However, the mechanisms proposed were unwieldy and the Bill was voted down following the change in government later that year\n36. The personal grievance provisions of the ERA are intended to ____.\n[A]punish dubious corporate practices\n[B]improve traditional hiring procedures\n[C]exempt employers from certain duties\n[D]protect the rights of ordinary workers\n37. It can be learned from the Paragraph 3 that the provision may _______.\n[A]hinder business development\n[B]undermine the managers' authorities\n[C]affect the public image of the firms\n[D]weaken the labor-management relations\n38. Which of the following measures would the Productivity Commission support?\n[A]Imposing reasonable wage restraints.\n[B]Enforcing employment protections.\n[C]Limiting the powers of business owners.\n[D]Dismissing poorly performing managers.\n39. What might be an effect of ERA’s unjustified dismissal procedure?\n[A]High-paid managers may lose jobs.\n[B]Employees suffer salary cut.\n[C]Society sees a rise in well-being.\n[D]Employers hire new staff.\n40. It can be inferred that “ high-income threshold” in Australia_______.\n[A]has secured managers’ earnings\n[B]has produced undesired results\n[C]is beneficial to business owners\n[D]is difficult to put into practice", "input": "", "output": "36、【解题思路】由题干关键词 personal grievance provisions 和 ERA 结合题文同序原则,首先 定位至第一段第一句找到相关话题。第一段的三句话是对 personal grievance provisions 内容的介 绍, 而题意旨在提问其意图,往后定位至第二段首句可以看出该句紧扣题意: “Personal grievance procedures weredesigned to guard the jobs of ordinary workers from ‘unjustified dismissals’ .”可以看 出 personalgrievance provisions 的设计意图是保护普通人的工作免受不公正解雇,正确答案 D“保 护普通人的权利”是对该句句意的正确体现。【干扰排除】选项【A】“惩罚有问题的公司实践” 、【B】“改善传统的雇佣流程”(hiring 与原文的firing 也不符)、【C】“免除公司部分义务”在文段中均未有体现。\n37、【解题思路】由题干已知定位段落和关键词 provisions 定位到第三段的句①和句② 。句①主干已明确指出,“这些规定给公司制造了困难”(these provisions create difficulties for businesses)。 此外,句②主干进一步指出,“限制公司解雇表现不佳而收入颇高的经理如同一个手刹,无法促进(企业)生产力和整体业绩” (constraining firms from firing poorly performing, high-earningmanagers is a handbrakeon boosting productivity and overall performance.)。最后,如果①②两句还无法摸透,还可以辅助句③进一步确认 。句③有大写字母, 因此是事实例证, 例子可以佐证身为观点的①②两句。句③指出,“C 等级和 A 等级的经理的区别很可能就反映出公司成或败的区别” 。因此经理若业绩表现不佳,公司很可能会与成功失之交臂 。总而言之,选项中的“阻碍企业发展” (hinder business management)为最佳的和原文关键句同义改写的选项, 即【A】选项。【干扰排除】选项【B】原文仅提及经理表现不佳且薪水颇丰的问题, 并未提到“瓦解经理的权威” 这一事实, 因此 undermine 和 authorities 文中未提及,属于无中生有,故排除;选项【C】原文仅 提及企业因为相关规定无法解雇表现不佳的经理,从而影响企业的生产力和业绩,但文中未提 及企业的“公共形象”(public image)是否有受影响, 因此该选项也属于无中生有,故排除 。选 项【D】 weaken the labor-management relations 文中未提���,属于无中生有,故排除。\n38、【解题思路】由题干关键词 Productivity Commission 定位至第五段第②句 。该句指出委员 会认为低质量的管理是国家生产力增加纪录变现不佳的原因 。言外之意就是表现不佳的经理导 致了低生产力 。委员会必然想提高生产力,那么就要把表现不佳的经理开除,故选 D。【干扰排除】选项【A】是根据第一段①句中的worker’s wages设置的干扰项。该句说的是这些法律会限制工人的工资,跟Productivity Commission没有关系;【B】选项无中生有,此段没有提到就业保 护;【C】选项是强干扰项,首先要明确生产力委员会必然想提高生产力 。第①句说到这项法案使 得企业老板没有办法开除表现不佳的经理,而第②句说到正是这些表现不佳的经理导致低生产 力 。那么想要提高生产力就要给老板权力让他们可以开除表现不佳的人, 而不是限制老板的权 力,故排除。\n39、【解题思路】先由题干关键词 ERA’s unjustified dismissal procedure 定位至第六段第一句, 根据句意可初步确定影响为负面的,且根据题干所问 an effect 可推测原文中不止一个影响,需 要向后读继续从原文验证 。第四句主干提到公司给员工支付了更少的工资,对应到选项中会发 现与【B】员工遭受降薪相符,为正确答案。【干扰排除】选项【A】是根据第六段首句的 highly paid managers 设置的干扰项,但文中未提及他们会丢掉工作 。第二句中出现 employers are more cautious about hiring new staff,说明更不容易招 聘新员工了, 而选项【D】内容说雇主会招聘更多员工, 与原文不符, 故排除 。选项【C】是关于 society,可定位至第七段第一句,根据 Society also suffers 可再次确认对社会的影响是负面的, 但【C】的表述是正向, 因此排除。\n40、【解题思路】由题干关键词 high-income threshold(高收入门槛)定位至最后一段。该段第 一句交代澳大利亚解决 unjustified dismissals paradox(不公平解雇悖论)的方法是通过把高收入 雇员排除在对雇主不公平的法规保护范围之外 。但这一句只是陈述了澳大利亚的做法, 并不能 得知具体的执行效果 。该段最后一句,提到 the mechanisms proposed were unwieldy (所提出的 机制是难操作的),则可推知这一做法是难以付诸实现的,故选 D。【干扰排除】选项【A】 has secured managers’ earnings (保障了经理人的收入)与文意相反,澳大利 亚提出 high-income threshold 的做法为的就是让经理级别的高收入人员免于法律的保护,从而使 雇主可以根据工作表现选择解雇他们而不受法规约束。选项【B】has produced undesired results(造 成不良结果)和选项【C】is beneficial to business owners(对雇主有利)均属于过度推测,段落中 没有细节得以支撑。选项【C】干扰性较大, 因为较为符合文章的主旨大意,但如果没有具体细节 说明澳大利亚的做法是成功的,则也不能推知这一做法对雇主有利。", "task_type": {"major": ["试题"], "minor": ["考研", "选择题"]}, "domain": ["英语"], "metadata": "year:2022, score:2", "answer_from": "human", "human_verified": false, "copyright": "暂无版权及作者信息"} +{"instruction": "结合考古发现和传世文献, 与商代相比, 西周各方面都发生了巨变, 但其也有共通之处, 下列属于共通之处的是\nA. 金属工具的广泛使用\nB. 国家治理模式\nC. 血缘氏族影响巨大\nD.王位传承方式", "input": "", "output": "亲族血缘关系在商代政治和西周政治中都发挥着重要作用, 如西周的宗法制, C 项正确。金属工具的广泛使用最早要到春秋战国时期甚至更晚, A 项错误;商代的国家治理模式是内外服制, 西周的国家治理模式是分封制, B 项错误; 商代的王位继承方式是兄终弟及与父死子继相结合, 西周的王位继承方式是宗法制之下的嫡长子继承制, D 项错误。", "task_type": {"major": ["试题"], "minor": ["考研", "选择题"]}, "domain": ["历史"], "metadata": "year:2023, score:4", "answer_from": "human", "human_verified": false, "copyright": "暂无版权及作者信息"} +{"instruction": "有关东汉史学论著中, \"世家大族\"'豪强\"常用来界定东汉两个特定的阶层, 两阶层显著差异表现在\nA. 宗族势力强弱\nB. 土地占有数量\nC. 文化价值取向\nD.政治地位差异", "input": "", "output": "世家和豪族在经济地位和血缘关系上同属宗法封建性大地主, 但是在政治上属于不同的等级和阶层, 即统治阶层和非统治阶层。其中世家指权利世袭的家族系统, “世官”是世家的根本条件, 包括官僚世家、地主世家、儒学世家等, 而豪族则是一种地方势力, 故 $D$ 项正确。世家和豪族都有较强的宗族势力和较多的土地, 在文化价值取向上都尊崇儒学, $A B C$ 项排除。", "task_type": {"major": ["试题"], "minor": ["考研", "选择题"]}, "domain": ["历史"], "metadata": "year:2023, score:4", "answer_from": "human", "human_verified": false, "copyright": "暂无版权及作者信息"} +{"instruction": "北魏末年, 统治阶层奢侈腐败, 社会矛盾尖锐, 528 年尔朱荣发动\"河阴之变\",沉胡太后和元钊于黄河, 并诛杀了朝廷公卿两千余人, 其利用的是\nA. 部落势力\nB. 六镇戍兵\nC. 羽林禁军\nD. 豪强武装", "input": "", "output": "尔朱荣本身属于契胡部落, 但在奉命征讨六镇的过程中收降了大批六镇将领和士兵, 实力空前膨胀并形成军事政治集团, 为其发动\"河阴之变\"积累了军事力量, 故 B 项正确。", "task_type": {"major": ["试题"], "minor": ["考研", "选择题"]}, "domain": ["历史"], "metadata": "year:2023, score:4", "answer_from": "human", "human_verified": false, "copyright": "暂无版权及作者信息"} +{"instruction": "下列中国古史典籍中,依照《通典》体例与主体内容编纂而成的是\nA. 《史通》\nB. 《资治通鉴》\nC. 《通志》\nD. 《文献通考》", "input": "", "output": "马端临《文献通考》全书以杜佑《通典》为蓝本, 在《通典》基础上新加《经籍》、《物异》等 5 门内容, 体例与《通典》相同, 为典志体史书, 故 D 项正确。《史通》是刘知几的史学理论著作, 《资治通鉴》是司马光作的编年体史书, 《通志》是郑樵所作的纪传体通史, 三者在内容体例方面均与《通典》不同, 故 A、B、C 三项错误。", "task_type": {"major": ["试题"], "minor": ["考研", "选择题"]}, "domain": ["历史"], "metadata": "year:2023, score:4", "answer_from": "human", "human_verified": false, "copyright": "暂无版权及作者信息"} +{"instruction": "辽朝捺钵制度分为春捺钭制度(春水), 秋捺钵制度( 秋山), 其目的是A. 接受各部族首领前来朝贡B. 各部首领交流渔猎技艺C. 协调朝廷与各属部的关系D. 处理辽朝对外交往事务", "input": "", "output": "四时捺钭制度是辽朝皇帝在四季带领军队臣僚出巡、加强对地方控制的制度。其中春捺钵期间周围千里之内的属国、属部首领都要到捺钵朝见辽帝, 以示臣服, 秋捺钵也要召见各部落首领, 游猎养性和朝贡只是其表层行为, 根本目的是控制和安抚属部, 协调朝廷与各属部的关系, 故 C 项正确。", "task_type": {"major": ["试题"], "minor": ["考研", "选择题"]}, "domain": ["历史"], "metadata": "year:2023, score:4", "answer_from": "human", "human_verified": false, "copyright": "暂无版权及作者信息"} +{"instruction": "1859 年, 英国对法国进口货物征收大约 15\\%的进口税, 比当时中国对同类货物所征收的进口税率高 2-3 倍, 表明A. 清朝开始丧失关税自主权B.清朝降低关税以增加进口C. 列强利用协定关税以压低中国进口关税D. 列强对华经济侵略形式发生了根本变化", "input": "", "output": "本题考察的是协定关税的相关知识点。材料指出对同类货物, 中国进口关税远远低于英国, 这是由于清政府与列强间的协定关税政策, 列强可利用协定关税政策压低中国关税, 从而向中国倾销商品, 故选 C;鸦片战争后, 1842 年中英《南京条约》规定协定关税, 清朝就开始丧失关税自主权, 故 A 错误; 中国关税较低是列强依据协定关税所要求的, 非清政府自己降低关税, 故 B 错误; 材料只说了中国关税较低的问题, 并无显示出列强对华经济侵略形式发生了根本变化,故 D 错误。", "task_type": {"major": ["试题"], "minor": ["考研", "选择题"]}, "domain": ["历史"], "metadata": "year:2023, score:4", "answer_from": "human", "human_verified": false, "copyright": "暂无版权及作者信息"} +{"instruction": "1905 年, 清朝颁布《各学堂奖励章程》, 根据毕业考试结果获得相应的奖励出身, 如高等学堂毕业生授予举人功名, 中学堂和高等小学堂授予生员功名, 官府根据不同情形分别予以选录, 此举A.减轻了教育改革阻力\nB. 使新式学堂名存实亡\nC.消除立宪党人的反对\nD. 维持了旧有考试制度", "input": "", "output": "本题考察的是清末新政的相关知识点。材料指出官府可以根据学生不同的学堂学历授予与科举相似的身份, 其意在减轻新政教育改革的阻力, 尤其是准备科举的生员的反对, 故选 A; 清末新政中, 废除了科举制, 推行新式教育, 使新式学堂得到了迅速发展, 故 $B 、 D$ 错误; 立宪党人并非全部反对教育改革, 且较多主张废除科举, 故 C 错误。", "task_type": {"major": ["试题"], "minor": ["考研", "选择题"]}, "domain": ["历史"], "metadata": "year:2023, score:4", "answer_from": "human", "human_verified": false, "copyright": "暂无版权及作者信息"} +{"instruction": "1941 年 5 月, 毛泽东在延���干部工作会议上尖锐批判主观主义作风, 号召全党树立理论与实际相统一的马克思主义作风, 为整风运动做了准备, 这一报告是\nA. 《反对党八股》\nB. 《改造我们的学习》\nC. 《整顿党的作风》\nD. 《反对本本主义》", "input": "", "output": "1941 年 5 月毛泽东所作的《改造我们的学习》的报告, 批评了我们党内在教育学习上存在的所学与所用、理论与实际相脱节的毛病, 号召全党要树立理论与实际相结合的马克思主义学风, 由此开启了全党整风学习的序幕, 故选 B。《整顿党的作风》和《反对党八股》是毛泽东在 1942 年作的报告, 时间不符,故 A、C 错误; 《反对本本主义》是毛泽东在 1930 年针对党内存在的教条主义错误倾向而写的文章,故 D 错误。", "task_type": {"major": ["试题"], "minor": ["考研", "选择题"]}, "domain": ["历史"], "metadata": "year:2023, score:4", "answer_from": "human", "human_verified": false, "copyright": "暂无版权及作者信息"} +{"instruction": "1954 年, 我国某企业利润分配表显示, 国家税收占 34.5\\%,职工福利占 $15 \\%$,公积金占 $30 \\%$,资本家红利占 $20.5 \\%$, 该企业性质是\nA. 国家资本主义\nB. 民族资本主义\nC.社会主义公有制\nD. 初级合作社", "input": "", "output": "本题考察的是对资本主义工商业改造的相关知识点。国家对资本主义工商业的改造是通过多种形式的国家资本主义形式, 把资本主义私有制逐步改造为社会主义的全民所有制,材料所述\"四马分肥\"主要指资本主义工商业改造的第二阶段-一个别企业公私合营阶段的企业利润分配方式, 故选 A; 工商业改造前的企业性质是民族资本主义, 完成改造后是社会主义公有制, 故 B、C 错误; 初级合作社是农业社会主义改造的发展阶段,和企业利润分配无关,故 D 错误。", "task_type": {"major": ["试题"], "minor": ["考研", "选择题"]}, "domain": ["历史"], "metadata": "year:2023, score:4", "answer_from": "human", "human_verified": false, "copyright": "暂无版权及作者信息"} +{"instruction": "1984 年, 邓小平南巡指出: \"我们建立经济特区, 实行开放政策, 有个指导思想要明确, 就是不是收, 而是放。\"当时的主要国内背景是\nA. 社会经济发展速度放慢\nB. 产生姓\"社\"还是姓\"资\"争论\nC.受国际形势变动的不利影响\nD.社会主义初级阶段理论的创立", "input": "", "output": "20 世纪 80 年代初, 中央决定在深圳、珠海、汕头和厦门试办经济特区,但关于特区\"姓资姓社\"的争论始终不断, 试办特区的城市承受了巨大的压力。鉴于这种背景,1984 年邓小平南巡,故选 B。1984 年,社会经济发展速度并未放缓, 故 A 错误; 受国际形势变动的不利影响主要值得是 1992 年南巡, 故 C 错误;社会主义初级阶段理论的创立在十三大以后, 时间不符, 故 D 错误。", "task_type": {"major": ["试题"], "minor": ["考研", "选择题"]}, "domain": ["历史"], "metadata": "year:2023, score:4", "answer_from": "human", "human_verified": false, "copyright": "暂无版权及作者信息"} +{"instruction": "印度列国时代, 阶级矛盾突出, 涌现出许多新思潮, 这些新思潮的共同特点是\nA. 批判专制王权\nB. 反对婆罗门教\nC. 抨击宿命思想\nD. 否定生死轮回", "input": "", "output": "印度列国时代, 出现了很多种新的思潮, 这些思潮的共同特点就是反对婆罗门教在思想领域的一家独大地位,从各个角度对婆罗门教进行冲击,故选 B。", "task_type": {"major": ["试题"], "minor": ["考研", "选择题"]}, "domain": ["历史"], "metadata": "year:2023, score:4", "answer_from": "human", "human_verified": false, "copyright": "暂无版权及作者信息"} +{"instruction": "在埃及中部阿玛尔纳出土的楔形文字诏书中,法老与中巴比伦王国之间的通信占比较大, 内容主要为中巴比伦所面临的威胁, 该威胁主要指\nA. 喜克索斯人的入侵\nB. 神庙祭司阶层兴起\nC.中亚述王国的兴起\nD.腓尼基人殖民扩张", "input": "", "output": "在埃及阿玛尔纳曾发现大批泥板文献,是公元前 1385 年至公元前 1355年间,埃及法老与当时西亚各国国王之间往来的书信,被称为阿玛尔纳书信集,周边国家的发展态势构成二人通信的主要内容, 尤其是亚述的日益崛起, 是两人都担心的内容, 故选 C。", "task_type": {"major": ["试题"], "minor": ["考研", "选择题"]}, "domain": ["历史"], "metadata": "year:2023, score:4", "answer_from": "human", "human_verified": false, "copyright": "暂无版权及作者信息"} +{"instruction": "1164 年, 英格兰颁布《克拉伦登宪章》,规定: 未经国王允许, 高级神职人员不得离开王国, 教会领地领主空缺, 国王有权占有收益, 反映了\nA. 国王司法权力扩大\nB. 等级代表会议建立\nC. 国王与罗马教廷分裂\nD. ��大宪章》原则", "input": "", "output": "《克拉伦登宪章》是 1164 年英王亨利二世颁布的扩大王权, 限制教会特权和教会法庭权力的宪章。亨利二世企图以此收回教会的圣职推荐权, 以此来加强国王的司法权力, 进一步强化王权, 故 A 正确。等级代表会议出现在 13 世纪\n后, 故 B 错误, 此时英王还没有与罗马分裂, 只是有矛盾, C 错误, 《大宪章》颁布于 1215 年, D 错误。\n\n因此答案为:A", "task_type": {"major": ["试题"], "minor": ["考研", "选择题"]}, "domain": ["历史"], "metadata": "year:2023, score:4", "answer_from": "human", "human_verified": false, "copyright": "暂无版权及作者信息"} +{"instruction": "中古阿拉伯文明促进了东西之间交流,能印证史实A. 印度数字传播B. 训民正音创制C.犍陀罗艺术诞生D. 那烂陀寺建造", "input": "", "output": "本题目主要考查阿拉伯文明在东西方之间的文化传播作用, 我们所熟知的阿拉伯数字, 实际是印度数字, 只不过由阿拉伯人传到欧洲, 以讹传讹命名为阿拉伯数字, 故选 A。B、C、D 都是地方性文明代表成果, 无法体现东西方之间的文化交流与阿拉伯人在其中的作用。", "task_type": {"major": ["试题"], "minor": ["考研", "选择题"]}, "domain": ["历史"], "metadata": "year:2023, score:4", "answer_from": "human", "human_verified": false, "copyright": "暂无版权及作者信息"} +{"instruction": "15 世纪以前, 东方香料、丝绸、瓷器等长途贩运至欧洲, 价格高昂。此贸易兴盛时, 欧洲与东方往来最密切区域是\nA. 北海、波罗的海\nB. 地中海贸易区\nC. 香槟集市\nD.大西洋贸易区", "input": "", "output": "本题主要考查中世纪的经济, 中世纪时地中海贸易区主要承接的就是来自东方的转口贸易, 来自东方的货物经过丝绸之路, 再由君士坦丁堡分发至整个地中海贸易区, 尤其是意大利商人在其中发挥重要作用, 故选 B。其他选项在当时主要是欧洲区域内部的贸易区。", "task_type": {"major": ["试题"], "minor": ["考研", "选择题"]}, "domain": ["历史"], "metadata": "year:2023, score:4", "answer_from": "human", "human_verified": false, "copyright": "暂无版权及作者信息"} +{"instruction": "1588 年, 丰臣秀吉发布\"刀狩令\", 以建造大佛需要大量铁器为由, 收缴民间一切武器, 目的是\nA. 垄断国内武器制造\nB. 践行佛教和平主张\nC.扶持国内农业生产\nD.推行兵农分离政策", "input": "", "output": "本题主要考查古代日本的基本史实, 丰臣秀吉颁布刀狩令的主要目的是解决战国后期日益严重的流浪武士和武器泛滥问题, 以此来实现兵农分离, 打击非幕府的武装力量, 故 D 项正确。A 项不符合基本史实, B 项过于表面, C 项题干中未体现。", "task_type": {"major": ["试题"], "minor": ["考研", "选择题"]}, "domain": ["历史"], "metadata": "year:2023, score:4", "answer_from": "human", "human_verified": false, "copyright": "暂无版权及作者信息"} +{"instruction": "英国首相 1898 年说: \"大国的斗争已经不再像 1830 年或者甚至 1860 年那样 - - 只限于欧洲问题, 而是牵涉到遍布全球的市场和领土问题\"。主要表现为\nA. 自由竞争、商品输出\nB. 资本输出、加强垄断C. 推销机器、掠夺资源 \nD.技术封锁、关税壁垒", "input": "", "output": "本题主要考查资本主义世界经济体系的形成,1898 年世界已经进入资本输出阶段,资本主义世界市场已经形成,世界已经基本上瓜分完毕,因此各国在世界市场上的竞争进入存量阶段而日益激烈,加强垄断是各国必然的选择,故选 B。", "task_type": {"major": ["试题"], "minor": ["考研", "选择题"]}, "domain": ["历史"], "metadata": "year:2023, score:4", "answer_from": "human", "human_verified": false, "copyright": "暂无版权及作者信息"} +{"instruction": "到 1944 年,美国有 100 万黑人进入制造业工作,其中 30 万是妇女,大量黑人还参加反法西斯战争,这说明\nA. 种族就业隔离正式废除\nB. 黑人走上了武装抗暴道路\nC. 黑人民权运动逐渐高涨\nD. 美国女性获得选举权", "input": "", "output": "本题主要考查黑人民权问题,在 1944 年,二战这个特殊的时间节点上,由于战争的需要, 美国政府迫不得已让大量黑人参军与参加工作以满足前线的需要, 黑人民权运动逐渐高涨即是结果也是原因, 故选 $\\mathrm{C}$ 。种族就业隔离正式废除在 20 世纪 60 年代后, 故 A 错误; 材料并未体现黑人武装争取权力, 故 B 错误; 1920 年, 美国妇女获得了选举权, 故 D 错误", "task_type": {"major": ["试题"], "minor": ["考研", "选择题"]}, "domain": ["历史"], "metadata": "year:2023, score:4", "answer_from": "human", "human_verified": false, "copyright": "暂无版权及作者信息"} +{"instruction": "1959 年, 艾森豪威尔邀请赫鲁晓夫访问美国, 会��中, 美国拒绝苏联提出的裁军议题, 苏联拒绝美国提出的核武器议题, 但双方就柏林问题举行四国首脑会议, 这表明\nA. 多极化趋势冲击两极格局\nC.美苏意图共同终结冷战\nB. 冷战进入苏攻美守阶段\nD.美苏在对抗中寻求平衡", "input": "", "output": "本题主要考查冷战史, 1959 年冷战逐渐进入高潮, 但是美苏之间依旧维持着必要的互动, 以防止局势无序升级, 在对抗中寻求平衡, 故选 D。多极化趋势还未产生, A 错误。美苏没有意图终结冷战, C 错误。 20 世纪 70 年代才是苏攻美守阶段, $\\mathrm{B}$ 错误。", "task_type": {"major": ["试题"], "minor": ["考研", "选择题"]}, "domain": ["历史"], "metadata": "year:2023, score:4", "answer_from": "human", "human_verified": false, "copyright": "暂无版权及作者信息"} +{"instruction": "二战后, 列维-斯特劳斯对存在主义哲学提出批判, 他所代表的哲学思潮被称为\nA. 结构主义\nB. 理性主义\nC. 科学哲学\nD. 现代解释学", "input": "", "output": "本题考查现代思想的基本史实,列维-斯特劳斯代表的是结构主义思潮,故选 A", "task_type": {"major": ["试题"], "minor": ["考研", "选择题"]}, "domain": ["历史"], "metadata": "year:2023, score:4", "answer_from": "human", "human_verified": false, "copyright": "暂无版权及作者信息"} +{"instruction": "油价上涨 $5 \\%$ 后, 加一箱油比原来多花 20 元; 一个月后油价下降 $4 \\%$, 则现在加一箱油需要花 ( ) 元.\n(A) 384\n(B) 401\n(C) 402.8\n(D) 403.2\n(E) 404", "input": "", "output": "设原来加一箱油需要花 $x$ 元, 油价上涨 $5 \\%$ 后, 多花 20 元, 可列式\n$$\n5 \\% x=20 \\Rightarrow x=400 \\text {, }\n$$\n\n因此涨价后一箱油需要 420 元, 一个月后油价下降 $4 \\%$, 则所需花费为 $420 \\times(1-4 \\%)=403.2$元\n\n因此答案为:D", "task_type": {"major": ["试题"], "minor": ["考研", "选择题"]}, "domain": ["管理学"], "metadata": "year:2023, score:3", "answer_from": "human", "human_verified": false, "copyright": "暂无版权及作者信息"} +{"instruction": "已知甲、乙两公司的利润之比为 $3: 4$, 甲、丙两公司的利润之比为 $1: 2$. 若乙公司的利润为 3000 万元, 则丙公司的利润为 ( ) 万元.\n(A) 5000\n(B) 4500\n(C) 4000\n(D) 3500\n(E) 2500", "input": "", "output": "由题意可得, 甲:乙 $=3: 4$, 甲:丙 $=1: 2$, 根据甲的量进行比例统一, 可得甲:乙:丙 $=3: 4: 6$.由乙公司的实际利润为 3000 万元, 可知 4 份代表 3000 , 每份代表 750 , 则丙公司的利润为 $6 \times 750=4500$ (万元).\n\n因此答案为:B", "task_type": {"major": ["试题"], "minor": ["考研", "选择题"]}, "domain": ["管理学"], "metadata": "year:2023, score:3", "answer_from": "human", "human_verified": false, "copyright": "暂无版权及作者信息"} +{"instruction": "一个分数的分母和分子之和为 38 , 其分子分母都减去 15 , 约分后得到 $\\frac{1}{3}$, 则这个分数的分母与分子之差为 ( ).\n(A) 1\n(B) 2\n(C) 3\n(D) 4\n(E) 5", "input": "", "output": "方法一: 设分子为 $x$, 分母为 $38-x$, 根据题意可得 $\frac{x-15}{38-x-15}=\frac{1}{3}$, 解得 $x=17$,\n分母 $=38-17=21$,\n则分子分母之差为 $21-17=4$.\n方法二:当这个分数的分子与分母都减去 15 之后,分子与分母之和为 $38-15 \times 2=8$; 约分后得到 $\frac{1}{3}$, 可得分子与分母的份数之和为 $1+3=4$, 显然约分时约去了 2 倍. 即分子为 2 , 分母为 6 ,\n分子分母之差为 4 , 都加上 15 之后, 原分子分母之差不变, 故原来这个分数的分母与分子之差也为 4 .\n\n因此答案为:D", "task_type": {"major": ["试题"], "minor": ["考研", "选择题"]}, "domain": ["管理学"], "metadata": "year:2023, score:3", "answer_from": "human", "human_verified": false, "copyright": "暂无版权及作者信息"} +{"instruction": "某公司财务部有男员工 2 名, 女员工 3 名; 销售部有男员工 4 名, 女员工 1 名. 现要从中选出 2 男 1 女组成工作小组, 并要求每个部门至少有 1 名员工人选, 则工作小组的不同构成方式有 ( ) 种.\n(A) 24\n(B) 36\n(C) 50\n(D) 51\n(E) 68", "input": "", "output": "方法一: 从反面考虑.反面为 2 男 1 女全部来自于财务部或者全部来自于销售部.任选 2 男 1 女的总的情况数为 $\\mathrm{C}_{6}^{2} \\mathrm{C}_{4}^{1}$, 全部来自于财务部的情况数为 $\\mathrm{C}_{2}^{2} \\mathrm{C}_{3}^{1}$, 全部来自于销售部的情况数为 $\\mathrm{C}_{4}^{2} \\mathrm{C}_{1}^{1}$.因此正面情况为 $\\mathrm{C}_{6}^{2} \\mathrm{C}_{4}^{1}-\\mathrm{C}_{2}^{2} \\mathrm{C}_{3}^{1}-\\mathrm{C}_{4}^{2} \\mathrm{C}_{1}^{1}=60-3-6=51$ (种).\n\n因此答案为:D", "task_type": {"major": ["试题"], "minor": ["考研", "选择题"]}, "domain": ["管理学"], "metadata": "year:2023, score:3", "answer_from": "human", "human_verified": false, "copyright": "暂无版权及作者信息"} +{"instruction": "甲、乙两人从同一地点出发, 甲先出发 10 分钟, 若乙跑步追赶甲, 则 10 分钟可追上; 若乙骑车追赶甲, 每分钟比跑步多行 100 米, 则 5 分钟可追上, 那么甲每分钟走的距离为 ( ) 米.\n(A) 50\n(B) 75\n(C) 100\n(D) 125\n(E) 150", "input": "", "output": "设甲的速度为 $v_{1}$ 米/分钟, 乙跑步的速度为 $v_{2}$ 米/ 分钟, 则甲先出发 10 分钟后, 甲乙两人之间的距离为 $10 v_{1}$ 米.根据乙两次追及的时间不同, 可列方程组为 $\\left\\{\begin{array}{l}\\left(v_{2}-v_{1}\right) \times 10=10 v_{1}, \\ \\left(100+v_{2}-v_{1}\right) \times 5=10 v_{1},\\end{array}\right.$, 解得 $v_{1}=100$ 米 / 分钟.\n\n因此答案为:C", "task_type": {"major": ["试题"], "minor": ["考研", "选择题"]}, "domain": ["管理学"], "metadata": "year:2023, score:3", "answer_from": "human", "human_verified": false, "copyright": "暂无版权及作者信息"} +{"instruction": "由于疫情防控, 电影院要求不同家庭之间至少间隔一个座位, 同一家庭的成员座位要相连. 两个家庭看电影, 一家 3 人, 一家 2 人, 现有一排 7 个相连的座位, 则符合要求的坐法有 ( ) 种。\n(A) 36\n(B) 48\n(C) 72\n(D) 144\n(E) 216", "input": "", "output": "由于每个家庭的成员之间采用捆绑法, 捆绑元素内部排序为 $\\mathrm{A}_{3}^{3} \\mathrm{~A}_{2}^{2}$;还剩下两个空発子, 由于两个家庭之间不能相邻, 采用插空法, 空発子中间形成 3 个空,因此插空排序为 $\\mathrm{A}_{3}^{2}$; 故符合要求的坐法为 $\\mathrm{A}_{3}^{2} \\mathrm{~A}_{3}^{3} \\mathrm{~A}_{2}^{2}=72$ (种).\n\n因此答案为:C", "task_type": {"major": ["试题"], "minor": ["考研", "选择题"]}, "domain": ["管理学"], "metadata": "year:2023, score:3", "answer_from": "human", "human_verified": false, "copyright": "暂无版权及作者信息"} +{"instruction": "方程 $x^{2}-3|x-2|-4=0$ 的所有实根之和为 ( ).\n(A) -4\n(B) -3\n(C) -2\n(D) -1\n(E) 0", "input": "", "output": "当 $x \\geq 2$ 时, 方程化为 $x^{2}-3(x-2)-4=x^{2}-3 x+2=0$, 解得 $x=1$ (舍去) 或 $x=2$;当 $x<2$ 时, 方程化为 $x^{2}-3(2-x)-4=x^{2}+3 x-10=0$, 解得 $x=2$ (舍去)或 $x=-5$.故方程的所有实根之和为 $-5+2=-3$.\n\n因此答案为:B", "task_type": {"major": ["试题"], "minor": ["考研", "选择题"]}, "domain": ["管理学"], "metadata": "year:2023, score:3", "answer_from": "human", "human_verified": false, "copyright": "暂无版权及作者信息"} +{"instruction": "跳水比赛中, 裁判给某选手的一个动作打分, 其平均值为 8.6 , 方差为 1.1 , 若去掉一个最高得分 9.7 和一个最低得分 7.3 , 则剩余得分的 ( ).\n(A)平均值变小, 方差变大\n(B)平均值变小, 方差变小\n(C)平均值变小, 方差不变\n(D) 平均值变大, 方差变大\n(E)平均值变大, 方差变小", "input": "", "output": "去掉一个最高得分和一个最低得分后, 整体得分的极差在变小, 故方差变小;最高分比平均分高 9.7-8.6=1.1(分), 最低分比平均分低 $8.6-7.3=1.3$ (分), $1.3>1.1$.显然将最高分和最低分同时去掉, 平均分将比之前提高.\n\n因此答案为:E", "task_type": {"major": ["试题"], "minor": ["考研", "选择题"]}, "domain": ["管理学"], "metadata": "year:2023, score:3", "answer_from": "human", "human_verified": false, "copyright": "暂无版权及作者信息"} +{"instruction": "设 $x$ 为正实数, 则 $\\frac{x}{8 x^{3}+5 x+2}$ 的最大值为 ( ).\n(A) $\\frac{1}{15}$\n(B) $\\frac{1}{11}$\n(C) $\\frac{1}{9}$\n(D) $\\frac{1}{6}$\n(E) $\\frac{1}{5}$", "input": "", "output": "原代数式可化为 $\\frac{1}{8 x^2+\\frac{2}{x}+5}$, 分母应用均值不等式, 得\n$$\n8 x^2+\\frac{2}{x}+5=\\frac{1}{x}+\\frac{1}{x}+8 x^2+5 \\geq 3 \\sqrt[3]{\\frac{1}{x} \\cdot \\frac{1}{x} \\cdot 8 x^2}+5=11 .\n$$\n\n故分母最小值为 11 , 因此代数式的最大值为 $\\frac{1}{11}$\n\n因此答案为:B", "task_type": {"major": ["试题"], "minor": ["考研", "选择题"]}, "domain": ["管理学"], "metadata": "year:2023, score:3", "answer_from": "human", "human_verified": false, "copyright": "暂无版权及作者信息"} +{"instruction": "快递员收到 3 个同城快递任务, 取送地点各不相同, 取送件可穿插进行, 不同的送件方式有 ( ) 种.\n(A) 6\n(B) 27\n(C) 36\n(D) 90\n(E) 360", "input": "", "output": "假设三个取件地点分别为 $A 、 B 、 C$, 三个对应的送件地点分别为 $\\mathrm{a} 、 \\mathrm{~b} 、 \\mathrm{c}$.方法一:快递员送完所有快递需要将上述六个地点全部经过, 因此经过六个地点的全部顺序为 $\\mathrm{A}_{6}^{6}$.但实际情况需要先取快递再送快递, 因此相同字母之间一定是大写字母在前, 小写字母在后,顺序固定, 需要消序, 故不同的情况数有 $\frac{\\mathrm{A}_{6}^{6}}{\\mathrm{~A}_{2}^{2} \\mathrm{~A}_{2}^{2} \\mathrm{~A}_{2}^{2}}=90$ (种).\n方法二:在六个地点选两个作为 $\\mathrm{A}$ 和 $\\mathrm{a}$, 其中取件点在前, 送件点在后, 不用考虑顺序, 方法数为 $\\mathrm{C}_{6}^{2}$; 从剩下四个地点选两个作为 $\\mathrm{B}$ 和 $\\mathrm{b}$, 方法数为 $\\mathrm{C}_{4}^{2}$, 最后两个地点作为 $\\mathrm{C}$ 和 $\\mathrm{c}$,方法数为 $\\mathrm{C}_{2}^{2}$, 总方法数为 $\\mathrm{C}_{6}^{2} \\mathrm{C}_{4}^{2} \\mathrm{C}_{2}^{2}=90$ (种).\n\n因此答案为:D", "task_type": {"major": ["试题"], "minor": ["考研", "选择题"]}, "domain": ["管理学"], "metadata": "year:2023, score:3", "answer_from": "human", "human_verified": false, "copyright": "暂无版权及作者信息"} +{"instruction": "爱因斯坦思想深刻、思维创新。他不仅是一位伟大的科学家, 还是一位思想家和人道主义者,同时也是一位充满个性的有趣人物。他一生的经历表明, 只有拥有诙谐幽默、充满个性的独立人格, 才能做到思想深刻、思维创新。根据以上陈述, 可以得出以下哪项?\n(A) 有的思想家不是人道主义者。\n(B)有些伟大的科学家拥有诙谐幽默、充满个性的独立人格。\n(C)科学家一旦诙谐幽默、充满个性, 就能做到思想深刻、思维创新。\n(D)有些人道主义者诙谐幽默、充满个性,但做不到思想深刻、思维创新。\n(E)有的思想家做不到诙谐幽默、充满个性,但能做到思想深刻、思维创新。", "input": "", "output": "题干既有假言命题, 又有性质命题, 但皆可表示为箭头, 故可当作假言串联模型解题。同时, 题干条件符合双 A 串联公式。\n第 1 步: 画箭头。(1)爱因斯坦 $\rightarrow$ 思想深刻、思维创新。(2)爱因斯坦 $\rightarrow$ 科学家 $\\wedge$ 思想家 $\\wedge$ 人道主义者 $\\wedge$ 充满个性的有趣人物。(3)思想深刻、思维创新 $\rightarrow$ 诙谐幽默、充满个性。\n第 2 步: 串联。串联(1)和(3)可得: (4)爱因斯坦 $\rightarrow$ 思想深刻、思维创新 $\rightarrow$ 诙谐幽默、充满个性。\n第 3 步: 分析选项, 找答案。\nA 项, 由(2)可知, 有的思想家是人道主义者, 与此项是下反对关系, 一真另不定, 故此项可真可假。\nB 项, 条件(2)和(4), 符合双 A 串联公式, 即: 由(2)可得:有的科学家 $\rightarrow$ 爱因斯坦, 从而与(4)串联得: (5)有的科学家 $\rightarrow$ 爱因斯坦 $\rightarrow$ 思想深刻、思维创新 $\rightarrow$ 诙谐幽默、充满个性。故此项为真。\nC 项, 诙谐幽默、充满个性 $\rightarrow$ 思想深刻、思维创新, 根据箭头指向原则, 由(3)可知, 此项不符合题干。\nD 项, 条件(2)和(4), 符合双 A 串联公式, 即: 有的人道主义者 $\rightarrow$ 爱因斯坦 $\rightarrow$ 思想深刻、思维创新 $\rightarrow$ 诙谐幽默、充满个性。故可知, 有的人道主义者思想深刻、思维创新, 与“有的人道主义者做不到思想深刻、思维创新”是下反对关系, 故此项可真可假。$\\mathrm{E}$ 项, 条件(2)和(4), 符合双 A 串联公式, 即: 有的思想家 $\rightarrow$ 爱因斯坦 $\rightarrow$ 思想深刻、思维创新 $\rightarrow$ 诙谐幽默、充满个性。故可知, 有的思想家诙谐幽默、充满个, 与“有的思想家做不到诙谐幽默、充满个性”是下反对关系, 故此项可真可假。\n\n因此答案为:B", "task_type": {"major": ["试题"], "minor": ["考研", "选择题"]}, "domain": ["管理学"], "metadata": "year:2023, score:2", "answer_from": "human", "human_verified": false, "copyright": "暂无版权及作者信息"} +{"instruction": "处理餐时垃圾的传统方式主要是厌氧发酵和填埋, 前者利用垃圾产生的沼气发电, 投资成本高;后者不仅浪费土地, 还污染环境。近日, 某公司尝试利用蟙螂来处理垃圾。该公司饲养了 3 亿只“美洲大蠊”蟑螂, 每天可以吃掉 15 吨餐厨垃圾。有专家据此认为, 用“蟑螂吃掉垃圾”这一生物处理方式解决餐厨垃圾, 既经济又环保。\n\n\n以下哪项如果为真, 最能质疑上述专家的观点?\n(A)餐㕌垃圾经发酵转化为能源的处理方式已被国际认可,我国这方面的技术也相当成熟。\n(B)大量人工养殖后, 很难保证蟑螂不逃离控制区域, 而一旦蟑螂逃离, 则会危害周边生态环境。\n(C) 政府前期在工厂土地划拨方面对该项目给予了政策扶持,后期仍需进行公共安全检测和环境评估。\n(D)我国动物蛋白饲料非常缺乏, 1 吨蟑螂及其所产生的卵鞘, 可产生 1 吨昆虫蛋白饲料, 饲养蟑螂将来盈利十分可观。\n(E)该公司正在建设新车间, 竣工后将能饲养 20 亿只蟑螂, 它们虽然能吃掉全区的餐厥垃圾,但全市仍有大量餐厨垃圾需要通过传统方式处理。", "input": "", "output": "题干:该公司饲养了 3 亿只美洲大蟑螂, 每天可吃掉 15 吨餐厨垃圾 $\\longrightarrow$ 证明 用“蟑螂吃掉垃圾”这一生物处理方式解决餐厨垃圾, 既经济又环保。“蟙螂吃掉垃圾” 是一种措施, “解决餐厨垃圾, 既经济又环保” 是一种目的, 可知此题为措施且的模型。\nA 项, 无关选项, 该项指出发酵转化处理厨房垃圾是被国际认可且可行的, 与题干的措施“蟑螂吃掉垃圾”无关 (干扰项·转移话题) 。\nB 项,此项说明该种措施会危害周围环境,达不到题干中“环保”这一目的,可以削弱。\nC 项, “后期仍需进行公共安全检测和环境评估”就可能出现两种结果: (1)评估没问题, 则支持题干; (2)评估有问题,则削弱题干(干扰项·两可选项)。\nD 项,此项说明饲养蟑螂可以带来收益,支持题干中的措施。\n$E$ 项, “它们虽然能吃掉全区的餐厨垃圾”, 说明该措施是有效的, 支持题干 (干扰项- 明否暗肯)。\n\n因此答案为:B", "task_type": {"major": ["试题"], "minor": ["考研", "选择题"]}, "domain": ["管理学"], "metadata": "year:2023, score:2", "answer_from": "human", "human_verified": false, "copyright": "暂无版权及作者信息"} +{"instruction": "记者:贵校是如何培养创新型人才的?\n\n受访者: 大学生踊跃创新创业是我校的一个品牌。在相关课程学习中, 我们注重激发学生创业的积极性, 引导学生想创业; 通过实训、体验, 让学生能创业; 通过学校提供专业化的服务,帮助学生创成业。在高校创业者收益榜中, 我们学校名列榜首。\n\n以下哪项最可能是上述对话中受访者论述的假设?\n(A)不懂创新就不懂创业。\n(B)创新能力越强, 创业收益越高。\n(C)创新型人才培养主要是创业技能的培训和提升。\n(D) 培养大学生创业能力只是培养创新型人才的任务之一。\n(E)创新型人才的主要特征是具有不拘陈规、勇于开拓的创新精神。", "input": "", "output": "记者: 贵校是如何培养创新型人才的?受访者首先表明“大学生踊跃创新创业是我校的一个品牌”, 继而介绍了学校在培养创业技能方面做的工作。可见, 记者提问的是“培养创新型人才”, 而受访者回答的是“创新创业”, 二者不一致, 故此题为拆桥搭桥模型。\n$A$ 项, 7 懂创新 $\rightarrow 7$ 懂创业, 即: 懂创业 $\rightarrow$ 懂创新; 题干论证不涉及“懂创新”和“懂创业”之间的关系。\nB 项,无关选项,题干不涉及创新能力对创业收益的影响。\nC 项,搭建了“培养创新型人才”与“创新创业培训”的桥梁,故必须假设。\nD 项,不必假设,即使培养大学生创业能力是培养创新型人才的全部任务也符合题干。\n$E$ 项,无关选项,题干不涉及创新型人才的特征。\n\n因此答案为:C", "task_type": {"major": ["试题"], "minor": ["考研", "选择题"]}, "domain": ["管理学"], "metadata": "year:2023, score:2", "answer_from": "human", "human_verified": false, "copyright": "暂无版权及作者信息"} +{"instruction": "某部门抽检了肉制品、白酒、乳制品、干果、蔬菜、水产品、饮料等 7 类商品共 521 种样品,发现其中合格样品 515 种, 不合格样品 6 种。已知:\n\n(1) 蔬菜、白酒中有 2 种不合格样品。\n\n(2) 肉制品、白酒、蔬菜、水产品中有 5 种不合格样品。\n\n(3) 蔬菜、乳制品、干果中有 3 种不合格样品。\n\n根据上述信息, 可以得出以下哪项?\n(A)乳制品中没有不合格样品。\n(B) 肉制品中没有不合格样品。\n(C) 蔬菜中没有不合格样品。\n(D) 白酒中没有不合格样品。\n(E) 水产品中没有不合格样品。", "input": "", "output": "题干中给出几组数量关系, 故此题为数量关系模型。\n由(1)和(2)可知:(4)肉制品、水产品中有 3 种不合格样品。\n由(3)和(4)结合可知, 蔬菜、乳制品、干果、肉制品、水产品中共有有 6 种不合格样品。因此, 抽检发现的 6 种不合格的样品均出自蔬菜、乳制品、干果、肉制品、水产品, 故白酒和饮料中没有不合格样品。D 项正确", "task_type": {"major": ["试题"], "minor": ["考研", "选择题"]}, "domain": ["管理学"], "metadata": "year:2023, score:2", "answer_from": "human", "human_verified": false, "copyright": "暂无版权及作者信息"} +{"instruction": "时时刻刻总在追求幸福的人不一定能获得最大的幸福, 刘某说自己获得了最大的幸福, 所以,刘某从来不曾追求幸福。\n\n以下哪项与上述论证方式最为相似?\n\n(A)年年岁岁总是帮助他人的人不一定能成为名人, 李某说自己成了名人, 所以, 李某从来不曾帮助他人。\n\n(B) 口口声声不断说喜欢你的人不一定最喜欢你, 陈某现在说他最喜欢你, 所以, 陈某过去从未喜欢过你。\n\n(C)冷冷清清空无一人的商场不一定没有利润,某商场今年亏损,所以,该商场总是空无一人。\n\n(D)日日夜夜一直躲避死亡的士兵反而最容易在战场上丧命, 林某在一次战斗中重伤不治, 所以,林某从来没有躲避死亡。\n\n(E) 分分秒秒每天抢时间工作的人不一定是普通人, 宋某看起来很普通, 所以, 宋某肯定没有每天抢时间工作。", "input": "", "output": "题干的问题为“以下哪项与上述论证方式最为相似”, 可知此题为结构相似题。题干:时时刻刻总在追求幸福的人不一定能获得最大的幸福, 刘某说自己获得了最大的幸福,所以, 刘某从来不曾追求幸福。形式化: A 不一定 B, B, 因此, 从来不 $\\mathrm{A}$ 。\n$\\mathrm{A}$ 项, $\\mathrm{A}$ 不一定 $\\mathrm{B}, \\mathrm{B}$, 因此, 从来不 $\\mathrm{A}$ 。与题干相同。\n$\\mathrm{B}$ 项, $\\mathrm{A}$ 不一定 $\\mathrm{B}$, 现在 $\\mathrm{B}$, 因此, 过去不 $\\mathrm{A}$ 。与题干不同。\n$\\mathrm{C}$ 项, $\\mathrm{A}$ 不一定 $\\mathrm{B}, \\mathrm{C}$ (注意:没有利润本损),因此,总是 $\\mathrm{A}$ 。故与题干不同。\n$\\mathrm{D}$ 项, $\\mathrm{A}$ 容易 $\\mathrm{B}, \\mathrm{B}$, 因此, 从来不 $\\mathrm{A}$ 。与题干不同。\n$\\mathrm{E}$ 项, $\\mathrm{A}$ 不一定 $\\mathrm{B}$, 看起来 $\\mathrm{B}$, 因此, 肯定没有 $\\mathrm{A}$ 。故与题不同。\n\n因此答案为:A", "task_type": {"major": ["试题"], "minor": ["考研", "选择题"]}, "domain": ["管理学"], "metadata": "year:2023, score:2", "answer_from": "human", "human_verified": false, "copyright": "暂无版权及作者信息"} +{"instruction": "某中学举行田径运动会, 高二(3) 班甲、乙、丙、丁、戊、已 6 人报名参赛。在跳远、跳高和铅球 3 项比赛中,他们每人都报名 1 2 项。其中 2 人报名跳远, 3 人报名跳高,3 人报名铅球。另外,还知道:\n\n(1)如果甲、乙至少有 1 人报名铅球,则丙也报名铅球。\n\n(2)如果已报名跳高,则乙和已均报名跳远。\n\n(3)如果丙、戊至少有 1 人报名铅球,则已报名跳高。\n\n根据以上信息, 可以得出以下哪项?\n(A)甲报名铅球,乙报名跳远。\n(B)乙报名跳远,丙报名铅球。\n(C)丙报名跳高, 丁报名铅球。\n(D) 丁报名跳远,戊报名跳高。\n(E)戊报名跳远,已报名跳高。\n\n如果甲、乙均报名跳高,则可以得出以下哪项?\n(A)丁、戊均报名铅球。\n(B)乙、丁均报名铅球。\n(C)甲、戊均报名铅球。\n(D)乙、戊均报名铅球。\n(E)甲、丁均报名铅球。", "input": "", "output": "#### 1. 对于第一个问题\n\n题干由数量、假言命题、匹配关系构成, 故此题为数量假言匹配模型。第 1 步:数量关系优先算由题干“2 人报名跳远, 3 人报名跳高, 3 人报名铅球”可知, 累计报名 8 人次, 已知有 6 人,每人都报名 1-2 项,故 6 人报名项目数量的情况为: 2 项、 2 项、 1 项、 1 项、 1 项、 1 项。第 2 步: 串联找矛盾串联 (1) 、(3) 、(2) 可得: 甲铅球 $\\vee$ 乙铅球 $\rightarrow$ 丙铅球 $\rightarrow$ 丙铅球 $\\vee$ 戊铅球 $\rightarrow$ 己跳高 $\rightarrow$ 乙跳远 $\\wedge$ 己跳远。逆否可得: ${ }_{7}$ 乙跳远 $\\vee_{7}$ 己跳远 $\rightarrow 7$ 己跳高 $\rightarrow_{7}$ 丙铅球 $\\wedge_{7}$ 戊铅球 $\rightarrow_{\rceil}$甲铅球 $\\wedge$ 乙铅球。因此, 由“ๆ乙跳远 $\\vee_{7}$ 己跳远”、“已跳高”均能推出有 4 人不参加铅球, 与“3 人报名铅球”矛盾; 因此,乙和已均报名跳远、已报名跳高。由于己报名了跳高和跳远,因此,甲、乙、丙、丁、戊 5 人中有 3 人报名铅球。再由条件 (1) 可知, 若丙不报名铅球, 则甲、乙均不报名铅球; 与“甲、乙、丙、丁、戊 5 人中由 3 人报名铅球”矛盾, 因此, 丙报名铅球。故 (B) 项正确。\n\n#### 对于第二个问题\n\n此题的提问中又补充新的事实,故此题为事实假言+数量匹配模型。引用上题推理结果:乙和已均报名跳远、已报名跳高、丙报名铅球。本题新补充事实:甲、乙均报名跳高, 故:甲、乙、已三人报名跳高。由于“乙和已均报名跳远”, 结合数量关系可知, 丙、丁、戊三人报名铅球。故 (A) 项正确。", "task_type": {"major": ["试题"], "minor": ["考研", "选择题"]}, "domain": ["管理学"], "metadata": "year:2023, score:2", "answer_from": "human", "human_verified": false, "copyright": "暂无版权及作者信息"} +{"instruction": "进人移动互联网时代, 扫码点餐、在线挂号、网购车票、电子支付等智能化生活日益普及, 人们的生活越来越便捷。然而, 也有很多老年人因为不会使用智能手机等设备, 无法进人菜场、超市和公园, 也无法上网娱乐与购物, 甚至在新冠疫情期间因无法从手机中调出健康码而被拒绝乘坐公共交通。对此,某专家指出,社会正在飞速发展,不可能“慢”下来等老年人,老年人应该加强学习, 跟上时代发展。\n\n\n以下哪项如果为真,最能质疑该专家的观点?\n\n(A)老年人也享有获得公共服务的权利, 为他们保留老办法, 提供传统服务, 既是一种社会保障, 更是一种社会公德。\n\n(B)有些老年人学习能力较强, 能够熟练使用多种电子产品, 充分感受移动互联网时代的美好。\n\n(C) 目前中国有 2 亿多老年人,超 4 成的老年人存在智能手机使用障碍,仅会使用手机打电话。\n\n(D)社会管理和服务不应只有一种模式, 而应更加人性化和多样化, 有些合理的生活方式理应得到尊重。\n\n(E)有些老年人感觉自己被时代抛弃了, 内心常常充斥着客迫与挫败感, 这容易导致他们与社会的加速脱离。", "input": "", "output": "题干: 某专家指出, 社会正在飞速发展, 不可能“慢”下来等老年人, 老年人应该加强学习, 跟上时代发展。注意: 此题专家观点前面的内容, 阐述的均为当前社会中出现的某些现象, 但对专家观点的成立并未起到支撑作用, 故均为背景信息。因此, 本题题干中没有论据只有观点, 为无论据模型。对于题干无论据模型的削弱题,直接分析选项是否质疑题干即可。A 项,此项说明应该设法帮助老年人解决问题,即“慢”下来等老年人,削弱专家的观点。B 项, 有的老年人能够熟练使用各种电子产品, 不能削弱题干中的“有很多老年人不会使用智能手机等设备”(干扰项·有的不)。C 项,此项重复了题干的背景信息,但不影响专家的观点的成立性。$\\mathrm{D}$ 项, 此项指出有的合理的生活方式应得当尊重, 但题干论证的是老年时否应该加强学习以跟上时代的发展, 两者话题并不一致(干扰项·转移话题)。$\\mathrm{E}$ 项,无关选项,题干的论证不涉及“心态”对老年人的影响(干扰项·转移话题)。\n\n因此答案为:A", "task_type": {"major": ["试题"], "minor": ["考研", "选择题"]}, "domain": ["管理学"], "metadata": "year:2023, score:2", "answer_from": "human", "human_verified": false, "copyright": "暂无版权及作者信息"} +{"instruction": "某单位采购了一批图书, 包括科学和人文两大类。具体情况如下:\n\n(1)哲学类图书都是英文版的。\n\n(2) 部分文学类图书不是英文版的。\n\n(3) 历史类图书都是中文版的。\n\n(4) 没有一本书是中英双语版的。\n\n(5)科学类图书既有中文版的, 也有英文版的。\n\n(6) 人文类图书既有哲学类的, 也有文学类的, 还有历史类的。\n\n根据以上信息, 关于该单位采购的这批图书, 可以得出以下哪项?\n(A)有些文学类图书是中文版的。\n(B)有些历史类图书不属于哲学类。\n(C) 英文版图书比中文版图书数量多。\n(D)有些图书既属于哲学类也属于科学类。\n(E)有些图书既属于文学类也属于历史类。", "input": "", "output": "题干出现多个性质命题, 这些性质命题中存在重复元素, 故本题为性质串联模型。题干有以下信息:(1)哲学类 $\rightarrow$ 英文版。(2)有的文学类 $\rightarrow\rceil$ 英文版。(3)历史类 $\rightarrow$ 中文版。(4) 7 ( 中文版 $\\wedge$ 英文版 $)=$ 中文版 $\rightarrow\\urcorner$ 英文版 $=$ 英文版 $\rightarrow\\urcorner$ 中文版。(5)有的科学类 $\rightarrow$ 中文版, 有的科学类 $\rightarrow$ 英文版。(6)有的人文类 $\rightarrow$ 中文版,有的人文类 $\rightarrow$ 英文版,有的人文类 $\rightarrow$ 历史类。$A$ 项, 由(2)可知, 有的文学类 $\rightarrow 7$ 英文版, 但由“ 7 英文版”无法推出是“中文版”, 故此项可真可假。$B$ 项,串联(3)、(4)和(1)可得: 历史类 $\rightarrow$ 中文版 $\rightarrow\rceil$ 英文版 $\rightarrow\rceil$ 哲学类。故此项为真。$\\mathrm{C}$ 项, 题干不涉及数量关系, 故此项可真可假。$\\mathrm{D}$ 项, 此项等价于 “有的科学类图书是哲学类图书”。串联(5)(4)(1)可得: 有的科学类 $\rightarrow$ 中文版 $\rightarrow\rceil$ 英文版 $\rightarrow\rceil$ 哲学类。故有“有的科学类图书不是哲学类图书”, 与此项为下反对关系, 一真另不定。故此项可真可假。$\\mathrm{E}$ 项, (2)可知, 有的文学类 $\rightarrow 7$ 英文版, 但由“ $\\urcorner$ 英文版”无法推出结论, 故此项可真可假。\n\n因此答案为:B", "task_type": {"major": ["试题"], "minor": ["考研", "选择题"]}, "domain": ["管理学"], "metadata": "year:2023, score:2", "answer_from": "human", "human_verified": false, "copyright": "暂无版权及作者信息"} +{"instruction": "曾几何时, “免费服务”是互联网的重要特征之一, 如今这一情况正在发生改变。有些人在网上开辟知识付费平台, 让寻求知识、学习知识的读者为阅读 “买单”, 这改变了人们通过互联网免费阅读的习惯。近年来, 互联网知识付费市场的规模正以连年翻番的速度增长。但是有专家指出, 知识付费市场的发展不可能长久, 因为人们大多不愿为网络阅读付费。\n\n以下哪项如果为真, 最能质疑上述专家的观点?\n\n(A) 高强度的生活节奏使人无法长时间、系统性阅读纸质文本, 见缝插针、随时呈现式的碎片化、网络化阅读��成为获取知识的常态。\n\n(B) 日常工作的劳累和焦虑使得人们更喜欢在业余时间玩网络游戏, 看有趣视频或与好友进行微信聊天。\n\n(C) 日益增长的竞争压力促使当代人不断学习新知识, 只要知识付费平台做得足够好, 他们就愿意为此付费。\n\n(D) 当前网上知识付费平台竞争激烈, 尽管内容丰富, 形式多样, 但是鱼龙混杂、缺少规范, 一些年轻人沉湎其中难以自拔。\n\n(E) 当前, 许多图书资料在互联网上均能免费获得, 只要合理用于自身的学习和研究一般不会产生知识产权问题。", "input": "", "output": "专家: 人们大多不愿为网络阅读付费 $\\longrightarrow$ 证明 知识付费市场的发展不可能长久。题干的论点在对知识付费市场的未来发展情况做出预测, 故此题为预测结果模型。预测结果型的削弱题, 需要我们找个理由来说明结果预测不当。A 项, 无关选项, 此项指出碎片化、网络化阅读已成为获取知识的常态, 但并没有直接涉及知识付费市场的发展。B 项,无关选项,题干的论证不涉及网络游戏、微信聊天等娱乐方式。$\\mathrm{C}$ 项, 说明人们是愿意为知识而付费, 质疑专家的观点。$\\mathrm{D}$ 项,此项指出知识付费平台现存的一些弊端,支持知识付费平台的发展不可能长久。$E$ 项, 此项说明在互联网上能免费获取知识, 那么就不必为了知识而付费, 支持专家的观点。\n\n因此答案为:C", "task_type": {"major": ["试题"], "minor": ["考研", "选择题"]}, "domain": ["管理学"], "metadata": "year:2023, score:2", "answer_from": "human", "human_verified": false, "copyright": "暂无版权及作者信息"} +{"instruction": "甲:如今,独特性正成为中国人的一种生活追求。试想周末我穿一件心仪的衣服走在大街上,突然发现你迎面走来, 和我穿得一模一样, “撞衫” 的感觉八成会是滥尤之中带着一丝不快, 因为自己不再独一无二。\n\n乙: 独一无二真的那么重要吗? 想想上世纪七十年代满大街的中山装、八十年代遍地的喇叭裤,每个人也活得很精彩。再说“撞衫”总是难免的, 再大的明星也有可能“撞衫”, 所谓的独特只是一厢情愿, 走自己的路, 不要管自己是否和别人一样。\n\n以下哪项是对甲、乙对话最恰当的评价?\n(A)甲认为独一无二是现在每个中国人的追求,而乙认为没有人能做到独一无二。\n(B) 甲关心自己是否和别人 “撞衫”,而乙不关心自己是否和别人一样。\n(C)甲认为“撞衫”八成会让自己感到不爽,而乙认为自己想怎么样就怎么样。\n(D) 甲关心的是个人生活的独特性,而乙关心的是个人生活的自我认同。\n(E)甲认为乙遇到“撞衫”无所谓,而乙认为别人根本管不着自己穿什么。", "input": "", "output": "此题的问题为: “以下哪项是对甲、乙对话最恰当的评价”, 但观察选项可知, 本题分析的是甲、乙两人的分歧,故此题为争论焦点模型。甲的观点是: “独特性正成为中国人的一种生活追求”。乙的观点是: “独特只是一厢情愿, 走自己的路, 不要管自己是否和别人一样”。甲、乙两人对话中的“撞衫”仅仅是例子, 并不是两人的观点, 故排除 $\\mathrm{B} 、 \\mathrm{C} 、 \\mathrm{E}$ 三项。A 项, “现在每个中国人的追求”过于绝对, 甲并没有表达这样绝对化的观点, 排除。$\\mathrm{D}$ 项, 甲的观点更倾向于要追求个人独特性, 而乙的观点更倾向于做自, 故此项最为准确的概括了两个人的观点。\n\n因此答案为:D", "task_type": {"major": ["试题"], "minor": ["考研", "选择题"]}, "domain": ["管理学"], "metadata": "year:2023, score:2", "answer_from": "human", "human_verified": false, "copyright": "暂无版权及作者信息"} +{"instruction": "某研究所甲、乙、丙、丁、戊 5 人拟定去我国四大佛教名山普陀山、九华山、五台山、峨眉山考察,他们每人去了上述两座名山,且每座名山均有其中的 2 3 人前往,丙与丁结伴考察,已知:\n\n(1)如果甲去五台山,则乙和丁都去五台山。\n\n(2)如果甲去峨眉山,则丙和戊都去峨眉山。\n\n(3)如果甲去九华山,则戊去九华山和普陀山。\n\n根据以上信息, 可以得出以下哪项?\n(A) 甲去五台山和普陀山。\n(B)乙去五台山和峨眉山。\n(C)丙去九华山和五台山。\n(D) 戊去普陀山和峨眉山。\n(E)丁去峨眉山和五台山。\n\n如果乙去普陀山和九华山, 则 5 人去四大名山 (按题干所列顺序) 的人次之比是:\n(A) $3: 3: 2: 2$\n(B) $2: 3: 3: 2$\n(C) $2: 2: 3: 3$\n(D) $3: 2: 2: 3$\n(E) $3: 2: 3: 2$", "input": "", "output": "#### 第一题\n\n本题中的已知条件由数量、假言命题、匹配关系组成, 故此题为数量假言匹配模型。第 1 步: 数量关系优先算。\n\n每人去了上述两座名山, 即: 五人共去了 10 个趟次; 结合每座名山均有其中的 2 3 人前往,可知, 名山对应的人次为: 3 次、 3 次、 2 次、 2 次\n\n第 2 步: 数量关系找矛盾。\n由“丙和丁结伴考察”可知, 若条件 (1) 和条件 (2) 的前件为真, 则五台山、峨眉山均会有 4 人前往, 与题干数量矛盾。因此, 条件 (1) 和 (2) 的前件均为假, 即: 甲不去五台山, 也不去峨眉山。\n\n再结合每人 去了两座名山可知, 甲去普陀山和九华山。\n由甲去九华山可知, 条件 (3) 前件为真, 根据口诀“肯前必肯后”, 可得: 戊去九华山和普 陀山。此时, 可得下表:\n\n| | 普陀山 | 九华山 | 五台山 | 峨眉山 |\n| :---: | :---: | :---: | :---: | :---: |\n| 甲 | $\\sqrt{ }$ | $\\sqrt{ }$ | $\\times$ | |\n| 乙 | | | | |\n| 丙、丁 | | | | |\n| 戊 | $\\sqrt{ }$ | $\\sqrt{ }$ | $\\times$ | $\\times$ |\n\n九华山和普陀山均已有 2 人前往。故由“丙和丁结伴考察”结合“每座名山均有其中的 2 3 人前往”可知, 丙、丁不去九华山, 也不去普陀山。\n\n再有“每人去了上述两座名山”可知, 丙、丁去峨眉山和五台山。即:\n\n| | 普陀山 | 九华山 | 五台山 | 峨眉山 |\n| :---: | :---: | :---: | :---: | :---: |\n| 甲 | $\\sqrt{ }$ | $\\sqrt{ }$ | $\\times$ | $\\times$ |\n| 乙 | | | | |\n| 丙、丁 | $\\times$ | $\\times$ | $\\sqrt{ }$ | $\\sqrt{ }$ |\n| 戊 | $\\sqrt{ }$ | $\\sqrt{ }$ | $\\times$ | $\\times$ |\n\n故(E)项正确\n\n#### 第二题\n【解析】此题的提问中又补充新的事实, 故此题为事实假言+数量匹 配模型。\n从事实出发, 由乙去普陀山和九华山, 结合上题的推理结果可得下表:\n\n| | 普陀山 | 九华山 | 五台山 | 峨眉山 |\n| :---: | :---: | :---: | :---: | :---: |\n| 甲 | $\\sqrt{ }$ | $\\sqrt{ }$ | $\\times$ | |\n| 乙 | $\\sqrt{ }$ | $\\sqrt{ }$ | $\\times$ | $\\times$ |\n| 丙丁 | $\\times$ | $\\times$ | $\\sqrt{ }$ | |\n| 戊 | $\\sqrt{ }$ | $\\sqrt{ }$ | $\\times$ | $\\times$ |\n\n根据上表可知, 5 人去四大名山(按题干所列顺序)的人次之比是 $3: 3: 2: 2$ 。\n故 (A) 项正确。", "task_type": {"major": ["试题"], "minor": ["考研", "选择题"]}, "domain": ["管理学"], "metadata": "year:2023, score:2", "answer_from": "human", "human_verified": false, "copyright": "暂无版权及作者信息"} +{"instruction": "水在温度高于 $374^{\\circ} \\mathrm{C}$ 、压力大于 $22 \\mathrm{MPa}$ 的条件下, 称为超临界水。超临界水能与有机物完全互溶, 同时还可以大量溶解空气中的氧, 而无机物特别是盐类在超临界水中的溶解度很低。由此,研究人员认为, 利用超临界水作为特殊溶剂, 水中的有机物和氧气可以在极短时间内完成氧化反应, 把有机物彻底“秒杀”。\n\n\n以下哪项如果为真, 最能支持上述研究人员的观点?\n(A)有机物在超临界水中通过分离装置可瞬间转化为无毒无害的水、无机盐以及二氧化碳等气体, 并最终在生产和生活中得到回收利用。\n(B) 超临界水氧化技术具有污染物去除率高、二次污染小、反应迅速等特征, 被认为是废水处理技术中的“杀手锏”,具有广阔的工业应用前景。\n(C) 超临界水只有兼具气体与液体的高扩散性、高溶解性、高反应活性及低表面张力等优良特性, 才能把有机物彻底“秒杀”。\n(D) 超临界水氧化技术对难以降解的农化、石油、制药等有机废水尤为适用。\n(E)如果超临界水氧化技术成功应用于化工、制药等行业的污水处理, 可有效提升流域内重污染行业的控源减排能力。", "input": "", "output": "题干: 研究人员认为, 利用超临界水作为特殊溶剂, 水中的有机物和氧气可以在极短时间内完成氧化反应,把有机物彻底“秒杀”。“利用超临界水”可看作措施, “把有机物彻底“秒杀” 可看作目的, 故此题也为措施目的模型。措施目的模型的支持题, 常用四种方法: 措施可行, 措施有效, 利大于弊, 措施有必要。A 项, 支持题干, 此项说明确实可以利用超临界水把有机物转化为其他无害的物质, 同时还能实现回收利用,做到“秒杀”,措施有效。其余各项都说明了超临界水的优点,但都没有涉及“有机物”,故均为无关选项。\n\n因此答案为:A", "task_type": {"major": ["试题"], "minor": ["考研", "选择题"]}, "domain": ["管理学"], "metadata": "year:2023, score:2", "answer_from": "human", "human_verified": false, "copyright": "暂无版权及作者信息"} +{"instruction": "小陈与几位朋友商定利用假期到某地旅游。他们在桃花坞、第一山、古生物博物馆、新四军军部旧址、琉璃泉、望江图 6 个景点", "input": "", "output": "本题中的已知条件由数量和假言命题组成, 故此题为数量假言模型。题干假言命题可表示如下:(1)桃花坞 $\rightarrow 7$ 占生物博物馆 $\\wedge$ 望江阁;(2)望��阁 $\rightarrow 7$ 第一山 $\\wedge$ 新四军军部旧址。由(1)、(2)串联得: 桃花坞 $\rightarrow$ 望江阁 $\rightarrow 7$ 第一山 $\\wedge$ 新四军军部旧址;可得: (3)桃花坞 $\rightarrow$ 望江阁 $\rightarrow$ 新四军军部旧址。逆否(3)可得: 7 新四军军部旧址 $\rightarrow 7$ 望江阁 $\rightarrow\rceil$ 桃花坞。故若不去新四军军部旧址, 则有 3 个地方不去, 与 6 选 4 矛盾; 因此, 一定选择新四军军部旧址。故 (C) 项正确。", "task_type": {"major": ["试题"], "minor": ["考研", "选择题"]}, "domain": ["管理学"], "metadata": "year:2023, score:2", "answer_from": "human", "human_verified": false, "copyright": "暂无版权及作者信息"} +{"instruction": "张先生欲花 5 万元购置㴟柜、卫浴或供暖设备。已知:\n(1)如果买棏柜, 就不买卫浴, 也不买供暖设备。\n(2)如果不买榯柜, 就买卫浴。\n(3)如果卫浴、啀柜至少有一种不买, 则购买供暖设备。\n\n根据以上陈述, 关于张先生的购买打算, 可以得出以下哪项?\n(A) 买櫉柜和卫浴。\n(B) 买㴟柜和供暖设备。\n(C) 买椨柜, 但不买卫浴。\n(D) 买卫浴和供暖设备。\n(E) 买卫浴, 但不买供暖设备。", "input": "", "output": "本题中的已知条件全部为假言命题, 选项均为事实, 故此题为假言推事实模型。条件 (3) 、(1) 和 (2) 串联可得: (4) $\\urcorner$ 卫浴 $\\vee\\urcorner$ 崕柜 $\rightarrow$ 供暖设备 $\rightarrow\rceil$ 榶柜 $\rightarrow$ 卫浴。由(4)可得: 卫卫浴 $\rightarrow$ 卫浴。故由“ $\rceil$ 卫浴”出发推出了矛盾, 故“ $\\urcorner$ 卫浴”为假, “卫浴”为真。由“卫浴”为真可知, 条件 (1) 后件为假, 根据口诀“否后必否前”, 可得: 7 樞柜。由“7榶柜”可知, 条件 (3) 前件为真, 根据口诀“肯前比肯后”, 可得: 供暖设备。综上, 一定购买卫浴和供暖设备; 故 (D) 项正确。", "task_type": {"major": ["试题"], "minor": ["考研", "选择题"]}, "domain": ["管理学"], "metadata": "year:2023, score:2", "answer_from": "human", "human_verified": false, "copyright": "暂无版权及作者信息"} +{"instruction": "某台电脑的登录密码由 $0 \\sim 9$ 中的 6 个数组成, 每个数字最多出现一次, 关于该 6 位密码, 已知: (1) 741605 中, 共有 4 个数字正确, 其中 3 个位置正确, 1 个位置不正确。\n\n(2) 320968 中, 恰有 3 个数字正确且位置正确。\n\n(3) 417280 中, 共有 4 个数字不正确。\n\n根据上述信息, 可以得出该登录密码的前两位是:\n(A) 71 。\n(B) 42 。\n(C) 72 。\n(D) $31 。$\n(E) 34 。", "input": "", "output": "此题的 3 个已知条件中, 均给出了 6 位数的密码, 并且均有对错, 故本题可视为二个人多个判断模型。采用重复元素分析法, 观察题干已知条件发现, 条件 (1) 和 (3) 中均涉及 $1 、 4 、 7$; 若 1 、 $4 、 7$ 均为正确的密码数字, 则与条件 (3) 矛盾; 若 $1 、 4 、 7$ 均为错误的密码数字, 则与条件(1)矛盾; 因此,1、4、7有“2 假 1 真”和“2 真 1 假”两种情况。情况较少,可分类讨论: (1)若为“2 假 1 真”, 由条件 (1) 可知“6、0、5”三个数字均正确。由“ 0 和 6 ”均正确, 结合条件 (2) 可知, 0 在第三位、 6 在第五位。此时, 条件 (1) 中“ 0 ”和“ 6 ”的位置均错误, 与条件 (1)矛盾。因此,“2 假 1 真”这种情况不成立。故,1、4、7 这三个数字“ 2 真 1 假”。由条件 (3) 可知, “ $2 、 8 、 0$ ”这三个数均不是正确的密码数字。再结合条件 (2) 可知, 3、9、6 三个数字正确, 且分别是第一个、第四个、第五个。由“6”在第四个位置可知, 条件 (1) 中“6”位置不正确。再由第一位是“3”可知, 条件(1)中的“7”错误。综上, 条件 (1) 中错误的数字为“7”、“0”, 位置不正确的是“6”。因此, 开机密码为 341965 。故 (E) 项正确。", "task_type": {"major": ["试题"], "minor": ["考研", "选择题"]}, "domain": ["管理学"], "metadata": "year:2023, score:2", "answer_from": "human", "human_verified": false, "copyright": "暂无版权及作者信息"} +{"instruction": "研究表明, 鱼油中的不饱和脂肪酸能有效降低人体血脂水平并软化血管。因此, 鱼油通常被用来预防由高血脂引起的心脏病、动脉㤡样硬化和高胆固醇等疾病, 降低死亡风险。但有研究人员认为,食用鱼油不一定能够有效控制血脂水平并预防由高血脂引起的各种疾病。以下哪项如果为真,最能支持上述研究人员的观点?\n(A)鱼油虽然优于猪油、牛油,但毕竟是脂肪,如果长期食用,容易引起肥胖。\n(B)鱼油的概念很模糊, 它既指鱼体内的脂肪, 也包括被做成保健品的鱼油制剂。\n(C)不饱和脂肪酸很不稳定, 只要接触空气、阳光, 就会氧化分解。\n(D) 通过长期服用鱼油制品来控制体内血脂的观点始终存在学术争论。\n(E)���们若要身体健康最好注重膳食平衡, 而不仅仅依靠服用浓缩鱼油。", "input": "", "output": "专家: 鱼油中的不饱和脂肪酸能有效降低人体血脂水平并软化血管, 但是, 食用鱼油不一定能够有效控制血脂水平并预防由高血脂引起的各种疾病。“但是”前面的对象是“鱼油中的不饱和脂肪酸”, “但是”后面的对象是“食用鱼油”, 二者不一致,故此题为拆桥搭桥模型。A 项, 无关选项, 题干讨论的话题是鱼油对血脂的控制和预防, 而该项讨论的是肥胖, 二者并非同一概念 (干扰项·偷换概念) 。B 项,无关选项,题干的论证不涉及对“鱼油”这一概念的辨析。$\\mathrm{C}$ 项, 说明不饱和脂肪酸容易氧化分解, 因此鱼油中未必会含有不饱和脂肪酸, 故而不一定能够有效控制血脂水平并预防由高血脂引起的各种疾病。支持题干。D 项,无关选项,在学术上存在争论并不能说明鱼油是否有效果(干扰项·诉诸权威)。$\\mathrm{E}$ 项, 此项讨论的话题是“身体健康”, 题干要论的是“控制血脂水平并预防由高血脂引起的各种疾病”,两者并非同一概念,排除。\n\n因此答案为:C", "task_type": {"major": ["试题"], "minor": ["考研", "选择题"]}, "domain": ["管理学"], "metadata": "year:2023, score:2", "answer_from": "human", "human_verified": false, "copyright": "暂无版权及作者信息"} +{"instruction": "近年来, 一些地方修改了见义勇为的相关条例, 强调对生命的敬畏和尊重, 既肯定大义凛然、挺身而出的见义勇为, 更鼓励和倡导科学、合法、正当的“见义智为”。有专家由此指出, 从鼓励见义勇为到倡导“见义智为”, 反映了社会价值观念的进步。\n\n\n以下各项如果为真, 则除了哪项均能支持上述专家的观点?\n\n(A) “见义智为”强调以人为本、合理施救, 表明了科学理性、互帮互助的社会价值取向。\n\n(B) 有时见义勇为需要专业技术知识, 普通民众如果没有相应的知识, 最好不要贸然行事, 应及时报警求助。\n\n(C) 所有的生命都是平等的, 救人者与被救者都具有同等的生命价值, 救人者的生命同样应得到尊重和爱护。\n\n(D) 我国中小学正在引导学生树立应对突发危机事件的正确观念, 教育学生如何在保证自身安全的情况下 “机智”救助他人。\n\n(E)倡导 “见义智为” 容易给一些自私懦弱的人逃避社会责任制造借口, 见死不救的惨痛案例可能增多, 社会道德水平可能因此而下滑。", "input": "", "output": "专家指出:从鼓励见义勇为到倡导“见义智为”, 反映了社会价值观念的进步。此题论点前的内容, 阐述了当前社会中的现象, 但这些现象对论点的成立并未起到支撑的作用,故均为背景信息。故此题为无论据模型。对于此种类型的题,直接分析选项即可。$A 、 B 、 C 、 D$ 四项均说明“见义智为”的重要性和必要型,从“见义勇为”到“见义智为”确实反映了社会价值观念的进步。$E$ 项,该项说明“见义智为”可能会造见死不救等情形增多、社会的道德水平下降等负面影响;补充反面论据, 削弱题干。\n\n因此答案为:E", "task_type": {"major": ["试题"], "minor": ["考研", "选择题"]}, "domain": ["管理学"], "metadata": "year:2023, score:2", "answer_from": "human", "human_verified": false, "copyright": "暂无版权及作者信息"} +{"instruction": "近期一项调查数据显示: 中国不缺少外科医生, 而缺少能做手术的外科医生; 中国人均拥有的外科医生数量同其他中高收人国家相当,但中国人均拥有的外科医生所做的手术数量却比那些国家少 $40 \\%$ 。\n\n以下哪项如果为真, 最能解释上述现象?\n\n(A)年轻外科医生一般总要花费数年时间协助资深外科医生手术, 然后才有机会亲自主刀上阵,这已成为国内外医疗行业的惯例。\n\n(B)近年来, 我国能做手术的外科医生的人均手术量已与其他中高收人国家外科医生的人均手术量基本相当。\n\n(C) 患者在需要外科手术时, 都想请经验丰富的外科医生为其主刀, 不愿成为年轻医生的练习对象,对此医院一般都会有合理安排。\n\n(D) 资深外科医生经常收到手术邀请, 他们常年奔波在多所医院为年轻医生主刀示范, 培养了不少新人。\n\n(E) 从一名医学院学生成长为能做手术的外科医生, 需要经历漫长的学习过程, 有些人中途不得不放弃梦想而另谋职业。", "input": "", "output": "本题为解释题, 等解释的现象是: 中国不缺少外科医生, 而缺少能做手术的外科医生; 中国人均拥有的外科医生数量同其他中高收入国家相当, 但中国人均拥有的外科医生所做的手术数量却比那些国家少 $40 \\%$ 。题干中涉及数量关系, 故此题为数量关系模型。人均拥有的外科���生所做的手术数量 $=\frac{\text { 外科医生所做手术总数量 }}{\text { 总人口 }}$外科医生所做手术数总量 $=$ 能做外科手术的医生数量 $\times$ 这些医生的人均手术量。也就是说, 题干中的现象有两个影响因素: (1)“能做外科手术的医生数量”; (2)“这些医生的人均手术量”。而题干认为, 中国缺少能做手术的外科医生, 即只考虑到了因素(1), 因此, 要排除因素(2)的影响, 故此题选 B。其余各项均不涉及题干中的数量关系, 故均为无关选项。\n\n因此答案为:B", "task_type": {"major": ["试题"], "minor": ["考研", "选择题"]}, "domain": ["管理学"], "metadata": "year:2023, score:2", "answer_from": "human", "human_verified": false, "copyright": "暂无版权及作者信息"} +{"instruction": "某单位购买了《尚书》《周易》《诗经》《论语》《老子》《孟子》各 1 本, 分发给甲、乙、丙、丁、戊 5 个部门, 每个部门至少 1 本。已知:\n\n(1)若《周易》《老子》《孟子》至少有 1 本分发给甲或乙部门, 则《尚书》分发给丁部门且《论语》分发给戊部门。\n\n(2)若《诗经》《论语》至少有 1 本分发给甲或乙部门,则《周易》分发给丙部门且《老子》分发给戊部门。\n\n若《尚书》分发给丙部门, 则可以得出以下哪项?\n(A)《诗经》分发给甲部门。\n(B)《论语》分发给乙部门。\n(C)《老子》分发给丙部门。\n(D)《孟子》分发给丁部门。\n(E)《周易》分发给戊部门。", "input": "", "output": "本题中的已知条件由事实、假言命题以及匹配关系组成, 故此题为事实假言匹配模型。\n从事实出发, 由“《尚书》分发给丙部门”可知, 条件 (1) 后件为假, 根据口诀“否后必否前”可得: 《周易》、《老子》、《孟子》均未分发给甲、乙部门。因此, 《诗经》、《论语》分发给甲、乙部门(不一定一一对应)。\n故条件 (2) 前件为真, 根据口诀“肯前必肯后”, 可得: 《周易》分发给丙部门且《老子》分发给戊部门。\n再结合“每个部门至少 1 本”可知, 《孟子》分发给丁部门。故 (D) 项正确。", "task_type": {"major": ["试题"], "minor": ["考研", "选择题"]}, "domain": ["管理学"], "metadata": "year:2023, score:2", "answer_from": "human", "human_verified": false, "copyright": "暂无版权及作者信息"} +{"instruction": "某单位购买了《尚书》《周易》《诗经》《论语》《老子》《孟子》各 1 本, 分发给甲、乙、丙、丁、戊 5 个部门, 每个部门至少 1 本。已知:\n\n(1)若《周易》《老子》《孟子》至少有 1 本分发给甲或乙部门, 则《尚书》分发给丁部门且《论语》分发给戊部门。\n\n(2)若《诗经》《论语》至少有 1 本分发给甲或乙部门,则《周易》分发给丙部门且《老子》分发给戊部门。\n\n若《老子》分发给丁部门, 则以下哪项是不可能的?\n(A)《周易》分发给甲部门。\n(B)《周易》分发给乙部门。\n(C)《诗经》分发给丙部门。\n(D)《尚书》分发给丁部门。\n(E)《诗经》分发给戊部门。", "input": "", "output": "从事实出发, 由“《老子》分发给丁部门”可知, 条件 (2) 后件为假, 根据口诀否后必否前可得: 《诗经》、《论语》均不分发给甲和乙部门; 故, 《尚书》《周易》《孟子》中有两本分发给甲和乙; 因此, 条件 (1) 前件为真。根据口诀“肯前必肯后”, 可得: 《尚书》分发给丁部门且《论语》分发给戊部门。由于甲和乙都不能分配《诗经》, 结合“每个部门至少 1 本”可得, 《诗经》分配给丙部门。故 (E) 项正确。", "task_type": {"major": ["试题"], "minor": ["考研", "选择题"]}, "domain": ["管理学"], "metadata": "year:2023, score:2", "answer_from": "human", "human_verified": false, "copyright": "暂无版权及作者信息"} +{"instruction": "“嫦娥”登月、“神舟”巡天,我国不断谱写飞天梦想的新篇章。基于太空失重环境的多重效应,研究人员正在探究植物在微重力环境下生存的可能性。他们设想, 如果能够在太空中种植新鲜水果和蔬菜, 则不仅有利于航天员的身体健康, 而且还可以降低食物的上天成本, 同时, 可以利用其消耗的二氧化碳产生氧气, 为航天员生活与工作提供有氧环境。\n\n\n以下哪项如果为真, 则可能成为研究人员实现上述设想的最大难题?\n(A) 为了携带种子、土壤等种植必需品上天, 飞船需要减少其他载荷以满足发射要求, 这可能影响其他科学实验的安排。\n(B)有些航天员虽然在地面准备阶段学习掌握了植物栽培技术,但在太空的实际操作中他们可能会遇到意想不到的情况。\n(C)太空中的失重、宇宙射线等因素会对植物的生长和发育产生不良影响, 食用这些植物可能有损航天员的健康。\n(D)有些航天员将植��带人太空, 又成功带回地面, 短暂的太空经历对这些植物后来的生长发育可能造成影响。\n(E)过去很多航天器携带植物上天, 因为缺乏生长条件, 这些植物都没有存活很长时间。", "input": "", "output": "本题的问题是: “以下哪项如果为真, 则可能成为研究人员实现上述设想的最大难题”,本题实际上是找出一个选项说明设想最不可能成立, 故此题为削弱题。此题为措施目的模型。研究人员设想的措施:在太空中种植新鲜水果和蔬菜研究人员设想的目的: (1)有利于航天员的身体健康, (2)降低食物的上天成本, (3)航天员生活与工作提供有氧环境。A 项, 该项说明在太空中种植新鲜水果和蔬菜可能会影响其他科学实验的安排, 即有一定的副作用, 但这些副作用与研究人员设想的三个目的没有直接关系, 故该项削弱力度有限。B 项, “意想不到的情况” 是中性词, 即可能是好的情况也可能是坏的情㳘, 故此项排除 (干扰项·两可选项)。$\\mathrm{C}$ 项,食用这些植物可能有损航天员的健康,即措施达不到目的 (1), 可以削弱题干。$\\mathrm{D}$ 项, 无关选项, 题干论证不涉及将植物带入太空, 又成功带回地面后会对植物产生何种影响 (干扰项·转移话题)。$\\mathrm{E}$ 项, 无关选项, 题干讨论的是未来的情况, 而此项讨论的是过去的情况。\n\n因此答案为:C", "task_type": {"major": ["试题"], "minor": ["考研", "选择题"]}, "domain": ["管理学"], "metadata": "year:2023, score:2", "answer_from": "human", "human_verified": false, "copyright": "暂无版权及作者信息"} +{"instruction": "十多年前曾有传闻: $M$ 国从不生产一次性筷子,完全依赖进口,而且 $M$ 国 $96 \\%$ 的一次性筷子来自中国。2019 年有媒体报道: “去年 M 国出口的木材中,约有 $40 \\%$ 流向了中国市场,而且今年中国订单的比例还在进一步攀升, 中国已成为 $\\mathrm{M}$ 国木材出口中占比最大的国家。”张先生据此认为, 中国和 $\\mathrm{M}$ 国木材进出口角色的转换, 表明中国人的环保意识已经超越 $\\mathrm{M}$ 国。以下哪项如果为真, 最能削弱张先生的观点?\n\n(A)十多年前的传闻不一定反映真实情况, 实际情形是中国的一次性筷子比其他国家的更便宜。\n\n(B) 从 2018 年起, 中国相关行业快速发展, 木材需求急剧增长; 而 M 国多年养护的速生林正处于采伐期, 出口量逐年递增。\n\n(C)近年中国修订相关规范, 原来只用于商品外包装的 $\\mathrm{M}$ 国杉木现也可用于木结构建筑物, 导致进口大增。\n\n(D) 制作一次性筷子的木材主要取自速生杨树或者桦树, 这类速生树种只占中国经济林的极小部分。\n\n(E) 中国和 $\\mathrm{M}$ 国在木材贸易上的角色转换主要是经济发展导致, 环保意识只是因素之一,但不是主要因素。", "input": "", "output": "题干: 中国和 $\\mathrm{M}$ 国木材进出目角色的转换(现象),表明中国人的环保意识已经超越 $\\mathrm{M}$ 国(原因)。题干先是描述了一种现象, 然后分析这种现象的原因, 故此题是现象原因模型。题干中讨论的是 2018 年中国和 M 国的情况, 故可快速定位到 B 项, 该项说明, 中国需求量急剧增长, $\\mathrm{M}$ 国怱口量递增; 进而导致了角色的转变,另有他因,削弱题干。A 项,题干论证不涉及中国和其他国家一次性筷子价格的比较(干扰项·无关新比较)。$\\mathrm{C}$ 项, 近年”的情况,与题干时间不一致,排除。D项,无关选项,题干不涉及杨树或者桦树在中国经济林中的占比(干扰项·无关比例)。$\\mathrm{E}$ 项, 此项说明环保意识虽不是木材进出口角色转换的主要原因, 但也确实是原因之一, 支持题干 (干扰项·明否暗肯) 。\n\n因此答案为:B", "task_type": {"major": ["试题"], "minor": ["考研", "选择题"]}, "domain": ["管理学"], "metadata": "year:2023, score:2", "answer_from": "human", "human_verified": false, "copyright": "暂无版权及作者信息"} +{"instruction": "某公司为了让员工多运动, 近日出台一项规定: 每月按照 18 万步的标准对员工进行考核, 如果没有完成步行任务, 则按照“一步一分钱”的标准扣钱。有专家认为, 此举鼓励运动, 看似对员工施加压力, 实质上能够促进员工的身心健康, 引导整个企业积极向上。\n\n以下各项如果为真, 则除哪项外均能质疑上述专家的观点?\n(A)按照我国《劳动法》等相关法律规定, 企业规章制度所涉及的员工行为应与工作有关, 而步行显然与工作无关。\n(B)步行有益身体健康, 但规定每月必须步行 18 万步, 不达标就扣钱, 显得有些简单粗暴, 这会影响员工对企业的认同感。\n(C)公司鼓励员工多运动, 此举不仅让员工锻炼身体, 还可释放工作压力, 培��良好品格, 改善人际关系。\n(D)有员工深受该规定的困扰, 为了完成考核, 他们甚至很晚不得不外出运动, 影响了正常休息。\n(E)该公司老张在网上购买了专门刷步行数据的服务, 只花 1 元钱就可轻松购得两万步。", "input": "", "output": "注意, 本题的问题是: “以下各项如果为真, 则除哪项外均能质疑上述专家的观点”故此题为削弱题,但找的是不能质疑的项。题干: 此举鼓励运动, 看似对员工施加压力, 实质上能够促进员工的身心健康, 引导整个企业积极向上。题干中做出了一个对结果的断定, 故此题为预测结果模型。此模型可直接分析选项。$\\mathrm{A}$ 项, 削弱题干, 此项说明公司的该项规定与工作无关, 是一种违法行为。B 项,削弱题干,此项说明此举虽有益身体健康,但会影响员工对企业的认同感。$C$ 项, 支持题干, 此项说明该规定有助于缓解压力、改善人际关系, 即: 有益于身心健康,引导整个企业积极向上。D 项,削弱题干,此项说明此举使员工受到困扰,影响员工的正常休息。$E$ 项, 削弱题干, 例证法, 说明公司的规定会使得员工采用不正当手段, 那就不能引导整个企业积极向上了。\n\n因此答案为:C", "task_type": {"major": ["试题"], "minor": ["考研", "选择题"]}, "domain": ["管理学"], "metadata": "year:2023, score:2", "answer_from": "human", "human_verified": false, "copyright": "暂无版权及作者信息"} +{"instruction": "通过第三方招聘进人甲公司从事销售工作的职员均具有会计学专业背景。孔某的高中同学均没有会计学专业背景。甲公司销售部经理孟某是孔某的高中同学, 而孔某是通过第三方招聘进人甲公司的。\n\n根据以上信息, 可以得出以下哪项?\n(A)孔某具有会计学专业背景。\n(B)孟某不是通过第三方招聘进人甲公司的。\n(C)孟某曾经自学了会计学专业知识。\n(D)孔某在甲公司做销售工作。\n(E)孔某和孟某在大学阶段不是同学。", "input": "", "output": "已知条件由事实和假言命题(性质命题也可视为假言)组成, 故此题为事实假言模型。题干信息如下:(1) 第三方招聘进入甲公司从事销售工作的职员 $\rightarrow$ 会计学专业背景。(2) 孔某的高中同学 $\rightarrow$ 没有会计学专业背景。(3) 甲公司销售部经理孟某是孔某的高中同学。从事实出发, 由 (3) 可知, (2) 的前件为真, 根据口诀“肯前必肯后”, 可得: 孟某没有会计学专业背景。进而可知, (1) 后件为假, 根据口诀“否后必否前”, 可得: 孟某不是第三方招聘进入甲公司从事销售工作的职员。即:孟某不是通过第三方招聘进入甲公司 $\\vee$ 孟某不从事销售。由于孟某是销售经理, 即从事销售。故可知: 孟某不是通过第三方招聘进入甲公司。故 (B) 项正确。", "task_type": {"major": ["试题"], "minor": ["考研", "选择题"]}, "domain": ["管理学"], "metadata": "year:2023, score:2", "answer_from": "human", "human_verified": false, "copyright": "暂无版权及作者信息"} +{"instruction": "人冬以来, 天气渐渐寒冷。11月 30 日, 某地气象台对未来 5 天的天气预报显示: 未来 5 天每天的最高气温从 $4^{\\circ} \\mathrm{C}$ 开始逐日下降至 $-1^{\\circ} \\mathrm{C}$; 每天的最低气温不低于 $-6^{\\circ} \\mathrm{C}$; 最低气温 $-6^{\\circ} \\mathrm{C}$ 只出现在其中一天。预报还包含如下信息:\n\n(1)未来 5 天中的最高气温和最低气温不会出现在同一天, 每天的最高气温和最低气温均为整数。\n\n(2)若 5 号的最低气温是未来 5 天中最低的, 则 2 号的最低气温比 4 号的高 $4^{\\circ} \\mathrm{C}$ 。\n\n(3) 2 号和 4 号每天的最高气温与最低气温之差均为 $5^{\\circ} \\mathrm{C}$ 。\n\n根据以上预报信息, 可以得出以下哪项?\n(A) 1 号的最低气温比 2 号的高 $2^{\\circ} \\mathrm{C}$ 。\n(B) 3 号的最高气温比 4 号的高 $1^{\\circ} \\mathrm{C}$ 。\n(C) 4 号的最高气温比 5 号的高 $1^{\\circ} \\mathrm{C}$ 。\n(D) 3 号的最低气温为 $-6^{\\circ} \\mathrm{C}$ 。\n(E) 2 号的最低气温为 $-3^{\\circ} \\mathrm{C}$ 。", "input": "", "output": "题干由事实和假言命题组成, 故为事实假言模型。由“未来 5 天每天的最高气温从 $4^{\\circ} \\mathrm{C}$ 开始逐日下降至 $-1^{\\circ} \\mathrm{C}$ ”可知, 1 号的最高气温为 $4^{\\circ} \\mathrm{C} 、 5$ 号的最高气温为 $-1^{\\circ} \\mathrm{C}$ 。因此, 2 号和 4 号的最高气温均不是 $-1^{\\circ} \\mathrm{C}$, 且均在 $0^{\\circ} \\mathrm{C} \\sim 3^{\\circ} \\mathrm{C}$ 之间;由“2 号和 4 号的最高气温均不是 $-1^{\\circ} \\mathrm{C}$ ”结合条件 (3) 可知, 2 号和 4 号的最低气温均不是 $-6^{\\circ} \\mathrm{C}$ 。由“2 号和 4 号的最高气温均均在 $0^{\\circ} \\mathrm{C} \\sim 3^{\\circ} \\mathrm{C}$ 之间”可知, 条件 (2) 后件为假, 根据口诀“否后必否前”, 可得: 5 号的最低气温不是未来 5 天中最低的。再由条件 (1) 中“未来 5 天中的最高气温和最低气温不会出现在同一天”可知, 1 号的最低气温也不是 $-6^{\\circ} \\mathrm{C}$ 。综上, 1 号、 2 号、4 号、 5 号的最低气温均不是 $-6^{\\circ} \\mathrm{C}$; 因此, 3 号的最低气温为 $-6^{\\circ} \\mathrm{C}$ 。故 (D) 项正确。", "task_type": {"major": ["试题"], "minor": ["考研", "选择题"]}, "domain": ["管理学"], "metadata": "year:2023, score:2", "answer_from": "human", "human_verified": false, "copyright": "暂无版权及作者信息"} +{"instruction": "甲: 张某爱出风头, 我不喜欢他。\n\n乙: 你不喜欢他没关系, 他工作一直很努力, 成绩很突出。\n\n以下哪项与上述反驳方式最为相似?\n\n(A)甲: 李某爱慕虚荣, 我很反对。\n\n乙: 反对有一定道理,但你也应该体谅一下他,他身边的朋友都是成功人士。\n\n(B)甲:贾某整天学习, 寡言少语, 神情严肃, 我很担心他。\n\n乙: 你的担心是多余的。他最近在潜心准备考研, 有些紧张是正常的。\n\n(C) 甲:韩某爱管闲事,我有点讨厌他。\n\n乙: 你的态度有问题, 爱管闲事说明他关心别人, 乐于助人。\n\n(D)甲:钟某爱看足球赛, 但自己从来不踢足球, 对此我很不理解。\n\n乙: 我对你的想法也不理解, 欣赏和参与是两回事啊。\n\n(E)甲:邓某爱读书但不求甚解,对此我很有看法。\n\n乙: 你有看法没用。他的文学素养挺高, 已经发表了 3 篇小说。", "input": "", "output": "此题的问题为“以下哪项与上述反驳方式最为相似”, 故此题为结构相似题。甲: 张某爱出风头, 我不喜欢他。乙: 你不喜欢他没关系,他工作一直很努力, 成绩很突出。乙提出了一个反面论据, 指出张某在其他方面有优点。A 项, 由“反对有一定道理”可知, 乙在一定程度上支持甲的观点, 与题干不相似。B 项, 乙阐述了贾某紧张的原因, 并未指出贾某在其他方面有的优点, 与题干不相似。C 项, 乙指出的是甲的态度有问题, 并未指出韩某在其他方面的优点, 与题干不相似。D 项, 乙提出甲犯了偷换概念的逻辑错误, 与题干不相似。$\\mathrm{E}$ 项, 乙反驳甲的观点, 并指出邓某在文学素养方面的优点; 与题干反驳方式一致。\n\n因此答案为:E", "task_type": {"major": ["试题"], "minor": ["考研", "选择题"]}, "domain": ["管理学"], "metadata": "year:2023, score:2", "answer_from": "human", "human_verified": false, "copyright": "暂无版权及作者信息"} +{"instruction": "某机关甲、乙、丙、丁 4 人参加本年度综合考评。在德、能、勤、绩、廉 5 个方面的单项考评中,他们之中都恰有 3 人被评为“优秀”,但没有人 5 个单项均被评为“优秀”。已知:\n\n(1)若甲和乙在德方面均被评为“优秀”,则他们在廉方面也均被评为“优秀”。\n\n(2)若乙和丙在德方面均被评为“优秀”,则他们在绩方面也均被评为“优秀”。\n\n(3)若甲在廉方面被评为“优秀”,则甲和丁在绩方面均被评为“优秀”。\n根据上述信息, 可以得出以下哪项?\n(A)甲在廉方面被评为 “优秀”。\n(B)丙在绩方面被评为“优秀”。\n(C)丙在能方面被评为 “优秀”。\n(D) 丁在勤方面被评为 “优秀”。\n(E)丁在德方面被评为“优秀”。\n\n 根据上述信息, 可以得出以下哪项?", "input": "", "output": "本题中的已知条件由数量关系、假言命题组成, 故此题为数量假言模型。由“他们之中都恰有 3 人被评为优秀”可知,“绩”由 3 人被评委优秀; 因此,条件(2)、的后件不可能同时为真。若 (2) 的后件为假, 根据口诀“否后必否前”可得:则乙、丙至少有一人在德方面未被评为“优秀”;若(3)的后件为假, 根据口诀“否后必否前”可得:甲在廉方面未被评为优秀; 再由“甲在廉方面未被评为优秀”可知, 条件 (1) 后件为假, 根据口诀“否后必否前”可得: 甲、乙至少有一人在德方面未被评为“优秀”。综上, 甲、乙、丙三人中至少有一人未在德方面被评为优秀; 再结合“他们之中都恰有 3 人被评为优秀”可知, 丁在德方面被评为优秀。故 (E) 项正确。", "task_type": {"major": ["试题"], "minor": ["考研", "选择题"]}, "domain": ["管理学"], "metadata": "year:2023, score:2", "answer_from": "human", "human_verified": false, "copyright": "暂无版权及作者信息"} +{"instruction": "某机关甲、乙、丙、丁 4 人参加本年度综合考评。在德、能、勤、绩、廉 5 个方面的单项考评中,他们之中都恰有 3 人被评为“优秀”,但没有人 5 个单项均被评为“优秀”。已知:\n\n(1)若甲和乙在德方面均被评为“优秀”,则他们在廉方面也均被评为“优秀��。\n\n(2)若乙和丙在德方面均被评为“优秀”,则他们在绩方面也均被评为“优秀”。\n\n(3)若甲在廉方面被评为“优秀”,则甲和丁在绩方面均被评为“优秀”。\n若甲在绩方面未被评为“优秀”且丁在能方面未被评为“优秀”,则可以得出以下哪项?\n(A) 甲在勤方面未被评为“优秀”。\n(B) 甲在能方面未被评为“优秀”。\n(C)乙在德方面未被评为“优秀”。\n(D)丙在廉方面未被评为“优秀”。\n(E)丁在廉方面未被评为 “优秀", "input": "", "output": "本题补充了事实, 故此题为事实数量假言模型。由“他们之中都恰有 3 人被评为优秀”即共有 $5 \times 3=15$ 个优秀。“没有人 5 个单项均被评为优秀”可知,故每人至多有 4 个优秀。故四人对应的优秀个数为 $4 、 4 、 4 、 3$ 。本题新补充事实:(5)“甲在绩方面未被评为优秀”且“丁在能方面未被评为优秀”。从事实出发, 由“甲在绩方面未被评为优秀”可知, 条件 (3) 后件为假, 根据口诀“否后必否前”可得:甲在廉方面未被评为优秀。故甲在绩和廉均不优秀, 故甲优秀个数为 3 个, 可得:甲在德、能、勤方面获得优秀。再由“甲在廉方面未被评为优秀”可知, 条件 (1) 后件为假, 根据口诀“否后必否前”可得: 甲、乙至少有一人在德方面未被评为“优秀”。再由“甲在德方面优秀”可知,乙在德方面未被评为“优秀”。故 (C) 项正确。", "task_type": {"major": ["试题"], "minor": ["考研", "选择题"]}, "domain": ["管理学"], "metadata": "year:2023, score:2", "answer_from": "human", "human_verified": false, "copyright": "暂无版权及作者信息"} +{"instruction": "(1) 函数 $f(x)=\\left\\{\\begin{array}{c}\\frac{e^{x}-1}{x}, x \\neq 0 \\\\ 1, x=0\\end{array}\\right.$, 在 $x=0$ 处\n(A)连续且取极大值.\n(B)连续且取极小值.\n(C) 可导且导数为 0 .\n(D)可导且导数不为 0 .", "input": "", "output": "因为 $\\lim _{x \\rightarrow 0} f(x)=\\lim _{x \\rightarrow 0} \\frac{e^{x}-1}{x}=1=f(0)$, 故 $f(x)$ 在 $x=0$ 处连续;\n因为 $\\lim _{x \\rightarrow 0} \\frac{f(x)-f(0)}{x-0}=\\lim _{x \\rightarrow 0} \\frac{\\frac{e^{x}-1}{x}-1}{x-0}=\\lim _{x \\rightarrow 0} \\frac{e^{x}-1-x}{x^{2}}=\\frac{1}{2}$, 故 $f^{\\prime}(0)=\\frac{1}{2}$, 正确答案为 D.", "task_type": {"major": ["试题"], "minor": ["考研", "选择题"]}, "domain": ["数学"], "metadata": "year:2021, score:5", "answer_from": "human", "human_verified": false, "copyright": "暂无版权及作者信息"} +{"instruction": "设函数 $f(x, y)$ 可微, 且 $f\\left(x+1, e^{x}\\right)=x(x+1)^{2}, f\\left(x, x^{2}\\right)=2 x^{2} \\ln x$, 则 $d f(1,1)=$\n(A) $d x+d y$.\n(B) $d x-d y$.\n(C) $d y$.\n(D) $-d y$.", "input": "", "output": "$f_{1}^{\\prime}\\left(x+1, e^{x}\\right)+e^{x} f_{2}^{\\prime}\\left(x+1, e^{x}\\right)=(x+1)^{2}+2 x(x+1)$\n$f_{1}^{\\prime}\\left(x, x^{2}\\right)+2 x f_{2}^{\\prime}\\left(x, x^{2}\\right)=4 x \\ln x+2 x$\n分别将 $\\left\\{\\begin{array}{l}x=0 \\\\ y=0\\end{array},\\left\\{\\begin{array}{l}x=1 \\\\ y=1\\end{array}\\right.\\right.$ 带入(1)(2)式有\n$f_{1}^{\\prime}(1,1)+f_{2}^{\\prime}(1,1)=1, \\quad f_{1}^{\\prime}(1,1)+2 f_{2}^{\\prime}(1,1)=2$\n\n联立可得 $f_{1}^{\\prime}(1,1)=0, f_{2}^{\\prime}(1,1)=1, d f(1,1)=f_{1}^{\\prime}(1,1) d x+f_{2}^{\\prime}(1,1) d y=d y$, 故正确答案为 C.", "task_type": {"major": ["试题"], "minor": ["考研", "选择题"]}, "domain": ["数学"], "metadata": "year:2021, score:5", "answer_from": "human", "human_verified": false, "copyright": "暂无版权及作者信息"} +{"instruction": "设函数 $f(x)=\\frac{\\sin x}{1+x^{2}}$ 在 $x=0$ 处的 3 次泰勒多项式为 $a x+b x^{2}+c x^{3}$, 则\n(A) $a=1, b=0, c=-\\frac{7}{6}$.\n(B) $a=1, b=0, c=\\frac{7}{6}$.\n(C) $a=-1, b=-1, c=-\\frac{7}{6}$.\n(D) $a=-1, b=-1, c=\\frac{7}{6}$.", "input": "", "output": "根据麦克劳林公式有\n$f(x)=\frac{\\sin x}{1+x^2}=\\left[x-\frac{x^3}{6}+o\\left(x^3\right)\right] \\cdot\\left[1-x^2+o\\left(x^3\right)\right]=x-\frac{7}{6} x^3+o\\left(x^3\right)$\n故 $a=1, b=0, c=-\\frac{7}{6}$, 本题选 A.", "task_type": {"major": ["试题"], "minor": ["考研", "选择题"]}, "domain": ["数学"], "metadata": "year:2021, score:5", "answer_from": "human", "human_verified": false, "copyright": "暂无版权及作者信息"} +{"instruction": "设函数 $f(x)$ 在区间 $[0,1]$ 上连续, 则 $\\int_{0}^{1} f(x) d x=$\n(A) $\\lim _{n \\rightarrow \\infty} \\sum_{k=1}^{n} f\\left(\\frac{2 k-1}{2 n}\\right) \\frac{1}{2 n}$.\n(B) $\\lim _{n \\rightarrow \\infty} \\sum_{k=1}^{n} f\\left(\\frac{2 k-1}{2 n}\\right) \\frac{1}{n}$.\n(C) $\\lim _{n \\rightarrow \\infty} \\sum_{k=1}^{2 n} f\\left(\\frac{k-1}{2 n}\\right) \\frac{1}{n}$.\n(D) $\\lim _{x \\rightarrow 0} \\sum_{k=1}^{2 n} f\\left(\\frac{k}{2 n}\\right) \\cdot \\frac{2}{n}$.", "input": "", "output": "由定积分的定义知, 将 $(0,1)$ 分成 $n$ 份, 取中间点的函数值, 则 $\\int_{0}^{1} f(x) d x=\\lim _{n \\rightarrow \\infty} \\sum_{k=1}^{n} f\\left(\\frac{2 k-1}{2 n}\\right) \\frac{1}{n}$, 即选 B.", "task_type": {"major": ["试题"], "minor": ["考研", "选择题"]}, "domain": ["数学"], "metadata": "year:2021, score:5", "answer_from": "human", "human_verified": false, "copyright": "暂无版权及作者信息"} +{"instruction": "二次型 $f\\left(x_{1}, x_{2}, x_{3}\\right)=\\left(x_{1}+x_{2}\\right)^{2}+\\left(x_{2}+x_{3}\\right)^{2}-\\left(x_{3}-x_{1}\\right)^{2}$ 的正惯性指数与负惯性指数依次为\n(A) 2,0 .\n(B) 1,1 .\n(C) 2,1 .\n(D) 1,2 .", "input": "", "output": "$f\\left(x_1, x_2, x_3\\right)=\\left(x_1+x_2\\right)^2+\\left(x_2+x_3\\right)^2-\\left(x_3-x_1\\right)^2=2 x_2{ }^2+2 x_1 x_2+2 x_2 x_3+2 x_1 x_3$所以 $A=\\left(\\begin{array}{lll}0 & 1 & 1 \\\\ 1 & 2 & 1 \\\\ 1 & 1 & 0\\end{array}\\right)$, 故特征多项式为\n$$\n|\\lambda E-A|=\\left|\\begin{array}{ccc}\n\\lambda & -1 & -1 \\\\\n-1 & -2 & -1 \\\\\n-1 & -1 & \\lambda\n\\end{array}\\right|=(\\lambda+1)(\\lambda-3) \\lambda\n$$\n\n令上式等于零, 故特征值为 $-1,3,0$, 故该二次型的正惯性指数为 1 , 负惯性指数为 1 . 故应选 B.", "task_type": {"major": ["试题"], "minor": ["考研", "选择题"]}, "domain": ["数学"], "metadata": "year:2021, score:5", "answer_from": "human", "human_verified": false, "copyright": "暂无版权及作者信息"} +{"instruction": "已知 $\\alpha_{1}=\\left(\\begin{array}{l}1 \\\\ 0 \\\\ 1\\end{array}\\right), \\alpha_{2}=\\left(\\begin{array}{l}1 \\\\ 2 \\\\ 1\\end{array}\\right), \\alpha_{3}=\\left(\\begin{array}{l}3 \\\\ 1 \\\\ 2\\end{array}\\right)$, 记 $\\beta_{1}=\\alpha_{1}, \\beta_{2}=\\alpha_{2}-k \\beta_{1}, \\beta_{3}=\\alpha_{3}-l_{1} \\beta_{1}-l_{2} \\beta_{2}$,若 $\\beta_{1}, \\beta_{2}, \\beta_{3}$ 两两正交, 则 $l_{1}, l_{2}$ 依次为\n(A) $\\frac{5}{2}, \\frac{1}{2}$.\n(B) $-\\frac{5}{2}, \\frac{1}{2}$.\n(C) $\\frac{5}{2},-\\frac{1}{2}$.\n(D) $-\\frac{5}{2},-\\frac{1}{2}$.", "input": "", "output": "$$\n\\begin{gathered}\n\\beta_2=\\alpha_2-\\frac{\\left[\\alpha_2, \\beta_1\\right]}{\\left[\\beta_1, \\beta_1\\right]} \\beta_1=\\left(\\begin{array}{l}\n0 \\\\\n2 \\\\\n0\n\\end{array}\\right), \\\\\n\\beta_3=\\alpha_3-\\frac{\\left[\\alpha_3, \\beta_1\\right]}{\\left[\\beta_1, \\beta_1\\right]} \\beta_1-\\frac{\\left[\\alpha_3, \\beta_2\\right]}{\\left[\\beta_2, \\beta_2\\right]} \\beta_2,\n\\end{gathered}\n$$\n\n故 $l_1=\\frac{\\left[\\alpha_3, \\beta_1\\right]}{\\left[\\beta_1, \\beta_1\\right]}=\\frac{5}{2}, l_2=\\frac{\\left[\\alpha_3, \\beta_2\\right]}{\\left[\\beta_2, \\beta_2\\right]}=\\frac{1}{2}$, 故选 A.", "task_type": {"major": ["试题"], "minor": ["考研", "选择题"]}, "domain": ["数学"], "metadata": "year:2021, score:5", "answer_from": "human", "human_verified": false, "copyright": "暂无版权及作者信息"} +{"instruction": "设 $A, B$ 为 $n$ 阶实矩阵, 下列不成立的是\n(A) $r\\left(\\begin{array}{cc}A & O \\\\ O & A^{T} A\\end{array}\\right)=2 r(A)$\n(B) $r\\left(\\begin{array}{ll}A & A B \\\\ O & A^{T}\\end{array}\\right)=2 r(A)$\n(C) $r\\left(\\begin{array}{cc}A & B A \\\\ O & A A^{T}\\end{array}\\right)=2 r(A)$\n(D) $r\\left(\\begin{array}{cc}A & O \\\\ B A & A^{T}\\end{array}\\right)=2 r(A)$", "input": "", "output": "(A) $r\\left(\\begin{array}{cc}A & O \\\\ O & A^{T} A\\end{array}\\right)=r(A)+r\\left(A^{T} A\\right)=2 r(A)$. 故 $A$ 正确.\n(B) $A B$ 的列向量可由 $A$ 的列线性表示, 故 $r\\left(\\begin{array}{ll}A & A B \\\\ O & A^{T}\\end{array}\\right)=r\\left(\\begin{array}{ll}A & O \\\\ 0 & A^{T}\\end{array}\\right)=r(A)+r\\left(A^{T}\\right)=2 r(A)$.\n(C) $B A$ 的列向量不一定能由 $A$ 的列线性表示.\n(D) $B A$ 的行向量可由 $A$ 的行线性表示, $r\\left(\\begin{array}{ll}A & B A \\\\ O & A^{T}\\end{array}\\right)=r\\left(\\begin{array}{cc}A & O \\\\ 0 & A^{T}\\end{array}\\right)=r(A)+r\\left(A^{T}\\right)=2 r(A)$.\n本题选 C.", "task_type": {"major": ["试题"], "minor": ["考研", "选择题"]}, "domain": ["数学"], "metadata": "year:2021, score:5", "answer_from": "human", "human_verified": false, "copyright": "暂无版权及作者信息"} +{"instruction": "设 $A, B$ 为随机事件, 且 $0P(A)$, 则 $P(\\bar{A} \\mid \\bar{B})>P(\\bar{A})$\n(C) 若 $P(A \\mid B)>P(A \\mid \\bar{B})$, 则 $P(A \\mid B)>P(A)$.\n(D) 若 $P(A \\mid A \\cup B)>P(\\bar{A} \\mid A \\cup B)$, 则 $P(A)>P(B)$.", "input": "", "output": "$P(A \\mid A \\cup B)=\\frac{P(A(A \\cup B))}{P(A \\bigcup B)}=\\frac{P(A)}{P(A)+P(B)-P(A B)}$\n$P(\\bar{A} \\mid A \\cup B)=\\frac{P(\\bar{A}(A \\cup B))}{P(A \\bigcup B)}=\\frac{P(\\bar{A} B)}{P(A \\bigcup B)}=\\frac{P(B)-P(A B)}{P(A)+P(B)-P(A B)}$\n因为 $P(A \\mid A \\cup B)>P(\\bar{A} \\mid A \\cup B)$, 固有 $P(A)>P(B)-P(A B)$, 故正确答案为 D.", "task_type": {"major": ["试题"], "minor": ["考研", "选择题"]}, "domain": ["数学"], "metadata": "year:2021, score:5", "answer_from": "human", "human_verified": false, "copyright": "暂无版权及作者信息"} +{"instruction": "设 $\\left(X_{1}, Y_{1}\\right),\\left(X_{2}, Y_{2}\\right), \\cdots,\\left(X_{n}, Y_{n}\\right)$ 为来自总体 $N\\left(\\mu_{1}, \\mu_{2} ; \\sigma_{1}^{2}, \\sigma_{2}^{2} ; \\rho\\right)$ 的简单随机样本, 令 $\\theta=\\mu_{1}-\\mu_{2}, \\bar{X}=\\frac{1}{n} \\sum_{i=1}^{n} X_{i}, \\bar{Y}=\\frac{1}{n} \\sum_{i=1}^{n} Y_{i}, \\hat{\\theta}=\\bar{X}-\\bar{Y}$, 则\n(A) $\\hat{\\theta}$ 是 $\\theta$ 的无偏估计, $D(\\hat{\\theta})=\\frac{\\sigma_{1}^{2}+\\sigma_{2}^{2}}{n}$\n(B) $\\hat{\\theta}$ 不是 $\\theta$ 的无偏估计, $D(\\hat{\\theta})=\\frac{\\sigma_{1}^{2}+\\sigma_{2}^{2}}{n}$\n(C) $\\hat{\\theta}$ 是 $\\theta$ 的无偏估计, $D(\\hat{\\theta})=\\frac{\\sigma_{1}^{2}+\\sigma_{2}^{2}-2 \\rho \\sigma_{1} \\sigma_{2}}{n}$\n(D) $\\hat{\\theta}$ 不是 $\\theta$ 的无偏估计, $D(\\hat{\\theta})=\\frac{\\sigma_{1}^{2}+\\sigma_{2}^{2}-2 \\rho \\sigma_{1} \\sigma_{2}}{n}$", "input": "", "output": "因为 $X, Y$ 是二维正态分布, 所以 $\\bar{X}$ 与 $\\bar{Y}$ 也服从二维正态分布, 则 $\\bar{X}-\\bar{Y}$ 也服从二维正态分布, 即 $E(\\hat{\\theta})=E(\\bar{X}-\\bar{Y})=E(\\bar{X})-E(\\bar{Y})=\\mu_{1}-\\mu_{2}=\\theta$,\n$D(\\hat{\\theta})=D(\\bar{X}-\\bar{Y})=D(\\bar{X})+D(\\bar{Y})-\\operatorname{cov}(\\bar{X}, \\bar{Y})=\\frac{\\sigma_{1}^{2}+\\sigma_{2}^{2}-2 \\rho \\sigma_{1} \\sigma_{2}}{n}$, 故正确答案为C $\\mathrm{C}$.", "task_type": {"major": ["试题"], "minor": ["考研", "选择题"]}, "domain": ["数学"], "metadata": "year:2021, score:5", "answer_from": "human", "human_verified": false, "copyright": "暂无版权及作者信息"} +{"instruction": "设 $X_{1}, X_{2} \\ldots, X_{16}$ 是来自总体 $N(\\mu, 4)$ 的简单随机样本, 考虑假设检验问题: $H_{0}: \\mu \\leq 10, H_{1}: \\mu>10 . \\Phi(x)$ 表示标准正态分布函数, 若该检验问题的拒绝域为 $W=\\{\\bar{X} \\geq 11\\}$,其中 $\\bar{X}=\\frac{1}{16} \\sum_{i=1}^{16} X_{i}$, 则 $\\mu=11.5$ 时, 该检验犯第二类错误的概率为\n(A) $1-\\Phi(0.5)$\n(B) $1-\\Phi(1)$\n(C) $1-\\Phi(1.5)$\n(D) $1-\\Phi(2)$", "input": "", "output": "所求概率为 $P\\{\\bar{X}<11\\} \\quad \\bar{X} \\sim N\\left(11.5, \\frac{1}{4}\\right)$,\n$$\nP\\{\\bar{X}<11\\}=P\\left\\{\\frac{\\bar{X}-11.5}{\\frac{1}{2}} \\leq \\frac{11-11.5}{\\frac{1}{2}}\\right\\}=1-\\Phi(1)\n$$\n\n故本题选 B.", "task_type": {"major": ["试题"], "minor": ["考研", "选择题"]}, "domain": ["数学"], "metadata": "year:2021, score:5", "answer_from": "human", "human_verified": false, "copyright": "暂无版权及作者信息"} +{"instruction": "设 $\\lim _{x \\rightarrow 1} \\frac{f(x)}{\\ln x}=1$, 则 $(\\quad)$.\nA. $f(1)=0$\nB. $\\lim _{x \\rightarrow 1} f(x)=0$\nC. $f^{\\prime}(1)=1$\nD. $\\lim _{x \\rightarrow 1} f^{\\prime}(x)=1$", "input": "", "output": "由于 $\\lim _{x \\rightarrow 1} \\frac{f(x)}{\\ln x}=1$, 所以 $\\lim _{x \\rightarrow 1} f(x)=0$. 故选 B.", "task_type": {"major": ["试题"], "minor": ["考研", "选择题"]}, "domain": ["数学"], "metadata": "year:2022, score:5", "answer_from": "human", "human_verified": false, "copyright": "暂无版权及作者信息"} +{"instruction": "设 $f(u)$ 可导, $z=x y f\\left(\\frac{y}{x}\\right)$, 若 $x \\frac{\\partial z}{\\partial x}+y \\frac{\\partial z}{\\partial y}=x y(\\ln y-\\ln x)$, 则 $(\\quad)$\nA. $f(1)=\\frac{1}{2}, f^{\\prime}(1)=0$\nB. $f(1)=0, f^{\\prime}(1)=\\frac{1}{2}$\nC. $f(1)=1, f^{\\prime}(1)=0$\nD. $f(1)=0, f^{\\prime}(1)=\\frac{1}{2}$", "input": "", "output": "$\\frac{\\partial z}{\\partial x}=y\\left[f\\left(\\frac{y}{x}\\right)+x f^{\\prime}\\left(\\frac{y}{x}\\right) \\cdot-\\frac{y}{x^{2}}\\right], \\frac{\\partial z}{\\partial y}=x\\left[f\\left(\\frac{y}{x}\\right)+y f^{\\prime}\\left(\\frac{y}{x}\\right) \\cdot \\frac{1}{x}\\right]$,\n则 $x \\frac{\\partial z}{\\partial x}+y \\frac{\\partial z}{\\partial y}=2 x y f\\left(\\frac{y}{x}\\right)=x y \\ln \\left(\\frac{y}{x}\\right)$. 因此 $2 f\\left(\\frac{y}{x}\\right)=\\ln \\left(\\frac{y}{x}\\right)$, 即 $f(u)=\\frac{1}{2} \\ln u$.\n故 $f(1)=0, f^{\\prime}(1)=\\frac{1}{2}$.\n\n因此答案为:D", "task_type": {"major": ["试题"], "minor": ["考研", "选择题"]}, "domain": ["数学"], "metadata": "year:2022, score:5", "answer_from": "human", "human_verified": false, "copyright": "暂无版权及作者信息"} +{"instruction": "下列是 $A_{3 \\times 3}$ 可对角化的充分而非必要条件是 ( )\nA. A 有 3 个不同特征值\nB. A 有 3 个无关的特征向量\nC. A 有 3 个两两无关的特征向量\nD. A 不同特征值对应的特征向量正交", "input": "", "output": "$\\boldsymbol{A}$ 有 3 个不同的特征值, 则 $\\boldsymbol{A}$ 有 3 个线性无关的特征向量, 此时 $\\boldsymbol{A}$ 可对角化, 由于矩阵可对角化的充要条件是线性无关特征向量个数等于矩阵阶数, 因此选项(A)符合题意", "task_type": {"major": ["试题"], "minor": ["考研", "选择题"]}, "domain": ["数学"], "metadata": "year:2022, score:5", "answer_from": "human", "human_verified": false, "copyright": "暂无版权及作者信息"} +{"instruction": "设 $\\alpha_{1}=\\left(\\begin{array}{l}\\lambda \\\\ 1 \\\\ 1\\end{array}\\right), \\alpha_{2}=\\left(\\begin{array}{l}1 \\\\ \\lambda \\\\ 1\\end{array}\\right), \\alpha_{3}=\\left(\\begin{array}{l}1 \\\\ 1 \\\\ \\lambda\\end{array}\\right), \\alpha_{4}=\\left(\\begin{array}{l}1 \\\\ \\lambda \\\\ \\lambda^{2}\\end{array}\\right)$, 若 $\\alpha_{1}, \\alpha_{2}, \\alpha_{3}$ 与 $\\alpha_{1}, \\alpha_{2}, \\alpha_{4}$ 等价, 则 $\\lambda \\in(\\quad)$.\nA. $\\{\\lambda \\mid \\lambda \\in \\mathbb{R}\\}$\nB. $\\{\\lambda \\mid \\lambda \\in \\mathbb{R}, \\lambda \\neq-1\\}$\nC. $\\{\\lambda \\mid \\lambda \\in \\mathbb{R}, \\lambda \\neq-1, \\lambda \\neq-2\\}$\nD. $\\{\\lambda \\mid \\lambda \\in \\mathbb{R}, \\lambda \\neq-2\\}$", "input": "", "output": "由于\n$$\n\\begin{aligned}\n& \\left|\\alpha_1, \\alpha_2, \\alpha_3\\right|=\\left|\\begin{array}{lll}\n\\lambda & 1 & 1 \\\\\n1 & \\lambda & 1 \\\\\n1 & 1 & \\lambda\n\\end{array}\\right|=\\lambda^3-3 \\lambda+2=(\\lambda-1)^2(\\lambda+2), \\\\\n& \\left|\\alpha_1, \\alpha_2, \\alpha_4\\right|=\\left|\\begin{array}{lll}\n\\lambda & 1 & 1 \\\\\n1 & \\lambda & \\lambda \\\\\n1 & 1 & \\lambda^2\n\\end{array}\\right|=\\lambda^4-2 \\lambda^2+1=(\\lambda-1)^2(\\lambda+1)^2 .\n\\end{aligned}\n$$\n\n 当 $\\lambda=1$ 时, $\\alpha_1=\\alpha_2=\\alpha_3=\\alpha_4=\\left(\\begin{array}{l}1 \\\\ 1 \\\\ 1\\end{array}\\right)$, 此时 $\\alpha_1, \\alpha_2, \\alpha_3$ 与 $\\alpha_1, \\alpha_2, \\alpha_4$ 等价.\n当 $\\lambda=-2$ 时, $2=r\\left(\\alpha_1, \\alpha_2, \\alpha_3\\right)r\\left(\\alpha_1, \\alpha_2, \\alpha_4\\right)=1, \\alpha_1, \\alpha_2, \\alpha_3$ 与 $\\alpha_1, \\alpha_2, \\alpha_4$ 不等价. 因 此当 $\\lambda=-2$ 或 $\\lambda=-1$ 时, $\\alpha_1, \\alpha_2, \\alpha_3$ 与 $\\alpha_1, \\alpha_2, \\alpha_4$ 不等价等价, 所以 $\\lambda$ 的取值范围为 $\\{\\lambda \\mid \\lambda \\in \\mathbb{R}, \\lambda \\neq-1, \\lambda \\neq-2\\}$.\n\n因此答案为:C", "task_type": {"major": ["试题"], "minor": ["考研", "选择题"]}, "domain": ["数学"], "metadata": "year:2022, score:5", "answer_from": "human", "human_verified": false, "copyright": "暂无版权及作者信息"} +{"instruction": "设 $X \\sim U(0,3), Y \\sim P(2), \\operatorname{Cov}(X, Y)=-1$, 求 $D(2 X-Y+1)=(\\quad)$.\n(A) 10\n(B) 9\n(C) 1\n(D) 0", "input": "", "output": "由 $X \\sim U(0,3), Y \\sim P(2)$ 知, $D(X)=\\frac{3}{4}, D(Y)=2$, 故\n\n$D(2 X-Y+1)=D(2 X-Y)=4 D(X)+D(Y)-4 \\operatorname{Cov}(X, Y) =4 \\cdot \\frac{3}{4}+2+4=9$.\n\n因此答案为:B", "task_type": {"major": ["试题"], "minor": ["考研", "选择题"]}, "domain": ["数学"], "metadata": "year:2022, score:5", "answer_from": "human", "human_verified": false, "copyright": "暂无版权及作者信息"} +{"instruction": "曲线 $y=x \\ln \\left(e+\\frac{1}{x-1}\\right)$ 的渐近线方程为( )\n(A) $y=x+e$\n(B) $y=x+\\frac{1}{e}$\n(C) $y=x$\n(D) $y=x-\\frac{1}{e}$", "input": "", "output": "$k=\\lim _{x \\rightarrow \\infty} \\frac{y}{x}=\\lim _{x \\rightarrow \\infty} \\frac{x \\ln \\left(e+\\frac{1}{x-1}\\right)}{x}=\\lim _{x \\rightarrow \\infty} \\ln \\left(e+\\frac{1}{x-1}\\right)=1$,\n$$\n\\begin{aligned}\nb & =\\lim _{x \\rightarrow \\infty}(y-k x)=\\lim _{x \\rightarrow \\infty}\\left[x \\ln \\left(e+\\frac{1}{x-1}\\right)-x\\right]=\\lim _{x \\rightarrow \\infty} x\\left[\\ln \\left(e+\\frac{1}{x-1}\\right)-1\\right] \\\\\n& =\\lim _{x \\rightarrow \\infty} x \\ln \\left[1+\\frac{1}{e(x-1)}\\right]=\\lim _{x \\rightarrow \\infty} \\frac{x}{e(x-1)}=\\frac{1}{e}\n\\end{aligned}\n$$\n\n所以斜渐近线方程为 $y=x+\\frac{1}{e}$.\n\n因此答案为:B", "task_type": {"major": ["试题"], "minor": ["考研", "选择题"]}, "domain": ["数学"], "metadata": "year:2023, score:5", "answer_from": "human", "human_verified": false, "copyright": "暂无版权及作者信息"} +{"instruction": "若微分方程 $y^{\\prime \\prime}+a y^{\\prime}+b y=0$ 的解在 $(-\\infty,+\\infty)$ 上有界, 则( )\n(A) $a<0, b>0$\n(B) $a>0, b>0$\n(C) $a=0, b>0$\n(D) $a=0, b<0$", "input": "", "output": "微分方程 $y^{\\prime \\prime}+a y^{\\prime}+b y=0$ 的特征方程为 $\\lambda^{2}+a \\lambda+b=0$,\n\n当 $\\Delta=a^{2}-4 b>0$ 时, 特征方程有两个不同的实根 $\\lambda_{1}, \\lambda_{2}$, 则 $\\lambda_{1}, \\lambda_{2}$ 至少有一个不等于零, 若 $C_{1}, C_{2}$ 都不为零, 则微分方程的解 $y=C_{1} e^{-\\lambda_{1} x}+C_{2} e^{-\\lambda_{2} x}$ 在 $(-\\infty,+\\infty)$\n无界;\n\n当 $\\Delta=a^{2}-4 b=0$ 时, 特征方程有两个相同的实根, $\\lambda_{1,2}=-\\frac{a}{2}$,\n\n若 $C_{2} \\neq 0$, 则微分方程的解 $y=C_{1} e^{-\\frac{a}{2} x}+C_{2} x e^{-\\frac{-a_{x}}{2}}$ 在 $(-\\infty,+\\infty)$ 无界;\n\n当 $\\Delta=a^{2}-4 b<0$ 时, 特征方程的根为 $\\lambda_{1,2}=-\\frac{a}{2} \\pm \\frac{\\sqrt{4 b-a^{2}}}{2} i$,\n\n则通解为 $y=e^{-\\frac{a}{2} x}\\left(C_{1} \\cos \\frac{\\sqrt{4 b-a^{2}}}{2} x+C_{2} \\sin \\frac{\\sqrt{4 b-a^{2}}}{2} x\\right)$,\n\n此时, 要使微分方程的解在 $(-\\infty,+\\infty)$ 有界, 则 $a=0$, 再由 $\\Delta=a^{2}-4 b<0$,知 $b>0$.\n\n因此答案为:C", "task_type": {"major": ["试题"], "minor": ["考研", "选择题"]}, "domain": ["数学"], "metadata": "year:2023, score:5", "answer_from": "human", "human_verified": false, "copyright": "暂无版权及作者信息"} +{"instruction": "设函数 $y=f(x)$ 由 $\\left\\{\\begin{array}{l}x=2 t+|t| \\\\ y=|t| \\sin t\\end{array}\\right.$ 确定, 则()\n\n(A) $f(x)$ 连续, $f^{\\prime}(0)$ 不存在 (B) $f^{\\prime}(0)$ 存在, $f^{\\prime}(x)$ 在 $x=0$ 处不连续\n\n(C) $f^{\\prime}(x)$ 连续, $f^{\\prime \\prime}(0)$ 不存在 (D) $f^{\\prime \\prime}(0)$ 存在, $f^{\\prime \\prime}(x)$ 在 $x=0$ 处不连续", "input": "", "output": "$t \\geq 0$ 时, $\\left\\{\\begin{array}{l}x=3 t \\\\ y=t \\sin t\\end{array}\\right.$, 得 $y=\\frac{x}{3} \\sin \\frac{x}{3} ; t<0$ 时, $\\left\\{\\begin{array}{l}x=t \\\\ y=-t \\sin t\\end{array}\\right.$, 得 $y=-x \\sin x$;\n\n综上, $y=\\left\\{\\begin{array}{l}\\frac{x}{3} \\sin \\frac{x}{3}, x \\geq 0 \\\\ -x \\sin x, x<0\\end{array}\\right.$,\n\n从而由 $y_{+}^{\\prime}(0)=\\lim _{x \\rightarrow 0^{+}} \\frac{\\frac{x}{3} \\sin \\frac{x}{3}-0}{x}=0, y_{-}^{\\prime}(0)=\\lim _{x \\rightarrow 0^{+}} \\frac{-x \\sin x-0}{x}=0$, 得 $y^{\\prime}(0)=0$;\n\n于是 $y^{\\prime}=\\left\\{\\begin{aligned} \\frac{1}{3} \\sin \\frac{x}{3}+\\frac{x}{9} \\cos \\frac{x}{3}, x & >0 \\\\ 0, x & =0, \\\\ -\\sin x-x \\cos x, x & <0\\end{aligned}\\right.$ 得 $y^{\\prime}$ 连续;\n又由 $y_{+}^{\\prime \\prime}(0)=\\lim _{x \\rightarrow 0^{+}} \\frac{\\frac{1}{3} \\sin \\frac{x}{3}+\\frac{x}{9} \\cos \\frac{x}{3}-0}{x}=\\frac{2}{9}, y_{+}^{\\prime \\prime}(0)=\\lim _{x \\rightarrow 0^{+}} \\frac{-\\sin x-x \\cos x-0}{x}=-2$,得 $y^{\\prime \\prime}(0)$ 不存在.\n\n因此答案为:C", "task_type": {"major": ["试题"], "minor": ["考研", "选择题"]}, "domain": ["数学"], "metadata": "year:2023, score:5", "answer_from": "human", "human_verified": false, "copyright": "暂无版权及作者信息"} +{"instruction": "已知 $a_{n}0)$ 是未知参数. 若 $\\hat{\\sigma}=a\\left|X_{1}-X_{2}\\right|$ 为 $\\sigma$ 的无偏估计, 则 $a=(\\quad)$\n(A) $\\frac{\\sqrt{\\pi}}{2}$\n(B) $\\frac{\\sqrt{2 \\pi}}{2}$\n(C) $\\sqrt{\\pi}$\n(D) $\\sqrt{2 \\pi}$", "input": "", "output": "由题可知 $X_{1}-X_{2} \\sim N\\left(0,2 \\sigma^{2}\\right)$.\n\n令 $Y=X_{1}-X_{2}$, 则 $Y$ 的概率密度为 $f(y)=\\frac{1}{\\sqrt{2 \\pi} \\sqrt{2} \\sigma} e^{-\\frac{y^{2}}{2 \\cdot 2 \\sigma^{2}}}$.\n$E(|Y|)=\\int_{-\\infty}^{+\\infty}|y| \\frac{1}{\\sqrt{2 \\pi} \\sqrt{2} \\sigma} e^{-\\frac{y^{2}}{2 \\cdot 2 \\sigma^{2}}} d y=\\frac{2}{\\sqrt{2 \\pi} \\sqrt{2} \\sigma} \\int_{0}^{+\\infty} y e^{-\\frac{y^{2}}{4 \\sigma^{2}}} d y=\\frac{2 \\sigma}{\\sqrt{\\pi}}$,\n\n$E\\left(a\\left|X_{1}-X_{2}\\right|\\right)=a E(|Y|)=a \\frac{2 \\sigma}{\\sqrt{\\pi}}$.\n\n由 $\\left.\\hat{\\sigma}=a\\left|X_{1}-X_{2}\\right|\\right)$ 为 $\\sigma$ 的无偏估计, 有 $E(\\hat{\\sigma})=\\sigma$, 得 $a=\\frac{\\sqrt{\\pi}}{2}$. 故选(A).", "task_type": {"major": ["试题"], "minor": ["考研", "选择题"]}, "domain": ["数学"], "metadata": "year:2023, score:5", "answer_from": "human", "human_verified": false, "copyright": "暂无版权及作者信息"} +{"instruction": "设矩阵 $A, B$ 均为 $n$ 阶方阵, 若 $A x=0$ 与 $B x=0$ 同解, 则 ( ).\nA. $\\left(\\begin{array}{ll}A & O \\\\ E & B\\end{array}\\right) x=0$ 仅有零解\nB. $\\left(\\begin{array}{cc}A B & B \\\\ O & A\\end{array}\\right) x=0$ 仅有零解\nC. $\\left(\\begin{array}{ll}A & B \\\\ O & B\\end{array}\\right) x=0$ 与 $\\left(\\begin{array}{ll}B & A \\\\ O & A\\end{array}\\right) x=0$ 同解\nD. $\\left(\\begin{array}{cc}A B & B \\\\ O & A\\end{array}\\right) x=0$ 与 $\\left(\\begin{array}{cc}B A & A \\\\ O & B\\end{array}\\right) x=0$ 同解", "input": "", "output": "设 $y=\\left(\\begin{array}{l}x_{1} \\\\ x_{2}\\end{array}\\right)$, 这里 $x_{i}(i=1,2)$ 是 $n$ 维列向量.\n若 $\\left(\\begin{array}{ll}A & B \\\\ O & B\\end{array}\\right) y=0$ 与 $\\left(\\begin{array}{ll}B & A \\\\ O & A\\end{array}\\right) y=0$ 同解即 $\\left(\\begin{array}{ll}A & B \\\\ O & B\\end{array}\\right)\\left(\\begin{array}{l}x_{1} \\\\ x_{2}\\end{array}\\right)=0$ 与 $\\left(\\begin{array}{ll}B & A \\\\ O & A\\end{array}\\right)\\left(\\begin{array}{l}x_{1} \\\\ x_{2}\\end{array}\\right)=0$ 同\n解. 由于 $A x=0$ 与 $B x=0$ 同解, 若 $A x=0 \\quad x_{i}(i=1,2)$, 则 $B x_{i}=0(i=1,2)$, 反之亦然. 因\n此 $\\left(\\begin{array}{ll}A & B \\\\ O & B\\end{array}\\right)\\left(\\begin{array}{l}x_{1} \\\\ x_{2}\\end{array}\\right)=0$ 等价于 $\\left(\\begin{array}{ll}B & A \\\\ O & A\\end{array}\\right)\\left(\\begin{array}{l}x_{1} \\\\ x_{2}\\end{array}\\right)=0$, 所以(C)选项符合题意", "task_type": {"major": ["试题"], "minor": ["考研", "选择题"]}, "domain": ["数学"], "metadata": "year:2022, score:5", "answer_from": "human", "human_verified": false, "copyright": "暂无版权及作者信息"} +{"instruction": "马克思认为,教育与生产劳动相结合是 ()\nA. 造就全面发展的人的唯一办法\nB. 实施综合技术教育的唯一手段\nC. 改造现代社会的唯一手段\nD.提高社会生产的唯一方法", "input": "", "output": "马克思在《资本论》中指出:“它”(教育与生产劳动相结合)不仅是提高社会生产的一种方法,而且是造就全面发展的人的唯一方法。\n\n因此答案为:A", "task_type": {"major": ["试题"], "minor": ["考研", "选择题"]}, "domain": ["教育学"], "metadata": "year:2023, score:2", "answer_from": "human", "human_verified": false, "copyright": "暂无版权及作者信息"} +{"instruction": "氨基酸是两性电解质, 下列氨基酸侧链基团中, 在生理条件下既可以作为质子受体, 又可以作为质子供体的是 ()。\nA. His 的㽤唑基\nB. $\\operatorname{Trp}$ 的吲哚基\nC. Arg 的胍基\nD. Cys 的放基", "input": "", "output": "根据侧链基团不同, 氨基酸分为极性不带电荷、极性带正电荷、极性带负电荷以及非极性几类。选项 B 色氨酸是非极性氨基酸, $\\mathrm{D}$ 半胱氨酸属于极性不带电荷氨基酸。 $\\mathrm{A}$ 组氨酸和 C 精氨酸均为极性带正电荷的氨基酸。而组氨酸的咪磋基解离值 $\\mathrm{pK}_{\\mathrm{R}}$ 为 6.04 , 说明在生理 $\\mathrm{pH}$ 条件下, 这个基团可以有一部分解离, 有一部分不解离,既可以作为质子的受体,也可作为质子的供体。\n \n\n因此答案为:A", "task_type": {"major": ["试题"], "minor": ["考研", "选择题"]}, "domain": ["植物生理学"], "metadata": "year:2022, score:2", "answer_from": "human", "human_verified": false, "copyright": "暂无版权及作者信息"} +{"instruction": "$\\alpha$ 螺旋是蛋白质二级结构的常见形式之一,稳定 $\\alpha$ 螺旋的主要作用力是( )。\nA. 酯键\nB. 氢键\nC. 离子键\nD. 二硫键", "input": "", "output": "蛋白质二级结构典型类型包括 $\\alpha$-螺旋和 $\\beta$-折���以及转角等。 $\\alpha$-螺旋和 $\\beta$-折叠的结构特征是考试的重点内容。这里测试的是维持 $\\alpha$-螺旋结构稳定性的主要作用力。选项 $\\mathrm{A}$ 酯键和选项 $\\mathrm{D}$ 二硫键都是共价键,选项 C 离子键在维持蛋白质二级结构中也起作用,但是主要作用是氨键。\n \n\n因此答案为:B", "task_type": {"major": ["试题"], "minor": ["考研", "选择题"]}, "domain": ["植物生理学"], "metadata": "year:2022, score:2", "answer_from": "human", "human_verified": false, "copyright": "暂无版权及作者信息"} +{"instruction": "生物体内的一碳单位与氮基酸代谢密切相关。下列基团或化合物中, 不属于一碳单位的是\nA. $-\\mathrm{CH}_{3}$\nB. $-\\mathrm{CHO}$\nC. $-\\mathrm{CH}=$\nD. $\\mathrm{CO}$", "input": "", "output": "一碳单位在氨基酸和核苷酸代谢中发挥重要作用,当然在其他过程中也有重要的作用。这里考查的是一碳单位的类型。题干中只有 D 选项 CO 不是一碳单位/一碳基团。可以参考《指南》的相应位置或者任何生化教材有关一碳单位的部分进行复习,掌握一碳单位的概念和种类。\n \n\n因此答案为:D", "task_type": {"major": ["试题"], "minor": ["考研", "选择题"]}, "domain": ["植物生理学"], "metadata": "year:2022, score:2", "answer_from": "human", "human_verified": false, "copyright": "暂无版权及作者信息"} +{"instruction": "当人体缺乏某种维生索导致暗适应能力下降时, 可以适当补充()。\nA. 维生素 A\nB. 烟酰胺\nC. 维生素 D\nD. 叶酸", "input": "", "output": "考点是维生素 A 的作用。有关维生素与辅酶的部分作为单选题进行考核是很普遍的事情, 这里每个维生素和辅酶的对应关系以及维生素的活性形式及作用需要掌握。\n \n\n因此答案为:A", "task_type": {"major": ["试题"], "minor": ["考研", "选择题"]}, "domain": ["植物生理学"], "metadata": "year:2022, score:2", "answer_from": "human", "human_verified": false, "copyright": "暂无版权及作者信息"} +{"instruction": "疏基是某些生物分子中的重要基团。下列化合物中,没有疏基的是()。\nA. ACP\nB. Met\nC. $\\operatorname{CoA}$\nD. GSH", "input": "", "output": "本题的考点是学生对含統基分子的了解。选项 A 是酰基载体蛋白 ACP, 参与脂肪酸从头合成途径;选项 $B$ 甲硫氨酸 Met, 它含有 $S$ 元素, 但是不含統基形式; 选项 $C$ 是 $C O A$, 它参与脂肪酸 $B$-氧化, 含有颈基;选项 D 是谷胱甘肽的还原形式, 简写成 GSH, 含有巯基。因此, 答案是选项 $\\mathrm{B}$ ,它不含統基。\n \n\n因此答案为:B", "task_type": {"major": ["试题"], "minor": ["考研", "选择题"]}, "domain": ["植物生理学"], "metadata": "year:2022, score:2", "answer_from": "human", "human_verified": false, "copyright": "暂无版权及作者信息"} +{"instruction": "下列化合物中, 属于三羧酸循环中间产物的是 ()。\nA. 磷酸烯醇式丙酮酸\nB. 延胡索酸\nC. 丙二酸\nD. 磔酸二䍩闪酮", "input": "", "output": "三羧酸循环是考试的重中之重。选项 $\\mathrm{A}$ 和 $\\mathrm{D}$ 是糖酵解和糖异生途径的共同中间产物; 选项 C 丙二酸不是三羧酸循环的中间产物, 只是在谈到琥珀酸脱氢酶的抑制剂时.提到丙二酸与琥珀酸结构类似, 是琥珀酸脱氢酶的竞争性抑制剂; 选项 B 延胡索酸是三羚酸循环的中间产物。\n \n\n因此答案为:B", "task_type": {"major": ["试题"], "minor": ["考研", "选择题"]}, "domain": ["植物生理学"], "metadata": "year:2022, score:2", "answer_from": "human", "human_verified": false, "copyright": "暂无版权及作者信息"} +{"instruction": "下列参与糖代谢的酶中, 既参与糖酔解又参与糖异生的是 ()。\nA. 转醛酶\nB. 丙酮酸激酶\nC. 转酮酶\nD. 鏗缩酶", "input": "", "output": "与三羧酸循环一样, 糖酵解和葡萄糖异生途径几乎是必考内容。选项 $\\mathrm{A}$ 和 C 是在磷酸戊糖途径非氧化阶段讲过的两个酶; 选项 $\\mathrm{B}$ 丙䤍酸激酶是催化糖酵解途径的最后一步反应的酶, 该反应不可逆, 是不参与葡萄糖异生的; 选项 D 醛缩酶是糖酵解和葡萄糖异生途径共有的酶。\n \n\n因此答案为:D", "task_type": {"major": ["试题"], "minor": ["考研", "选择题"]}, "domain": ["植物生理学"], "metadata": "year:2022, score:2", "answer_from": "human", "human_verified": false, "copyright": "暂无版权及作者信息"} +{"instruction": "下列酶中, 催化糖原生成 1-磷酸葡蓬糖的是()。\nA. 糖原磷酸化酶\nB. 糖原合酶\nC. UDPG 焦磷酸化酶\nD. 磷酸葡葏糖变位酶", "input": "", "output": "本题考查的是学生对糖原代谢的掌握情况。选项 B 糖原合酶参与的是糖原合成过程中糖链的延伸;选项 C UDPG 焦磷酸化酶参与的是糖原合成时活性葡萄糖 UDPG 的合成; 选项 D 磷酸葡萄糖变位酶催化的是 1-磷酸葡萄糖和 6-磷��葡萄糖之间的转化反应; 而选项 A 糖原磷酸化酶参与的才是和糖原降解直接相关的反应,从糖原分子的非还原端切断葡萄糖之间的 $a-1 , 4-$-糖苷键,产生 1-磷葡葡糖\n \n\n因此答案为:A", "task_type": {"major": ["试题"], "minor": ["考研", "选择题"]}, "domain": ["植物生理学"], "metadata": "year:2022, score:2", "answer_from": "human", "human_verified": false, "copyright": "暂无版权及作者信息"} +{"instruction": "下列有关大肠杆菌中转录的叙述, 正确的是()。\nA. RNA 聚合酶的作用需要引物\nB. 转录需要 $\\mathrm{dNTP}$ 作为底物\nC. 核心酶催化 RNA 链延伸\nD. 转录起始依赖 $\\rho$ 因子", "input": "", "output": "本题考查的是 RNA 合成和 RNA 聚合酶的功能。选项 A “RNA 聚合酶的作用需要引物” 是错误的, DNA 聚合酶合成 DNA 时需要引物; 选项 B RNA 聚合酶的底物是 NTP, 而不是 DNTP; 选项 D “转录起始依赖于 $\\rho$ 因子”, 其实原核生物转录起始依赖于 $\\sigma$ 因子, 而终止时依赖于 $\\rho$ 因子; 选项 C RNA 聚合酶核心酶能够化 RNA 的聚合, 所以该选项是正确的", "task_type": {"major": ["试题"], "minor": ["考研", "选择题"]}, "domain": ["植物生理学"], "metadata": "year:2022, score:2", "answer_from": "human", "human_verified": false, "copyright": "暂无版权及作者信息"} +{"instruction": "核酸变性后理化性质会发生变化。双键 DNA 热变性后会出现的现象是()。\nA. $T_{\\mathrm{n}}$ 值升高\nB. 移度升高\nC. 璔色效应\nD. 减色效", "input": "", "output": "本题考查的是 DNA 的变性。选项 A $\\mathrm{T}_{\\mathrm{m}}$, 熔解温度, 代表 DNA 变性一半时的温\n度, 该值不会随着 DNA 热变性而发生改变; 双键 DNA 变性后, 款度会减小, 所以选项 B 错误; 双䥽 DNA 热变性后, 由于碱基外翻会导致紫外吸收增加, 即产生所谓的增色效应; 当在合适的条件下, 解链后的单链 DNA 互补重新形成双螺旋时, 紫外吸收会减小, 产生减色效应。所以,选项 C 正确。\n\n因此答案为:C", "task_type": {"major": ["试题"], "minor": ["考研", "选择题"]}, "domain": ["植物生理学"], "metadata": "year:2022, score:2", "answer_from": "human", "human_verified": false, "copyright": "暂无版权及作者信息"} +{"instruction": "习近平总书记在为 《复兴文库》所写的序言中指出: “修史立典, 存史启智, 以文化人,这是中华民族延续几千年的一个传统。” “编纂出版《复兴文库》大型历史文献从书,就是要通过对近代以来重要思想文献的选编, 述录先人的开拓, 启迪来者的奋斗。” “历史是最好的教科书, 一切向前走, 都不能忘记走过的路; 走得再远、走到再光辉的未来, 也不能忘记走过的过去。”这表明\nA. 历史、现实、未来是相通的\nB. 一切历史都是当代史\nC. 历史过程不包含任何偶然的因素\nD. 历史事件往往可以完整重复和再配", "input": "", "output": "题干中强调了历史的重要作用以及未来如何发展也不能忘记过去, 体现了历史、现实、未来是想通的。选项 B 说法片面, C 说法错误, 历史过程既有偶然因素也有历史的必然, 选项 D 说法错误, 历史事件中反映出的规律可能再现, 但是不是完整的重复和再现。故本题正确选项为 A 。\n\n因此答案为:A", "task_type": {"major": ["试题"], "minor": ["考研", "选择题"]}, "domain": ["政治"], "metadata": "year:2023, score:1", "answer_from": "human", "human_verified": false, "copyright": "暂无版权及作者信息"} +{"instruction": "社会形态是关于社会运动的具体形式、发展阶段和不同质态的范畴, 是同生产力发展一定阶段相适应的经济基础与上层建筑的统一体。人类社会历史划分为原始社会、奴隶社会、封建社会、资本主义社会和共产主义社会(社会主义社会是其第一阶段)五种社会形态,其依据是\nA. 生产工具的质量和数量\nB. 统治集团的阶级和政治属性\nC. 经济基础特别是生产关系的性质\nD. 人们社会交往和分工的范围和水平", "input": "", "output": "本题考查社会形态更替的必然性, 社会形态更替的客观必然性, 主要是指社会形态依次更替的过程和规律是客观的, 其发展的基本趋势是确定不移的。社会形态更替归根结底是社会基本矛盾运动的结果。其中, 生产力的发展具有最终的决定意义。生产力与生产关系矛盾运动的规律性, 从根本上规定了社会形态更替的客观必然性。故本题正确选项为 C。\n\n因此答案为:C", "task_type": {"major": ["试题"], "minor": ["考研", "选择题"]}, "domain": ["政治"], "metadata": "year:2023, score:1", "answer_from": "human", "human_verified": false, "copyright": "暂无版权及作者信息"} +{"instruction": "马克思在《资本论》中指出: “不管生产力发生了什么变化。同一劳动在同样的时间内提供的价值量总是相同的。但它在同样的时间内提供的使用价值量是不同的, 生产力提高时就多些, 生产力降低时就少些。”这段话表明, 生产力的变化\nA. 不影响同一劳动在同样的时间内生产的商品数量\nB. 不改变同一劳动在同样的时间内的生产效率\nC. 影响同一劳动在同样的时间内生产的单个商品价值量\nD. 改变同一劳动在同样的时间内提供的价值总量", "input": "", "output": "本题考查商品的价值量, 商品价值的量是由生产商品所耗费的劳动量决定的,而劳动量则是按劳动时间来计量的。生产力的变化会影响劳动生产率, 商品的价值量与生产商品所耗费的劳动时间成正比, 与劳动生产率成反比。故本体正确选项为 $\\mathrm{C}$ 。\n\n因此答案为:C", "task_type": {"major": ["试题"], "minor": ["考研", "选择题"]}, "domain": ["政治"], "metadata": "year:2023, score:1", "answer_from": "human", "human_verified": false, "copyright": "暂无版权及作者信息"} +{"instruction": "垄断组织通过制定垄断价格来获取垄断利润。在市场交易过程中, 垄断组织获利高低实际上受到很多因素的影响。为了避免遭受净亏损, 垄断组织所能接受的最低价格是\nA. 成本价格\nB. 生产价格\nC. 垄断低价\nD. 垄断高价", "input": "", "output": "本题考查垄断利润和垄断价格。垄断价格是垄断组织在销售或购买商品时,凭借其垄断地位规定的、旨在保证获取最大限度利润的市场价格。其公式是: 垄断价格 = 成本价格 + 平均利润 + 垄断利润。故本题正确选项为 $\\mathrm{A}$ 。垄断低价是指垄断组织再购买非垄断企业所生产的原材料等生产资料时规定的低于生产价格的价格。其余选项不符合题意。", "task_type": {"major": ["试题"], "minor": ["考研", "选择题"]}, "domain": ["政治"], "metadata": "year:2023, score:1", "answer_from": "human", "human_verified": false, "copyright": "暂无版权及作者信息"} +{"instruction": "什么是社会主义, 怎样建设社会主义, 是邓小平在改革开放和现代化建设过程中, 不断提出和反复思考的首要的基本的理论问题。搞清这一基本理论问题, 关键是要\nA. 科学分析我国社会主要矛盾\nB. 廓清不合乎时代进步和社会发展规律的模糊观念\nC. 系统总结社会主义社会发展的经验\nD. 在坚持社会主义基本制度基础上进一步认识社会主义本质", "input": "", "output": "本题考查邓小平理论回答的基本问题,搞清楚什么是社会主义、怎样建设社会主义, 关键是要在坚持社会主义基本制度的基础上进一步认清社会主义的本质。故本题正确选项为 D 。\n\n因此答案为:D", "task_type": {"major": ["试题"], "minor": ["考研", "选择题"]}, "domain": ["政治"], "metadata": "year:2023, score:1", "answer_from": "human", "human_verified": false, "copyright": "暂无版权及作者信息"} +{"instruction": "共同富裕是中国特色社会主义的本质要求。分配制度是促进共同富裕的基础性制度。坚持按劳分配、多种分配方式并存, 构建初次分配、再分配、第三次分配协调配套的制度体系, 对于处理效率和公平关系, 逐步缩小差距具有非常重要的意义。在这一制度体系中, 除了健全完善初次分配、再分配机制外, 还要重视发挥第三次分配的作用。当前, 要做好第三次分配的作用, 需要做的是\nA. 提高劳动报酬在分配中比重\nB. 加大税收、社保、转移支付等的调节力度\nC. 引导、支持有意愿的有能力的企业、社会组织和个人积极参与公益慈善事业\nD. 健全劳动、资本、土地等生产要素按贡献决定报酬机制", "input": "", "output": "本题考查收入分配制度。\"第三次分配\"的内涵与特征, 主要包括以下四个方面: 第一, 第三次分配的动力机制是基于社会机制的爱心驱动, 区别于第一次分配是基于市场机制的利益分配,第二次分配是基于行政机制的强制性调节。慈善是以捐赠财产或者提供服务等方式自愿开展的公益活动, 是第三次分配的主要形式。故本题正确选项为 $\\mathrm{C}$ 。\n\n因此答案为:C", "task_type": {"major": ["试题"], "minor": ["考研", "选择题"]}, "domain": ["政治"], "metadata": "year:2023, score:1", "answer_from": "human", "human_verified": false, "copyright": "暂无版权及作者信息"} +{"instruction": "坚持总体国家安全观, 必须坚持国家利益至上, 以人民安全为宗旨, 以政治安全为根本, 以经济安全为基础, 以军事、科技、文化、社会安全为保障, 以促进国际安全为依托,统筹外部安全和内部安全、国土安全和国民安全、传统安全和非传统安全、自身安全和共同安全, 统筹维护国家安全和塑造国家安全。坚持总体国家安全观, 归根到底是为了\nA. 顺应世界变化发展的新趋势\nB. ��强全民国家安全意识\nC. 破解国家发展中遇到的安全难题\nD. 确保中华民族伟大复兴进程不被迟滞甚至中断", "input": "", "output": "本题考查坚持总体国家安全观。坚持总体国家安全观,适应了通行具有许多新的历史特点的伟大斗争的新要求。当前, 我国迎来了大有可为的历史机遇期, 前景十分光明, 但风险挑战也十分严峻。在新的历史起点上, 必须时刻准备应对各种风险考验。坚持总体国家安全观, 归根到底是为了更好维护我国发展的历史机遇期, 确保中华民族伟大复兴进程不被迟滞或打断。故本题正确选项为 D。\n\n因此答案为:D", "task_type": {"major": ["试题"], "minor": ["考研", "选择题"]}, "domain": ["政治"], "metadata": "year:2023, score:1", "answer_from": "human", "human_verified": false, "copyright": "暂无版权及作者信息"} +{"instruction": "作为无产阶级政党, 如何永远得到人民的拥护和支持, 如何跳出治乱兴衰周期率, 实现长期执政, 是中国共产党必须回答好、解决好的根本性问题。对于这一历史之问、时代之问, 毛泽东统治在延安窑洞里给出了第一个答案, “只有让人民来监督政府, 政府才不敢松解, 只有人人起来负责, 才不会人亡政息。”经过党十八大以来的不解努力, 我党找到了第二个答案, 这就是\nA. 坚定理想\nB. 自我革命\nC. 把权力关进制度的笼子\nD. 增强忧患意识", "input": "", "output": "本题考查党的建设。中国共产党立志于中华民族千秋伟业, 要把新时代坚持和发展中国特色社会主义这场伟大社会革命进行好, 党必须勇于进行自我革命, 全面增强党的执政本领,以领导人民进行的伟大社会革命。故本体正确选项为 B。\n\n因此答案为:B", "task_type": {"major": ["试题"], "minor": ["考研", "选择题"]}, "domain": ["政治"], "metadata": "year:2023, score:1", "answer_from": "human", "human_verified": false, "copyright": "暂无版权及作者信息"} +{"instruction": "中国是一个文明古国, 为人类文明作出了不可磨灭的贡献。经过工业革命, 到 19 世纪中叶, 西方资本主强国经济和技术飞速发展。中国错失了工业革命的机遇, 大幅落后于时代的原因是\nA. 中国的封建统治者夜郎自大、闭关锁国\nB. 资本一帝国主义国家对华进行资本输出\nC. 中国社会阶级关系发生深刻变动\nD. 中国被纳入资本主义世界体系", "input": "", "output": "本题考查中国封建社会的衰落。17 世纪下半叶至 18 世纪, 清朝康熙、雍正、乾隆年间是中国封建社会后期的鼎盛时期,但同时也开始走向了封建社会的末世,潜伏着许多危机, 而且闭关自守, 故步自封。到了鸦片战争前夜, 它已经衰相尽显, 与新兴的西方资本主义国家拉开了很大的差距。故本题正确选项为 A 。\n\n因此答案为:A", "task_type": {"major": ["试题"], "minor": ["考研", "选择题"]}, "domain": ["政治"], "metadata": "year:2023, score:1", "answer_from": "human", "human_verified": false, "copyright": "暂无版权及作者信息"} +{"instruction": "大革命失败后, 社会被白色恐怖笼罩着, 于是中共中央 1927 年 8 月 7 日在汉口召开了八七会议。这次会议\nA. 批评了大革命后期的左倾倾向\nB. 提出了党的工作重心由城市向乡村转移的必要性\nC. 提出了组织湘鄂粤赣农民秋收起义\nD. 确立了土地改革和武装起义方针", "input": "", "output": "本题考查八七会议。八七会议会议确定了土地革命和武装起义的方针。出席这次会议的毛泽东在发言中突出地强调: “以后要非常注意军事。须知政权是由枪杆子\n中取得的。”七会议给正处在思想混乱和组织涣散中的中国共产党指明了新的出路, 这是由大革命失败到土地革命战争兴起的历史性转变。故本题正确选项为 D。\n\n因此答案为:D", "task_type": {"major": ["试题"], "minor": ["考研", "选择题"]}, "domain": ["政治"], "metadata": "year:2023, score:1", "answer_from": "human", "human_verified": false, "copyright": "暂无版权及作者信息"} +{"instruction": "延安精神最主要体现了党的宗旨是\nA. 全心全意为人民服务\nB. 坚定正确的政治方向\nC. 解放思想实事求是\nD. 自力更生艰苦奋斗", "input": "", "output": "本题考查延安精神。延安精神体现党的宗旨是全心全意为人民服务。故本题正确选项为 $\\mathrm{A}$ 。\n\n因此答案为:A", "task_type": {"major": ["试题"], "minor": ["考研", "选择题"]}, "domain": ["政治"], "metadata": "year:2023, score:1", "answer_from": "human", "human_verified": false, "copyright": "暂无版权及作者信息"} +{"instruction": "中国社会步入社会主义社会的标志\n$A$ 社会主义政治制度的确立\n$B$ 社会主义经济制度的确立\nC 民主革命遗留任务的完成\nD 人民民主专政政权的建立", "input": "", "output": "本题考查社会主义基本制度的确立。随着社会主义改造的完成, 以生产资料公有制、按劳分配和计划经济体制为特征的社会主义经济制度建立起来,这是中国进入社会主义社会最主要的标志。故本题正确选项为 B。\n\n因此答案为:B", "task_type": {"major": ["试题"], "minor": ["考研", "选择题"]}, "domain": ["政治"], "metadata": "year:2023, score:1", "answer_from": "human", "human_verified": false, "copyright": "暂无版权及作者信息"} +{"instruction": "公民道德建设对提高人民思想觉悟、道德水准、文明素养, 提高全社会文明程度具有至关重要的作用。适应新时代新要求, 党中央根据变化了的形势和公民道德建设的新需要,于 2019 年颁布了《新时代公民道德建设实施纲要》, 明确强调新时代公民道德建设的着力点是\nA. 弘扬民族精神和时代精神\nB. 推动理想信念教育常态化制度化\nC. 推进社会公德、职业道德、家庭美德、个人品德建设\nD. 传承孝老爱亲、扶危济困、见义勇为等中华美德", "input": "", "output": "本题考查公民道德建设。新时代公民道德建设的着力点是推进社会公德、职业道德、家庭美德、个人品德建设。故本题正确选项为 C.\n\n因此答案为:C", "task_type": {"major": ["试题"], "minor": ["考研", "选择题"]}, "domain": ["政治"], "metadata": "year:2023, score:1", "answer_from": "human", "human_verified": false, "copyright": "暂无版权及作者信息"} +{"instruction": "坚持全面依法治国首先要\nA. 坚持依宪治国\nB. 坚持依法行政\nC. 坚持依法执政\nD. 坚持公正司法", "input": "", "output": "本题考查坚持全面依法治国。坚持全面依法治国首先要依宪治国。故本题正确选项为 $\\mathrm{A}$.\n\n因此答案为:A", "task_type": {"major": ["试题"], "minor": ["考研", "选择题"]}, "domain": ["政治"], "metadata": "year:2023, score:1", "answer_from": "human", "human_verified": false, "copyright": "暂无版权及作者信息"} +{"instruction": "15.2022 年 4 月 8 日, 背景冬奥会、冬残奥会总结表彰大会在背景举行。习近平总书记强调, 伟大事业孕育伟大精神, 伟大精神推进伟大事业。我国广大参与者珍惜伟大时代赋予的机遇, 在冬奥申办、筹办、举办的过程中, 共同创造了北京冬奥精神。北京冬奥精神是\nA. 更快、更高、更强、更团结\nB. 胸怀大局、自信开放、迎难而上、追求卓越、共创未来\nC. 勇气、决心、激励、平等\nD. 一起向未来", "input": "", "output": "冬奥精神是胸怀大局、自信开放、迎难而上、追求卓越、共创未来。故本题正确选项为 B。\n\n因此答案为:B", "task_type": {"major": ["试题"], "minor": ["考研", "选择题"]}, "domain": ["政治"], "metadata": "year:2023, score:1", "answer_from": "human", "human_verified": false, "copyright": "暂无版权及作者信息"} +{"instruction": "2022 年 11 月 11 日, 第二十五次中国-东盟领导人会议在柬埔寨金边举行, 发表了中国-东盟国家就南海问题达成的又一份重要文件。这份重要文件是\nA. 《中国和东盟国家外交部长关于全面有效落实〈南海各方行为宣言〉联合声明》\nB. 落实《南海各方行为宣言》指导方针\nC. 纪念《南海各方行为宣言》签署十周年声明\nD. 纪念 《南海各方行为宣言》签署二十周年声", "input": "", "output": "2022 年 11 月 11 日,第二十五次中国-东盟领导人会议在束埔寨首都金边举行。会议通过了《纪念<南海各方行为宣言>签署二十周年联合声明》。故本题正确选项为 $\\mathrm{D}$ 。\n\n因此答案为:D", "task_type": {"major": ["试题"], "minor": ["考研", "选择题"]}, "domain": ["政治"], "metadata": "year:2023, score:1", "answer_from": "human", "human_verified": false, "copyright": "暂无版权及作者信息"} +{"instruction": "恩格斯说: “单凭观察所得的经验, 是决不能充分证明必然性的。这是如此正确,以至于不能从太阳总是在早晨升起来判断它明天会再升起。”这说明 ()\nA. 感性认识有待于上升为理性认识\nB. 感性认识具有局限性\nC. 事物的必然性与感性认识毫无关系\nD. 归纳方法是有局限性的", "input": "", "output": "本题考查个人在社会历史中的作用。无产阶级领袖在历史上所提出的思想能够成为社会变革的先导, 引领社会发展的方向。能够影响历史事件的进程, 凝聚人民群众的力量, 但是, 不能创造人类社会的历史, 不管什么样的历史人物, 在历史上发挥什么样的作用, 都要受到社会发展客观规律的制约, 而不能决定和改变历史发展的总进程和总方向, 所以不能创造人类社会的历史。故选项 $\\mathrm{D}$ 说法错误, 本题正确选项为 $\\mathrm{ABC}$ 。", "task_type": {"major": ["试题"], "minor": ["考研", "选择题"]}, "domain": ["政治"], "metadata": "year:2023, score:2", "answer_from": "human", "human_verified": false, "copyright": "暂无版权及作者信息"} +{"instruction": "苏轼在《题西林壁》中写道\"横看成岭侧成峰, 远近高低各不同”人们之所以对同一个事物会有横侧、远近、高低等评价差异,是因为:\nA. 评价客体没有自身固有的本质与特性\nB. 评价过程与评价主体有着直接的关系\nC. 评价没有正确与错误、善恶的区别\nD. 评价结果随之客体的相互关系而改变。", "input": "", "output": "本题考查价值评价及其特点。评价结果取决于人对客体和主体的双重认识,这种认识不仅包括对客体属性和规律的认识, 也包括对主体的规定性和需要的认识。只有对主体和客体都有了正确认识, 才能对主客体间的价值关系作出正确评价。因此选项 D 说法正确, A 说法错误。选项 B 正确, 评价结果与评价主体直接相关。知识性认识是人的主观反映客观的过程, 认识结果不以主体意志为转移; 而价值评价则要揭示和表达客体对于主体的意义。因而主体的存在状态作为价值关系的构成要素必然会对评价结果产生影响, 使评价结果受到主体意志的影响。选项 C 说法错误, 评价结果有正确与错误之分, 价值评价也有科学与非科学之别。", "task_type": {"major": ["试题"], "minor": ["考研", "选择题"]}, "domain": ["政治"], "metadata": "year:2023, score:2", "answer_from": "human", "human_verified": false, "copyright": "暂无版权及作者信息"} +{"instruction": "马克思的平均利润理论深刻阐明了资本主义社会利润转换为平均利润以及剩余价值被进一步分割的原理,在利润率平均化的过程中,仍然能获得平均利润的资本有()\nA. 产业资本\nB. 农业资本\nC. 商业资本\nD. 借贷资本", "input": "", "output": "本题考查平均利润的形成。在利润平均化规律作用下, 产业资本家获得产业利润, 商业资本家获得商业利润: 银行资本家获得银行利润, 农业资本家获得农业利润。利润平均化规律, 反映了在瓜分剩余价值上, 资本家之间存在竞争和矛盾, 但在加强对工人阶级的剥削以樎取更大量的剩余价值这一点上,资本家之间有着共同的阶级利益。按等量资本取得等量利润的原则前提是资本产生的剩余价值转化为利润。借贷资本不属于等量获得平均利润的资本, 借贷资本的提供者不参与生产过程, 实际上是借贷资本的使用者使用借贷资本实现了剩余价值, 剩余价值转化为利润,所以借贷资本的利息率不等于平均利润率。\n\n因此答案为:A B C", "task_type": {"major": ["试题"], "minor": ["考研", "选择题"]}, "domain": ["政治"], "metadata": "year:2023, score:2", "answer_from": "human", "human_verified": false, "copyright": "暂无版权及作者信息"} +{"instruction": "自 20 世纪 70 年代由美国主导的步雷顿森林体系崩溃以来,资本主义国家的金融垄断资本获得了快速发展。金融垄断得以形成和壮大的重要制度条件是 ()\nA. 金融创新\nB. 金融市场监管加强\nC. 金融自由化\nD. 金融危机频繁发生", "input": "", "output": "本题考查金融资所资本的发展。金融自由化与金融创新是金融资所资本得以形成和法大的重要制度条件, 推动着资本主义经济的金融化程度不断提高。故本题正确选项为 $\\mathrm{AC}$ 。\n\n因此答案为:A C", "task_type": {"major": ["试题"], "minor": ["考研", "选择题"]}, "domain": ["政治"], "metadata": "year:2023, score:2", "answer_from": "human", "human_verified": false, "copyright": "暂无版权及作者信息"} +{"instruction": "十月革命胜利后, 为捍卫年轻的苏维埃政权, 列宁领导的苏维埃政权从 1918 年夏到 1921 年春实行了战时共产主义政策。战时共产主义政策是苏维埃俄国面临帝国主义武装干涉和国内战争被迫采取的一种临时性政策。战时国产主义政策主要内容有 () 。\nA. 余粮收集制\nB. 取消商品货币关系\nC. 发展多种所有制\nD. 支持农民耕种自留地", "input": "", "output": "本题考查社会主义要苏联一国的实践。从 1918 年夏到 1921 年春,列宁领导的苏维埃政权实行了以取消商品货币关系为主要特征的战时共产主义政策,在经济上采取一系列非常的、特殊的义带有强制性的措施。选项 AB 说法正确。选项 CD 说法错误,战时共产主义政策取消了自由贸易, CD 不符合题意。", "task_type": {"major": ["试题"], "minor": ["考研", "选择题"]}, "domain": ["政治"], "metadata": "year:2023, score:2", "answer_from": "human", "human_verified": false, "copyright": "暂无版权及作者信息"} +{"instruction": "中国共产党的历史, 是一部不断推进的马克思主义中国化、时代化的历史, 是一部不断推进理论创新、进行理论创造的历史。在这一历程中, 毛泽东是马克思主义中国���的伟大开拓者和毛泽东思想的主要创立者, 其主要贡献是\nA. 第一次明确提出了 “马克思主义中国化” 的科学命题和重大任务\nB. 深刻论证了马克思主义中国化的必要性和重要性\nC. 系统阐述了马克思主义中国化的科学内涵\nD. 全面清除了马克思主义中国化认识上的各种障碍", "input": "", "output": "本题考查毛泽东思想的历史地位。在中国共产党的历史上, 毛泽东第一个明确提出了“马克思主义中国化”的命题, 深刻论证了马克思主义中国化的必要性和极端重要性, 系统阐述了马克思主义中国化的科学内涵和实现马克思主义中国化的正确途径,开辟了马克思主义在中国发展的道路。故本题正确选项为 $\\mathrm{ABC}$ 。选项 D 说法错误。\n\n因此答案为:A B C", "task_type": {"major": ["试题"], "minor": ["考研", "选择题"]}, "domain": ["政治"], "metadata": "year:2023, score:2", "answer_from": "human", "human_verified": false, "copyright": "暂无版权及作者信息"} +{"instruction": "资本是社会主义市场经济的重要生产要素。党的十八大以来, 我们对资本性质的理解逐步深化, 对资本作用的认识更趋全面, 对资本规律的把握更加深入, 对资本运行的治理能力不断提高。对社会主义市场经济条件下的资本已经形成的认识主要有 ()\nA. 资本是带动各类生产要素集聚配置的重要纽带, 是促进社会生产力发展的重要力量\nB. 规范和约束资本的逐利本性, 防止对经济社会发展带来不可估量的危害\nC. 完善我国资本市场基础制度,为各类资本发展释放出更大空间\nD. 既注重保障资本参与社会分配获得增殖和发展, 更注重维护按劳分配的主体地位", "input": "", "output": "本题考查社会主义市场经济。习近平总书记在主持学习时强调, 资本是社会主义市场经济的重要生产要素, 在社会主义市场经济条件下规范和引导资本发展, 既是一个重大经济问题、也是一个重大政治问题, 既是一个重大实践问题、也是一个重大理论问题。资本和土地、劳动力、技术、数据等生产要素共同为社会主义市场经济繁荣发展做出了贡献, 其起到的积极作用应该得到充分肯定。因此, 要用科学、客观的态度对待资本,要正确认识和把握资本的历史意义,注重激发包括非公有资本在内的各类资本活力, 发挥其促进科技进步、繁荣市场经济、便利人民生活、参与国际竞争的积极作用,使之始终服务于人民和国家利益。应该承认,资本具有逐利本性,如不加以规范和约束,就会给经济社会发展带来危害。\"要正确认识和把握资本的特性和行为规律。社会主义市场经济是一个伟大创造, 社会主义市场经济中必然会有各种形态的资本,要发挥资本作为生产要素的积极作用, 同时有效控制其消极作用。\"选项 ABCD 说法都正确。\n\n因此答案为:A B C D", "task_type": {"major": ["试题"], "minor": ["考研", "选择题"]}, "domain": ["政治"], "metadata": "year:2023, score:2", "answer_from": "human", "human_verified": false, "copyright": "暂无版权及作者信息"} +{"instruction": "开放是人类文明进的重要动力, 是世界繁荣发展的必由之路。面对世界范围内保护主义抬头, 逆全球化趋势加剧的现实中国于 2018-2022 年连续五年举办了五届世界上首个以进口为主题的国宝级国际进口博览会。前四届有 120 多家世界级 500 强及行业龙头企业连续参赛, 累计意向成交额达 2700 多亿美元。今年参加的第五届中国国际进口博览会的世界 500 强和行业龙头超过 280 家, 回头率近 $90 \\%$, 累计意向成交 735.2 亿美元, 中国国际进口博览会的成功举办充分表明, 它正成为 $($ )。\nA. 世界经济增长的引擎.\nB. 我国构建新发展格局的窗\nC. 我国接近高水平对外开放的平台.\nD. 全球共享的国际公共产品", "input": "", "output": "本题考查进博会的积极意义。在中国国际进口博览会开幕式上习近平指出, 5 年前, 我宣布举办进博会, 就是要扩大开放, 让中国大市场成为世界大机遇。现在,进博会已经成为中国构建新发展格局的窗口、推动高水平开放的平台、全球共享的国际公共产品。故本题正确选项为 $\\mathrm{BCD}$ 。\n\n因此答案为:B C D", "task_type": {"major": ["试题"], "minor": ["考研", "选择题"]}, "domain": ["政治"], "metadata": "year:2023, score:2", "answer_from": "human", "human_verified": false, "copyright": "暂无版权及作者信息"} +{"instruction": "湿地保护是生态文明建设的重要内容。古往今来人类逐水而居, 文明伴水而生, 人类生产生活同湿地有着密切联系, 十年来, 我国大力推进生态文明建设, 加强湿地保护修复,构建保护制度体系, 出台《湿地保护法》, 使湿地生态状况得以持续��善, 目前, 我国湿地面积达到 5635 万公顷。在《湿地公约》认定的 43 个国际湿地城市中, 我国 13 个城市入选,是全球入选国际湿地城市数量最多的国家。湿地保护有利于 ()\nA. 促进人与自然和谐共生\nB. 推行草原、森林、河流、湖泊、湿地休养生息\nC. 提升生态系统多样性、稳定性、持续性\nD. 发挥湿地功能, 推进持续发展, 应对气候变化", "input": "", "output": "本题考查湿地保护的意义。湿地保护有利于 (促进人与自然和谐共生, 推行草原、森林、河流、湖泊、湿地休养生息, 提升生态系统多样性、稳定性、持续性, 发挥湿地功能, 推进可持续发展, 应对气候变化。故本题正确选项为 $\\mathrm{ABCD}$ 。\n\n因此答案为:A B C D", "task_type": {"major": ["试题"], "minor": ["考研", "选择题"]}, "domain": ["政治"], "metadata": "year:2023, score:2", "answer_from": "human", "human_verified": false, "copyright": "暂无版权及作者信息"} +{"instruction": "坚持和发展马克思主义, 必须同中华优秀传统文化相结合, 中华优秀传统文化源远流长, 博大精深是中华文明的智慧结晶, 蕴含着天下为公、民为根本、为政为德、革故鼎新、任人唯贤、天人合一、自强不息、厚德载物、讲信修睦、亲仁善邻等思想观念, 这些 ()\nA. 与马克思主义理论一脉相承\nB. 同科学社会主义价值观主张是有高度契合性\nC. 完整体现了马克思主义的主场观点和方法\nD. 体现着中国人民的宇宙观、天下观、社会观、道德观", "input": "", "output": "本题考查坚持和发展马克思主义。中华优秀传统文化源远流长、博大精深,是中华文明的智慧结晶, 其中蕴含的天下为公、民为邦本、为政以德、革故鼎新、任人唯贤、天人合一、自强不息、厚德载物、讲信修睦、亲仁善邻等, 是中国人民在长期生产生活中积累的字宙观、天下观、社会观、道德观的重要体现, 同科学社会主义价值观主张具有高度契合性。故本题正确选项为 $\\mathrm{BD}$ 。选项 $\\mathrm{A}$ 一脉相承不能用来描述中国传统文化和马克思主义之间的关系。选项 C 错在不是完整体现, 应该是部分体现。故本题正确选项为 $\\mathrm{BD}$ 。\n\n因此答案为:B D", "task_type": {"major": ["试题"], "minor": ["考研", "选择题"]}, "domain": ["政治"], "metadata": "year:2023, score:2", "answer_from": "human", "human_verified": false, "copyright": "暂无版权及作者信息"} +{"instruction": "1905 年至 1907 年间, 围绕中国社会变革问题, 资产阶级革命派与改变派分别从《民报》、《新民从报》为主要與论阵地, 展开了一场论战, 这场论战 ()\nA. 聚焦要不要以革命手段推翻清王朝\nB. 实质上是资产阶级思想与封建主义思想的交锋\nC. 传播了民主革命思想,促进了革命形势的发展\nD. 暴露了革命派在思想理论方面的弱点", "input": "", "output": "本题考查资产阶级革命派与改良派的论战。革命派与改良派论战的焦点是要不要以革命手段推翻清王朝, 选项 A 正确。通过这场论战, 划清了革命与改良的界限,传播了民主革命思想, 促进了革命形势的发展, 选项 C 正确。但是这场论战也暴露了革命派在思想理论方面的弱点, 选项 D 正确。选项 B 指的是维新派和守旧派的论战, 不符合题意, 不选。故本题正确选项为 $A C D$ 。", "task_type": {"major": ["试题"], "minor": ["考研", "选择题"]}, "domain": ["政治"], "metadata": "year:2023, score:2", "answer_from": "human", "human_verified": false, "copyright": "暂无版权及作者信息"} +{"instruction": "《反对本本主义》是毛泽东在土地革命战争时期为反对当时红军中的教条主义思想而写的重要著作。毛泽东在这篇著作中提出的重要思想有 $($ 。\nA. 关心群众, 注意工作方法\nB. 把城市工作和根据地工作提到同样重要的地位\nC. 没有调查, 没有发言权\nD. 中国革命的斗争的胜利要靠中国同志了解中国情况", "input": "", "output": "本题考查对中国革命新道路的探索。1930 年 5 月, 毛泽东在《反对本本主义》一文中,阐明了坚持辩证唯物主义的思想路线即坚持理论与实际相结合的原则的极端重要性, 提出了“没有调查, 没有发言权”和“中国革命斗争的胜利要靠中国同志了解中国情况”的重要思想, 表现了毛泽东开辟新道路、创造新理论的革命首创精神。故本题正确选项为 CD.\n\n因此答案为:C D", "task_type": {"major": ["试题"], "minor": ["考研", "选择题"]}, "domain": ["政治"], "metadata": "year:2023, score:2", "answer_from": "human", "human_verified": false, "copyright": "暂无版权及作者信息"} +{"instruction": "党的十一届三中全会标志着中国共产党重新确立了马克思主义的思想路线、政治路线、组织路线, 实现了新中国成��以来党的历史上具有深远意义的伟大转折, 开启了我国改革开放和社会主义现代化建设的新时期。全会作出实行改革开放的历史性决策, 是基于 ()\nA. 对党的和国家前途命运的深刻把握\nB. 对社会主义革命和建设实践的深刻总结\nC. 对时代潮流的深刻洞察\nD. 对人民群众期盼和需要的深刻体悟", "input": "", "output": "本题考查十一届三中金会。党的十一届三中全会的胜利召开,标志着粉碎“四人帮”后党和国家工作在律面中前进的局面的结束。全会作出实行改革开放的历史性决\n策, 是基于对党和国家前途命运的深刻把握, 是基于对社会主义革命和建设实践的深刻总结, 是基于对时代潮流的深刻洞察, 是基于对人民群众期盼和需要的深刻体悟。改革开放是中国共产党的一次伟大觉醒, 正是这个伟大觉醒, 孕育了党从理论到实践的伟大创造。故本题正确选项为 $A B C D$ 。\n\n因此答案为:A B C D", "task_type": {"major": ["试题"], "minor": ["考研", "选择题"]}, "domain": ["政治"], "metadata": "year:2023, score:2", "answer_from": "human", "human_verified": false, "copyright": "暂无版权及作者信息"} +{"instruction": "社会主义道德是崭新类型的道德, 与以往的社会道德形态相比, 社会主义道德具有显著的先进性特征。先进性主要体现在:\nA. 它是对人类优秀道德资源的批判继承和创新发展\nB. 它克服了以往阶级社会道德的片面性和局限性\nC. 它是调节社会一切行为规范的准则\nD. 它是社会主义经济基础的反映\n答案: $A B D$ : 与以往社会的道德形态相比, 社会主义道德具有显著的先进性特征。这种先进性主要体现在: 首先, 社会主义道德是社会主义经济基础的反映。在以生产资料公有制为主体的社会主义社会, 广大人民不仅在政治上实现了当家作主, 而且在道德上实现了由被动到主动的转变。其次, 社会主义道德是对人类优秀道德资源的批判继承和创新发展。最后,社会主义道德克服了以往阶级社会道德的片面性和局限性,坚持以为人民服务为核心,坚持以集体主义为原则, 展现出真实而强大的道义力量。所以 $\\mathrm{ABD}$ 三项正确, $\\mathrm{C}$ 项表述不正确。", "input": "", "output": "本题考查社会主义道德是崭新类型的道德。与以往社会的道德形态相比, 社会主义道德具有显著的先进性特征。这种先进性主要体现在: 首先, 社会主义道德是社会主义经济基础的反映。其次, 社会主义道德是对人类优秀道德资源的批判继承和创新发展。最后, 社会主义道德客服了以往阶级社会道德的片面性和局限性, 坚持以为人民服务为核心, 坚持以集体主义为原则, 展现出真实而强大的道义力量。选项 C 说法不正确。故本题正确选项为 $A B D$ 。\n\n因此答案为:A B D", "task_type": {"major": ["试题"], "minor": ["考研", "选择题"]}, "domain": ["政治"], "metadata": "year:2023, score:2", "answer_from": "human", "human_verified": false, "copyright": "暂无版权及作者信息"} +{"instruction": "法律义务是指由一定的社会物质生活条件所志愿的社会责任, 是保证法律所规定的义务人按照权利人要求从事一定行为或不从事一定行为以满足权利人权益的法律手段。正确的有:\nA. 法律义务必须依法设定\nB. 法律义务是一成不变的\nC. 法律义务源于现实需要\nD. 法律义务是义务人自主实施的行为", "input": "", "output": "本题考查法律义务。法律义务具有以下四个特点: 其一, 法律义务是历史的。其内容和履行方式是随着经济社会的发展和人权保障的进步而不断调整和变化的。其二, 法律义务源于现实需要。其三, 法律义务必须依法设定。其四, 法律义务可能发生变化。故本题正确选项为 $\\mathrm{AC}$ 。\n\n因此答案为:A C", "task_type": {"major": ["试题"], "minor": ["考研", "选择题"]}, "domain": ["政治"], "metadata": "year:2023, score:2", "answer_from": "human", "human_verified": false, "copyright": "暂无版权及作者信息"} +{"instruction": "32.2022 年 8 月 10 日,国务院台湾事务办公室,国务院新闻办公室发表《台湾问题与新时代中国统一事业》白皮书,该白皮书发表有利于 ()?\nA. 揭批台独分裂势力和外部势力勾连挑畔, 企图损害中国主权和领土完整, 阻挠破坏中国统一进程和恶劣言行\nB. 最大限度争取两岸同胞,国际社会的理解和支持\nC. 展现中国共产党和中国政府愿继续以最大诚意, 尽最大努力争取和平统一的立场和态度\nD. 提振全党全国各族人民矢志追求国家统一的精气神, 增强岛内和海外反 “独” 促统力量的信心和勇气", "input": "", "output": "选项 ABCD 都符合白皮书的意义, 全选。\n\n因此答案为:A B C D", "task_type": {"major": ["��题"], "minor": ["考研", "选择题"]}, "domain": ["政治"], "metadata": "year:2023, score:2", "answer_from": "human", "human_verified": false, "copyright": "暂无版权及作者信息"} +{"instruction": "2022 年 11 月 15 日二十国集团领导人第十七次峰会在印度尼西亚巴厘岛举行, 国家主席习近平出席并发表题为《共迎时代挑战,共建美好未来》的重要讲话,倡议推动( )?。\nA. 更具竞争的全球发展\nB. 更加包容的全球发展\nC 更加普惠的全球发展\nD. 更有韧性的全球发", "input": "", "output": "本题为时政题: 2022 年 11 月 15 日二十届集团领导人第十七次峰会在印度尼西亚巴厘岛举行, 习近平主席出席并发表重要讲话。习近平强调, 第一, 我们要推动更加包容的全球发展。第二, 我们要推动更加普惠的全球发展。第三, 我们要推动更有韧性的全球发展。故本题正确选项为 $\\mathrm{BCD}$ 。\n\n因此答案为:B C D", "task_type": {"major": ["试题"], "minor": ["考研", "选择题"]}, "domain": ["政治"], "metadata": "year:2023, score:2", "answer_from": "human", "human_verified": false, "copyright": "暂无版权及作者信息"} +{"instruction": "《论灵魂》是一部古老的著作, 它总结了当时西方人对心理的认识, 其作者是\nA. 柏拉图\nB.苏格拉底\nC. 亚里士多德\nD.笛卡尔", "input": "", "output": "亚里士多德对灵魂的实质、灵魂与肉体的关系、灵魂的种类与功能等问题从理论上进行了探讨。他的著作《论灵魂》是历史上第一部论述各种心理现象的著作。 \n \n\n因此答案为:C", "task_type": {"major": ["试题"], "minor": ["考研", "选择题"]}, "domain": ["心理学"], "metadata": "year:2022, score:2", "answer_from": "human", "human_verified": false, "copyright": "暂无版权及作者信息"} +{"instruction": "在严格控制条件下, 使用 fMRI 技术研究不同任务引起的大脑皮层区域的激活情况, 这种研究方法属于\nA.测验法\nB. 观察法\nC. 调查法\nD. 实验法", "input": "", "output": "实验法是在严格控制条件下对某种心理现象进行观察的方法。\n\n因此答案为:D", "task_type": {"major": ["试题"], "minor": ["考研", "选择题"]}, "domain": ["心理学"], "metadata": "year:2022, score:2", "answer_from": "human", "human_verified": false, "copyright": "暂无版权及作者信息"} +{"instruction": "下列心理现象中, 可以用视觉系统的侧抑制作用解释的是\nA.马赫带现象\nB. 普肯耶现象\nC. 闪光融合\nD. 视觉掩蔽", "input": "", "output": "侧抑制是指相邻的感受器之间能够互相抑制的现象。侧抑制作用较好地解释了一种重要的视觉现象一一马赫带。所谓马赫带是指人们在明暗变化的边界上, 常常在亮区看到一条更亮的光带,而在暗区看到一条更暗的线条。 \n \n\n因此答案为:A", "task_type": {"major": ["试题"], "minor": ["考研", "选择题"]}, "domain": ["心理学"], "metadata": "year:2022, score:2", "answer_from": "human", "human_verified": false, "copyright": "暂无版权及作者信息"} +{"instruction": "韦伯定律适用的刺激强度属于\nA. 低强度\nB. 中等强度\nC. 高强度\nD. 各种强度", "input": "", "output": "韦伯定律虽然揭示了感觉的某些规律, 但它只适用于刺激的中等强度。换句话说只有使用中等强度的刺激, 韦伯分数才是一个常数。 \n \n\n因此答案为:B", "task_type": {"major": ["试题"], "minor": ["考研", "选择题"]}, "domain": ["心理学"], "metadata": "year:2022, score:2", "answer_from": "human", "human_verified": false, "copyright": "暂无版权及作者信息"} +{"instruction": "在社会生活中, 人们常会表现出自利偏差 (self-serving bias) 能对这种偏差作出较好解释的理论是\nA. 社会学习理论\nB.印象管理理论\nC.社会交换理论\nD. 行为归因理论", "input": "", "output": "归因是指人们推论他人的行为或态度之原因的过程。维纳系统提出了动机的归因理论, 说明了归因的维度及归因对成功与失败行为的影响。内因和外因的区是归因的维度之一。归因理论常见的现象包括基本归因错误和自利偏差。自利偏差也称为自我服务偏见, 当人们加工和自\n我有关的信息时, 会出现一种潜在的偏见: 一边为自己的失败开脱, 一边欣然接受成功的荣耀, 在多数情况下, 人们觉得自己比别人好。自利偏差把成功的原因归于内部 (能力或努力), 把失败的原因归于外部 (运气或任务难度)。因此自利偏差可以用行为归因理论进行解释。 A 项, 社会学习理论指通过观察他们生活中重要人物的行为而学得社会行为, 这些观察以心理表象或其他符号表征的形式储存在大脑中, 来帮助他们模仿行为。B 项, 印象管理是指一个人通过一定的方法去影响别人对于自己的印象, 使别人所形成的印象符合自己期望的过程。C 项, 社会交换理论认为, ���们所做的许多事源于我们对利益和成本的衡量, 我们希望获得最大化的利益和付出最小化的代价。按照这一理论, 人们在助人行为中也试图追求最大的收益和最小的付出。\n\n因此答案为:D", "task_type": {"major": ["试题"], "minor": ["考研", "选择题"]}, "domain": ["心理学"], "metadata": "year:2022, score:2", "answer_from": "human", "human_verified": false, "copyright": "暂无版权及作者信息"} +{"instruction": "下列选项中, 不属于人耳对声音进行定向的物理线索的是\nA. 时间差\nB. 强度差\nC. 相位差\nD. 音色差", "input": "", "output": "听觉方位定向能力主要来自两耳听觉的差异, 包括时间差、强度差和相位差。\n\n因此答案为:D", "task_type": {"major": ["试题"], "minor": ["考研", "选择题"]}, "domain": ["心理学"], "metadata": "year:2022, score:2", "answer_from": "human", "human_verified": false, "copyright": "暂无版权及作者信息"} +{"instruction": "根据维纳的归因理论, 运气属于\nA. 内部、稳定、不可控因素\nB. 内部、不稳定、可控因素\nC. 外部、稳定、不可控因素\nD. 外部、不稳定、不可控因素", "input": "", "output": "维纳等人系统地提出了动机的归因理论, 证明了成功和失败的因果归因是成就活动过程的中心要素。维纳也把成就行为的归因划分为内部原因和外部原因, 同时把稳定性作为一个新的维度,把行为原因分为稳定的和不稳定的。运气属于外部的、不可控的、不稳定的。\n\n因此答案为:D", "task_type": {"major": ["试题"], "minor": ["考研", "选择题"]}, "domain": ["心理学"], "metadata": "year:2022, score:2", "answer_from": "human", "human_verified": false, "copyright": "暂无版权及作者信息"} +{"instruction": "下列关于个体进入催眠状态的表述中, 错误的是\nA. 进入睡眠状态\nB. 主动性反应降低\nC. 受暗示程度较高\nD. 与在清醒状态下记录到的脑电波相似", "input": "", "output": "催眠是另一种意识状态。被试进入催眠状态后好像是睡着了, 但其实并没有入睡,催眠时的脑电特征与个体在清醒状态时是相似的。 \n \n\n因此答案为:A", "task_type": {"major": ["试题"], "minor": ["考研", "选择题"]}, "domain": ["心理学"], "metadata": "year:2022, score:2", "answer_from": "human", "human_verified": false, "copyright": "暂无版权及作者信息"} +{"instruction": "“朝辞白帝彩云间, 千里江陵一日还; 两岸猿声啼不住, 轻舟已过万重山”, 读者根据诗中的描述在脑中出现的形象是\nA. 记忆表象\nB.遗觉表象\nC.想象表象\nD.感觉表象", "input": "", "output": "想象表象是在头脑中对记忆形象进行加工改组后形成的新形象, 根据诗中的描述形成的形象属于想象表象。 \n \n\n因此答案为:C", "task_type": {"major": ["试题"], "minor": ["考研", "选择题"]}, "domain": ["心理学"], "metadata": "year:2022, score:2", "answer_from": "human", "human_verified": false, "copyright": "暂无版权及作者信息"} +{"instruction": "小李办公桌上有一颗螺丝松了, 在找不到螺丝刀的情况下, 他并没有想到用桌子上的一把水果刀来拧紧螺丝,这反映的影响问题解决的因素是\nA. 功能固着\nB. 原型启发\nC. 动机水平\nD. 知识表征方式", "input": "", "output": "人们把某种功能赋予某种物体的倾向称为功能固着。小李只想到水果刀的功能是切水果, 没意识到还能用来拧紧螺丝。 \n \n\n因此答案为:A", "task_type": {"major": ["试题"], "minor": ["考研", "选择题"]}, "domain": ["心理学"], "metadata": "year:2022, score:2", "answer_from": "human", "human_verified": false, "copyright": "暂无版权及作者信息"} +{"instruction": "人们通过发音器官或手的活动, 把所要表述的想法说出来、写出来或用手势表达出来, 这一过程称为\nA. 语言理解\nB. 语言产生\nC. 言语表征\nD. 言语理解", "input": "", "output": "语言产生也叫语言表达, 是指人们通过发音器官或手的活动把所要表达的思想说出来、写出来或用手势表达出来的过程。 \n \n\n因此答案为:B", "task_type": {"major": ["试题"], "minor": ["考研", "选择题"]}, "domain": ["心理学"], "metadata": "year:2022, score:2", "answer_from": "human", "human_verified": false, "copyright": "暂无版权及作者信息"} +{"instruction": "与单词的视觉记忆有关, 并有助于实现视觉与听觉跨通道联合的皮层区域是\nA. 布洛卡区\nB. 威尔尼克区\nC. 艾克斯勒区\nD. 角回", "input": "", "output": "角回是语言视觉中枢, 该区负责书面语言和口语之间的相互转化。\n\n因此答案为:D", "task_type": {"major": ["试题"], "minor": ["考研", "选择题"]}, "domain": ["心理学"], "metadata": "year:2022, score:2", "answer_from": "human", "human_verified": false, "copyright": "暂无版权及作者信息"} +{"instruction": "下列选项中, 支持詹姆斯-兰格理���的是\nA. 脊髓受损伤患者的情绪体验强度降低\nB.药物引起的生理变化不会导致情绪的产生\nC. 机体上的生理变化在各种情绪状态下差异不大\nD. 植物性神经系统支配的机体变化缓慢", "input": "", "output": "詹姆斯-兰格理论强调情绪的产生是植物性神经系统活动的产物, 认为情绪刺激引起身体的生理反应, 而生理反应进一步导致情绪体验的产生。“脊髓受损伤患者的情绪体验强度降低” 支持詹姆斯-兰格理论。 \n \n\n因此答案为:A", "task_type": {"major": ["试题"], "minor": ["考研", "选择题"]}, "domain": ["心理学"], "metadata": "year:2022, score:2", "answer_from": "human", "human_verified": false, "copyright": "暂无版权及作者信息"} +{"instruction": "根据艾克曼 (P.EKman) 的研究, 人类的基本情绪不包括\nA.愤怒\nB. 悔恨\nC. 恐惧\nD. 惊奇", "input": "", "output": "艾克曼在 1972 年提出的基本情感愤怒、厌恶、恐惧、快乐、悲伤、惊讶。悔恨不属于人类的基本情绪。 \n \n\n因此答案为:B", "task_type": {"major": ["试题"], "minor": ["考研", "选择题"]}, "domain": ["心理学"], "metadata": "year:2022, score:2", "answer_from": "human", "human_verified": false, "copyright": "暂无版权及作者信息"} +{"instruction": "在吉尔福特的三维结构模型中, 智力的三个维度是\nA.内容、操作、产物\nB. 认知、语义、类别\nC. 记忆、关系、符号\nD. 行为、评价、蕴涵", "input": "", "output": "吉尔福特认为智力可以区分为三个维度:内容、操作、产物。 \n \n\n因此答案为:A", "task_type": {"major": ["试题"], "minor": ["考研", "选择题"]}, "domain": ["心理学"], "metadata": "year:2022, score:2", "answer_from": "human", "human_verified": false, "copyright": "暂无版权及作者信息"} +{"instruction": "对个人服从权威现象进行了经典实验研究的心理学家是\nA.阿希\nB.津巴多\nC. 谢里夫\nD. 米尔格拉姆", "input": "", "output": "服从是在外界压力的影响下被迫发生的。其经典实验是米尔格拉姆的电击实验。\n\n\n因此答案为:D", "task_type": {"major": ["试题"], "minor": ["考研", "选择题"]}, "domain": ["心理学"], "metadata": "year:2022, score:2", "answer_from": "human", "human_verified": false, "copyright": "暂无版权及作者信息"} +{"instruction": "按照社会渗透理论,与一个刚认识的人进行交往时应遵守的规范是\nA.相互性规范\nB. 一致性规范\nC.互补性规范\nD. 连续性规范", "input": "", "output": "社会心理学家奥尔特曼和泰勒用社会渗透理论解释人与人相互关系的水平。他们认为亲密的人际关系是在一个人 “渗透” 过个人的表面特征, 并逐步了解到一个人内部自我的过程中发展起来的。良好的人际关系, 是随着人们自我表露逐渐增加而发展的。随着对一个人的接纳性和信任感越来越高, 人们也越来越多地表露自我, 同时也要求别人也越来越多地表露他们自己。因此人们在自我表露时,遵循这种相互性原则。\n\n因此答案为:A", "task_type": {"major": ["试题"], "minor": ["考研", "选择题"]}, "domain": ["心理学"], "metadata": "year:2022, score:2", "answer_from": "human", "human_verified": false, "copyright": "暂无版权及作者信息"} +{"instruction": "下列选项中,与罹患心脏病相关最高的人格类型是\nA.A 型人格\nB.B 型人格\nC.C 型人格\nD.T 型人格", "input": "", "output": "A 型人格的主要特点是: 性情急躁、缺乏耐性。他们的成就欲高、上进心强、有苦干精神、工作投入、做事认真负责、时间紧迫感强、富有竞争意识、外向、动作敏捷、说话快、生活常处于紧张状态, 但办事夘忙、社会适应性差, 属不安定型人格。具有这种人格特征的人易患冠心病。 \n \n\n因此答案为:A", "task_type": {"major": ["试题"], "minor": ["考研", "选择题"]}, "domain": ["心理学"], "metadata": "year:2022, score:2", "answer_from": "human", "human_verified": false, "copyright": "暂无版权及作者信息"} +{"instruction": "人在自觉确立目标、调节其行为方式与控制水平时表现出来的性格特征是\nA.态度特征\nB. 理智特征\nC. 情绪特征\nD. 意志特征", "input": "", "output": "性格的意志特征是指人在自觉调节自己行为的方式与控制水平、目标明确程度以及在处理紧急问题方面表现出来的性格差异。\n\n因此答案为:D", "task_type": {"major": ["试题"], "minor": ["考研", "选择题"]}, "domain": ["心理学"], "metadata": "year:2022, score:2", "answer_from": "human", "human_verified": false, "copyright": "暂无版权及作者信息"} +{"instruction": "1922 年出版的《衰老: 人的后半生》一书的作者是\nA. 高尔顿\nB. 普莱尔\nC. 卡特尔\nD. 霍尔", "input": "", "output": "霍尔是最早正式研究老年心理的心理学家, 于 1922 年出版了《衰老: 人的后半生》一书。\n\n因此答案为:D", "task_type": {"major": ["试题"], "minor": ["考研", "选择题"]}, "domain": ["心理学"], "metadata": "year:2022, score:2", "answer_from": "human", "human_verified": false, "copyright": "暂无版权及作者信息"} +{"instruction": "艾里克森认为人格发展阶段中可能存在 “合法延缓期”,这种现象出现在\nA.幼儿期\nB. 青年期\nC.中年期\nD. 老年期", "input": "", "output": "艾里克森认为, 青年可以一时合法地延缓所必须承担的社会责任和义务, 因此, 青年期又被成为心理的合法延缓期。 \n \n\n因此答案为:B", "task_type": {"major": ["试题"], "minor": ["考研", "选择题"]}, "domain": ["心理学"], "metadata": "year:2022, score:2", "answer_from": "human", "human_verified": false, "copyright": "暂无版权及作者信息"} +{"instruction": "某幼儿在与母亲独处时, 会积极主动地探索环境, 与母亲分离后, 明显表现出不开心, 当母亲返回后,婴儿会欣赏并寻求身体上的接触,该婴儿所属的依恋类型是\nA. 回避型\nB. 反抗型\nC. 迟缓型\nD. 安全型", "input": "", "output": "安全型依恋的儿童, 母亲在场时婴儿感到足够的安全, 母亲离开时, 婴儿会表现苦恼, 母亲回来时, 容易抚慰而平静。\n\n\n因此答案为:D", "task_type": {"major": ["试题"], "minor": ["考研", "选择题"]}, "domain": ["心理学"], "metadata": "year:2022, score:2", "answer_from": "human", "human_verified": false, "copyright": "暂无版权及作者信息"} +{"instruction": "游戏是儿童认识新的复杂客体和事件的途径, 持这种观点的游戏理论是\nA.复演论\nB. 认知动力说\nC.精神分析理论\nD. 觉醒-寻求理论", "input": "", "output": "皮亚杰认为游戏是儿童认识新的复杂客体和事件的方法, 是巩固和扩大概念、技能的方法, 是使思维和行动结合起来的方法。\n\n因此答案为:B", "task_type": {"major": ["试题"], "minor": ["考研", "选择题"]}, "domain": ["心理学"], "metadata": "year:2022, score:2", "answer_from": "human", "human_verified": false, "copyright": "暂无版权及作者信息"} +{"instruction": "6-8 个月的婴儿之间的交往主要指向玩具或物品, 并且大部分社交行为由单方面发起, 这说明其同伴交往处于\nA. 以客体为中心阶段\nB.简单交往阶段\nC.互补性交往阶段\nD. 同伴游戏阶段", "input": "", "output": "早期同伴交往发展有三个阶段, 第一个阶段是以客体为中心, 这个阶段的特点是: 6-8 个月的婴儿的交往更多地集中在玩具或物品上,而不是对方本身。 \n \n\n因此答案为:A", "task_type": {"major": ["试题"], "minor": ["考研", "选择题"]}, "domain": ["心理学"], "metadata": "year:2022, score:2", "answer_from": "human", "human_verified": false, "copyright": "暂无版权及作者信息"} +{"instruction": "一般认为由具体形象思维过渡到抽象逻辑思维的关键年龄是\nA.4-5 岁\nB.7-8 岁\nC.10-11 岁\nD.13-14 岁", "input": "", "output": "由具体形象思维到抽象逻辑思维的过渡存在着明显的关键年龄, 这个关键年龄在四年级 $(10-11$ 岁)。\n\n因此答案为:C", "task_type": {"major": ["试题"], "minor": ["考研", "选择题"]}, "domain": ["心理学"], "metadata": "year:2022, score:2", "answer_from": "human", "human_verified": false, "copyright": "暂无版权及作者信息"} +{"instruction": "提出 “学生掌握学科基本结构的最好办法是发现法” 的心理学家是\nA. 桑代克\nB. 维果斯基\nC. 皮亚杰\nD. 布鲁纳", "input": "", "output": "布鲁纳认为学习就是要将学科的基本结构变成学生脑中的认知结构, 他提倡发现学习。 \n \n\n因此答案为:D", "task_type": {"major": ["试题"], "minor": ["考研", "选择题"]}, "domain": ["心理学"], "metadata": "year:2022, score:2", "answer_from": "human", "human_verified": false, "copyright": "暂无版权及作者信息"} +{"instruction": "在连续强化条件下, 新行为反应的特点是\nA. 建立快, 消退也快\nB. 建立慢,消退也慢\nC. 建立快, 消退慢\nD. 建立慢, 消退快", "input": "", "output": "连续强化指给予每个反应强化, 可以快速学习新行为, 但习得行为也会快速消失,例如表扬和恭维。 \n \n\n因此答案为:A", "task_type": {"major": ["试题"], "minor": ["考研", "选择题"]}, "domain": ["心理学"], "metadata": "year:2022, score:2", "answer_from": "human", "human_verified": false, "copyright": "暂无版权及作者信息"} +{"instruction": "某一受过处分的学生, 因有良好的行为表现而被撤销处分, 此后该生的良好行为表现大大增加,这一现象体现了\nA. 正强化作用\nB.负强化作用\nC.惩罚作用\nD. 消退作用", "input": "", "output": "负强化指消除厌恶刺激以增强反应频率, 题干中因为撤销处分 (厌恶刺激) 使得学\n生良好行为增加 (反应频率提升)。 \n \n\n因此答案为:B", "task_type": {"major": ["试题"], "minor": ["考研", "选择题"]}, "domain": ["心理学"], "metadata": "year:2022, score:2", "answer_from": "human", "human_verified": false, "copyright": "暂无版权及作者信息"} +{"instruction": "学生在掌握 “亲社会行为” 概念后, 在新冠疫情防控期间, 进一步认识到 “居家隔离” 和 “佩戴口罩” 也属于 “亲社会行为”, 这一学习过程属于\nA. 相关类属学习\nB. 派生类属学习\nC. 并列结合学习\nD.归纳概括学习", "input": "", "output": "派生类属指新的学习内容仅仅是学生已有的、包容面较广的命题的一个例证, 本题佩戴口罩就是亲社会行为的一个例子。 \n \n\n因此答案为:B", "task_type": {"major": ["试题"], "minor": ["考研", "选择题"]}, "domain": ["心理学"], "metadata": "year:2022, score:2", "answer_from": "human", "human_verified": false, "copyright": "暂无版权及作者信息"} +{"instruction": "掌握目标定向的学生倾向于将学业失败归因于\nA. 能力高低\nB. 努力程度\nC. 运气好坏\nD.任务难度", "input": "", "output": "持有掌握目标学生学习是为了自己的成长, 自信心比较强, 往往将自己的失败归因于内部、不稳定、可控的努力程度,将成功归因于自己的能力。\n\n因此答案为:B", "task_type": {"major": ["试题"], "minor": ["考研", "选择题"]}, "domain": ["心理学"], "metadata": "year:2022, score:2", "answer_from": "human", "human_verified": false, "copyright": "暂无版权及作者信息"} +{"instruction": "下列选项中,指向学习过程的内部动机成分是\nA. 认知内驱力\nB. 社会交往内驱力\nC.亲和内驱力\nD. 自我提高内驱力", "input": "", "output": "奥苏贝尔将学习动机分为认知内驱力、自我提高内驱力、附属内驱力, 其中认知内驱力属于内部动机。\n\n因此答案为:A", "task_type": {"major": ["试题"], "minor": ["考研", "选择题"]}, "domain": ["心理学"], "metadata": "year:2022, score:2", "answer_from": "human", "human_verified": false, "copyright": "暂无版权及作者信息"} +{"instruction": "依据心理学论文写作规范, 实验结果的常见表格形式是\nA.三线表\nB.四线表\nC.三格表\nD.四格表", "input": "", "output": "心理学论文实验结果通常用三线表呈现。 \n \n\n因此答案为:A", "task_type": {"major": ["试题"], "minor": ["考研", "选择题"]}, "domain": ["心理学"], "metadata": "year:2022, score:2", "answer_from": "human", "human_verified": false, "copyright": "暂无版权及作者信息"} +{"instruction": "在信号检测实验中, 每次向被试呈现 2-8 个刺激, 其中只有 1 个信号, 其余均为噪音, 要求被试判断哪个是信号, 最后根据实验次数和被试正确的判断次数计算被试的辨别力, 这种方法是\nA.迫选法\nB. 有无法\nC. 评价法\nD. 分段法", "input": "", "output": "A项迫选法一般每次给被试呈现刺激的数目为 $2 \\sim 8$ 个, 其中只有一个是信号, 其余的为噪声。当呈现多个刺激后, 只要求被试回答哪个是信号。被试回答是信号的依据是他对刺激量差异的感觉, 如果感觉到某一刺激与其他刺激有显著差异, 就可将其判断为信号。 B 项, 有无法要求事先选定 $\\mathrm{SN}$ 刺激和 N 刺激, 并规定 $\\mathrm{SN}$ 和 N 出现的概率, 然后以随机方式呈现 $\\mathrm{SN}$ 和 N, 要求被试回答, 刚才的刺激是 $\\mathrm{SN}$ 还是 N 。根据对被试回答的结果计算 $\\mathrm{P}(\\mathrm{y} / \\mathrm{SN})$ 和 $\\mathrm{P}(\\mathrm{y} / \\mathrm{N})$, 由此计算 $\\mathrm{d}^{\\prime}$ 和 $\\beta$ 。C 项, 评价法的刺激呈现方式同有无法一样, 但先定概率与奖惩办法随实验而定, 要求被试不仅回答有无刺激, 还要用概率(不同评价等级)来表示对回答的确信程度, 在这种实验中,被试会用到多个评价标准。D 项,不是信号检测论的基本方法。\n\n因此答案为:A", "task_type": {"major": ["试题"], "minor": ["考研", "选择题"]}, "domain": ["心理学"], "metadata": "year:2022, score:2", "answer_from": "human", "human_verified": false, "copyright": "暂无版权及作者信息"} +{"instruction": "沃(N.waugh)和诺曼(D.Norman)在短时记忆研究中, 分别考察间隔时间和间隔数字对遗忘的\n作用, 以便将痕迹消退和干扰两个因素分开, 该研究采用的方法是\nA. 节省法\nB. 重学法\nC.提示法\nD. 探测法", "input": "", "output": "沃(N.waugh)和诺曼(D.Norman)发展出探测法, 将痕迹消退和干扰两个因素巧妙分离开来。他们让被试听若干个数字组成的数字序列, 在数字序列呈现完毕后, 伴随着一个声音信号呈现一个探测数字, 这个探测数字曾经在前面出现过一次, 被试的任务就是回忆在探测数字后边是什么数字。在实验中他们采用了两种速度呈现数字, 一种是快速的, 一种是慢速的, 这样可以在间隔数字不变的情况下,改变间隔时间,从而使时间和干扰信息这两种因素分离开来。\n\n因此答案为:D", "task_type": {"major": ["试题"], "minor": ["考研", "选择题"]}, "domain": ["心理学"], "metadata": "year:2022, score:2", "answer_from": "human", "human_verified": false, "copyright": "暂无版权及作者信息"} +{"instruction": "研究 “吸烟与肺癌之间关系” 的适宜设计是\nA. 回溯设计\nB.匹配设计\nC. 重复设计\nD. 拉丁方设计", "input": "", "output": "事后回溯设计是在事件发生之后, 研究者系统地搜集相关资料, 经过分析、比较、推理、判断, 进而发现现象之间的前因后果关系, 获得一定研究结论的方法。题干研究 “吸烟与肺癌之间关系”, 只能把肺癌患者与正常人进行对比分析, 不能系统的操作自变量和规划因变量, 是一种事后回溯设计。 \n \n\n因此答案为:A", "task_type": {"major": ["试题"], "minor": ["考研", "选择题"]}, "domain": ["心理学"], "metadata": "year:2022, score:2", "answer_from": "human", "human_verified": false, "copyright": "暂无版权及作者信息"} +{"instruction": "在某实验中, 有三种情绪(积极、消极和中性)图片, 共有 6 名被试参与实验, 其实验设计方案如下表所示, 该研究设计属于\n 6 名被试分配表\n\n\\begin{center}\n\\begin{tabular}{ccc}\n\\hline\n积极情绪图片 & 消极情绪图片 & 中性情绪图片 \\\\\n$\\mathrm{O} 1$ & $\\mathrm{O} 1$ & $\\mathrm{O} 1$ \\\\\n$\\mathrm{O} 2$ & $\\mathrm{O} 2$ & $\\mathrm{O} 2$ \\\\\n$\\mathrm{O} 3$ & $\\mathrm{O} 3$ & $\\mathrm{O} 3$ \\\\\n$\\mathrm{O} 4$ & $\\mathrm{O} 4$ & $\\mathrm{O} 4$ \\\\\n$\\mathrm{O} 5$ & $\\mathrm{O} 5$ & $\\mathrm{O} 5$ \\\\\n$\\mathrm{O} 6$ & $\\mathrm{O} 6$ & $\\mathrm{O} 6$ \\\\\n\\hline\n\\end{tabular}\n\\end{center}\n\nA.单因素被试间设计\nB. 单因素被试内设计\nC. 多因素被试间设计\nD. 多因索被试内设计", "input": "", "output": "根据题干, 被试观看三种不同的情绪图片, 自变量是 “情绪”, 由三个水平 (积极、消极、中立),是单因素设计。根据被试分配表,每个被试均参与了 “积极” “消极” 和 “中立”三种水平的实验, 是被试内设计。综合, 则为单因素被试内设计。\n\n因此答案为:B", "task_type": {"major": ["试题"], "minor": ["考研", "选择题"]}, "domain": ["心理学"], "metadata": "year:2022, score:2", "answer_from": "human", "human_verified": false, "copyright": "暂无版权及作者信息"} +{"instruction": "被试固有的和习得的差异是影响研究内部效度的重要因素, 这些因素属于\nA. 前摄历史因素\nB. 后摄历史因素\nC. 选择因素\nD. 成熟因素", "input": "", "output": "前摄历史因素包括被试固有的和习得的差异, 成熟因素指在实验过程中随着时间的延续, 被试身心发生变化, 如变得较为成熟、疲倦、饥渴或对实验丧失兴趣等。 \n \n\n因此答案为:A", "task_type": {"major": ["试题"], "minor": ["考研", "选择题"]}, "domain": ["心理学"], "metadata": "year:2022, score:2", "answer_from": "human", "human_verified": false, "copyright": "暂无版权及作者信息"} +{"instruction": "实验研究结果能够推广到其它总体、变量条件、时间和情境的程度、代表着研究的\nA. 内部效度\nB. 外部效度\nC. 统计效度\nD. 结构效度", "input": "", "output": "外部效度指实验结果能够普遍推论到样本的总体和其他同类现象中去的程度, 即实验结果的普遍代表和适用性。 \n \n\n因此答案为:B", "task_type": {"major": ["试题"], "minor": ["考研", "选择题"]}, "domain": ["心理学"], "metadata": "year:2022, score:2", "answer_from": "human", "human_verified": false, "copyright": "暂无版权及作者信息"} +{"instruction": "从 100 人的总体中选取 25 人作为样本, 先随机抽出第 1 名被试, 然后每隔 4 名抽取一名被试以此类推, 直至抽满 25 名被试, 这种抽样方法是\nA. 分层随机抽样\nB. 整群随机抽样\nC. 系统随机抽样\nD. 简单随机抽样", "input": "", "output": "系统随机抽样也称机械随机抽样或等距随机抽样, 即将总体单位按某一标志 (如时间)排序,然后按一定间隔来随机抽取样本单位。题干先确定 “第 1 名” 被试,然后间隔一定距离抽样, 是一种系统随机抽样。\n\n因此答案为:C", "task_type": {"major": ["试题"], "minor": ["考研", "选择题"]}, "domain": ["心理学"], "metadata": "year:2022, score:2", "answer_from": "human", "human_verified": false, "copyright": "暂无版权及作者信息"} +{"instruction": "在同一感觉通道下, 选择反应时与选取数目的关系通常是\nA. 选择反应时与选项数目成正比关系\nB. 选择反应时与选项数目成反比关系\nC. 选择反应时与选项数目的对数成正比关系\nD. 选择反应时与选项数目的对数成反比关系", "input": "", "output": "在同一通道中, 选择反应时和选择数目的对数成正比, 即 $\\mathrm{RT}=\\operatorname{lgN}$ 。 \n \n\n因此答案为:C", "task_type": {"major": ["试题"], "minor": ["考研", "选择题"]}, "domain": ["心理学"], "metadata": "year:2022, score:2", "answer_from": "human", "human_verified": false, "copyright": "暂无版权及作者信息"} +{"instruction": "用刺激或指导语来引导被试注意一个明确的输入源, 然后将对这一输入源的加工和对其它输入源的加工进行比较, 这种注意研究范式是\nA.整体局部范式\nB.过程分离范式\nC.提示范式\nD. 过滤范式", "input": "", "output": "提示范式是用刺激或指导语来引导被试注意一个明确的输入源, 然后比较对提示输入源和对其它输入源的加工。\n\n因此答案为:C", "task_type": {"major": ["试题"], "minor": ["考研", "选择题"]}, "domain": ["心理学"], "metadata": "year:2022, score:2", "answer_from": "human", "human_verified": false, "copyright": "暂无版权及作者信息"} +{"instruction": "下列仪器设备中, 能改变双眼视差的是\nA.动景盘\nB. 闪光融合仪\nC. 实体镜\nD. “视崖” 实验装置", "input": "", "output": "实体镜是一种用平面镜或棱镜、木架、遮挡板等把左眼和右眼的视线分开, 使左眼只看见左图, 右眼只看见右图, 从而形成立体视觉图的器具。实体镜中的两张图片是根据双眼视差原理制成的,内容虽然一样,但略有偏斜,将其放入实体镜上观察,即能看到立体图像。 \n \n\n因此答案为:C", "task_type": {"major": ["试题"], "minor": ["考研", "选择题"]}, "domain": ["心理学"], "metadata": "year:2022, score:2", "answer_from": "human", "human_verified": false, "copyright": "暂无版权及作者信息"} +{"instruction": "在一个 $2 \\times 2$ 的实验研究中, 30 名被试中的 15 名接受其中 2 种实验处理, 另外 15 名被试接受另外 2 种实验处理, 该实验设计是\nA.ABA 设计\nB.混合设计\nC.匹配设计\nD. 被试间设计", "input": "", "output": "假设四种处理操作为 $\\mathrm{A} 1 \\mathrm{~B} 1, \\mathrm{~A} 1 \\mathrm{~B} 2, \\mathrm{~A} 2 \\mathrm{~B} 1, \\mathrm{~A} 2 \\mathrm{~B} 2$, 其中第一组 15 人接受前两种,第二组 15 人接受后两种, 即在 A 因素上属于被试间设计, B 因素上属于被试内设计, 因此属于混合设计。\n\n因此答案为:B", "task_type": {"major": ["试题"], "minor": ["考研", "选择题"]}, "domain": ["心理学"], "metadata": "year:2022, score:2", "answer_from": "human", "human_verified": false, "copyright": "暂无版权及作者信息"} +{"instruction": "可能造成注意瞬脱(attentional blink)现象的研究范式是\nA. 空间线索范式\nB. 启动范式\nC. 多目标追踪范式\nD. 快速系列呈现范式", "input": "", "output": "注意瞬脱是指在识别一系列刺激流时对某个刺激的准确识别会影响到其后对特定时间间隔(一般为 500 毫秒以内)的刺激识别。注意瞬脱是 Raymond, Shapiro 和 Arnell 在 1992 首次通过快速系列视觉呈现范式命名的现象。经典的快速系列呈现范式是指, 快速呈现一系列视觉刺激(如: 数字、字母), 呈现的刺激之间通常只有极小的时间间隔, 约 $100 \\mathrm{~ms}$ 。在一系列的视觉刺激中有两个被试需要报告的目标 $\\mathrm{T} 1$, $\\mathrm{T} 2$, 其余的刺激均为无关干扰项。 $\\mathrm{T} 1$ 与 $\\mathrm{T} 2$ 之间可能间隔 0-7 个干扰项。\n\n因此答案为:D", "task_type": {"major": ["试题"], "minor": ["考研", "选择题"]}, "domain": ["心理学"], "metadata": "year:2022, score:2", "answer_from": "human", "human_verified": false, "copyright": "暂无版权及作者信息"} +{"instruction": "用最小变化法测量差别阈限时, 从 “-” 到 “=” 的转折点称为下限, 从 “=” 到 “+” 的转折点称为上限, 上限与下限之间称为不肯定间距, 差别阈限等于 $1 / 2$ 不肯定间距, 这时求得的差别阈限为\nA. 主观差别阈限\nB. 客观差别阈限\nC. 绝对差别阈限\nD. 相对差别阈限", "input": "", "output": "不肯定间距的中点是主观相等点(PSE), 理论上 PSE 与标准刺激相等, 但实际上两者有一定的差距。 \n \n\n因此答案为:A", "task_type": {"major": ["试题"], "minor": ["考研", "选择题"]}, "domain": ["心理学"], "metadata": "year:2022, score:2", "answer_from": "human", "human_verified": false, "copyright": "暂无版权及作者信息"} +{"instruction": "经典 Stroop 实验所使用的因变量测量指标为\nA. 反应时\nB. 比率\nC. 持续时间\nD. 偏好程度", "input": "", "output": "经典 Stroop 实验呈现一个一个用不同颜色写成的单词, 要求被试尽快而且尽量正确地说出每个词的颜色, 而不理会这个词的名称及其所代表的意义。该实验的因变量是反应时。 \n \n\n因此答案为:A", "task_type": {"major": ["试题"], "minor": ["考研", "选择题"]}, "domain": ["心理学"], "metadata": "year:2022, score:2", "answer_from": "human", "human_verified": false, "copyright": "暂无版权及作者信息"} +{"instruction": "与地球 24 小时的昼夜节律变化不同, ���州十三号航天员在太空中大约每 90 分钟就会经历一个节律变化, 一项研究比较节律对地球和空间站的航天员睡眠效率的影响, 则节律是\nA. 自变量\nB. 因变量\nC. 控制变量\nD. 无关变量", "input": "", "output": "自变量是指实验者所操纵的, 对被试的反应产生影响的变量。题干比较 “节律” 对睡眠效率的影响, 节律是自变量。\n\n因此答案为:A", "task_type": {"major": ["试题"], "minor": ["考研", "选择题"]}, "domain": ["心理学"], "metadata": "year:2022, score:2", "answer_from": "human", "human_verified": false, "copyright": "暂无版权及作者信息"} +{"instruction": "在心理物理学中, 制作等距量表采用的主要方法是\nA. 等级排列法\nB. 对偶比较法\nC. 差别阈限法\nD. 分段法", "input": "", "output": "等距量表建立的方法主要有二分法和差别阈限法。 \n \n\n因此答案为:C", "task_type": {"major": ["试题"], "minor": ["考研", "选择题"]}, "domain": ["心理学"], "metadata": "year:2022, score:2", "answer_from": "human", "human_verified": false, "copyright": "暂无版权及作者信息"} +{"instruction": "明确提出 “凡是存在必有数量” “凡有数量必可测量” 的心理学家分别是\nA.桑代克、麦柯尔\nB. 桑代克、瑟斯顿\nC. 麦柯尔、高尔顿\nD.瑟斯顿、高尔顿", "input": "", "output": "CTT 心理特质的可测性假设是 “凡客观存在的事物都其数量” (E.L.Thorndike) “凡有数量的东西都可以测量” (W.A.McCall)。 \n \n\n因此答案为:A", "task_type": {"major": ["试题"], "minor": ["考研", "选择题"]}, "domain": ["心理学"], "metadata": "year:2022, score:2", "answer_from": "human", "human_verified": false, "copyright": "暂无版权及作者信息"} +{"instruction": "一项智力测验总体平均分为 75 、标准差为 10 , 某学生的测验分数为 85 , 若将其转换成平均分为 100、标准差为 15 的标准分数, 则转换后的分数是\nA. 85\nB. 100\nC. 110\nD.115", "input": "", "output": "已知智力分数 $=100+15 \\mathrm{Z}$, 则先需要计算 $\\mathrm{Z}, \\mathrm{Z}=\\frac{X-\\bar{X}}{S}$ 。本题原始分数 X 为 85 , 平均数为 75 , 标准差为 10 , 代入公式得 $Z=\\frac{85-75}{10}=1$ 。将 $Z=1$ 代入公式 $=100+15 \\times 1=115$, 因此答案选 $D$ 。\n\n因此答案为:D", "task_type": {"major": ["试题"], "minor": ["考研", "选择题"]}, "domain": ["心理学"], "metadata": "year:2022, score:2", "answer_from": "human", "human_verified": false, "copyright": "暂无版权及作者信息"} +{"instruction": "在一元线性回归方程 $Y=a+b X$, 预测源是\nA.a\nB.b\nC.X\nD.Y", "input": "", "output": "预测源也称预测变量, 当一元线性回归方程为 Y 对 X 做回归时, 则 X 为预测源头, Y 为因变量。 \n \n\n因此答案为:C", "task_type": {"major": ["试题"], "minor": ["考研", "选择题"]}, "domain": ["心理学"], "metadata": "year:2022, score:2", "answer_from": "human", "human_verified": false, "copyright": "暂无版权及作者信息"} +{"instruction": "下列有关真分数理论的表述中, 正确的是\nA. 真分数和观察分数的相关为 1\nB. 真分数和误差分数的相关为 0\nC. 真分数的期望值等于观察分数\nD. 平行测验观察的分数相等", "input": "", "output": "真分数理论的假设公理之一是:真分数和误差分数之间的相关为 0 。即: $\\rho_{\\mathrm{ET}}=0$ 。 \n \n\n因此答案为:B", "task_type": {"major": ["试题"], "minor": ["考研", "选择题"]}, "domain": ["心理学"], "metadata": "year:2022, score:2", "answer_from": "human", "human_verified": false, "copyright": "暂无版权及作者信息"} +{"instruction": "与诊断测验相比, 人事选拔测验更加重视\nA. 预测效度\nB. 内容效度\nC. 表面效度\nD. 构想效度", "input": "", "output": "诊断测验的功能是对个人的问题行为及其原因进行诊断, 重视内容效度, 人事选拔测验主要看选拔人员是否符合选拔标准, 更重视预测效度。 \n \n\n因此答案为:A", "task_type": {"major": ["试题"], "minor": ["考研", "选择题"]}, "domain": ["心理学"], "metadata": "year:2022, score:2", "answer_from": "human", "human_verified": false, "copyright": "暂无版权及作者信息"} +{"instruction": "在次数分布曲线中, 左尾较长的单峰分布最可能是\nA. 正偏态分布\nB. 负偏态分布\nC. 正态分布\nD. 二项分布", "input": "", "output": "偏态分布分为正偏态和负偏态, 负偏态众数位置靠右, 左尾较长。 \n \n\n因此答案为:B", "task_type": {"major": ["试题"], "minor": ["考研", "选择题"]}, "domain": ["心理学"], "metadata": "year:2022, score:2", "answer_from": "human", "human_verified": false, "copyright": "暂无版权及作者信息"} +{"instruction": "方差分析时, 总变异可以分解为意义明确、彼此相互独立的几个不同部分, 这样做的依据是因为方差具有\nA. 离散性\nB.集中性\nC. 变异性\nD. 可加性", "input": "", "output": "方差分析的基本假定 (条件) 为总体正态分布、变异的相互独立性、处理内方差要一致。其中因为变异的相互独立性, 也即可加性, 总变异可以分解成几个不同来源的部分。\n\n因此答案为:D", "task_type": {"major": ["试题"], "minor": ["考研", "选择题"]}, "domain": ["心理学"], "metadata": "year:2022, score:2", "answer_from": "human", "human_verified": false, "copyright": "暂无版权及作者信息"} +{"instruction": "对一个 $2 \\times 2 \\times 2$ 的研究设计, 如果要进行方差分析, 变异源的主效应一共有\nA. 2 个\nB. 3 个\nC.6 6\nD. 8 个", "input": "", "output": "此设计为三因素设计, 因素数 $=$ 主效应数, 即主效应数为 3 。 \n \n\n因此答案为:B", "task_type": {"major": ["试题"], "minor": ["考研", "选择题"]}, "domain": ["心理学"], "metadata": "year:2022, score:2", "answer_from": "human", "human_verified": false, "copyright": "暂无版权及作者信息"} +{"instruction": "下列选项中, 反映数据分布离散趋势的是\nA. 差异量数\nB. 偏态量数\nC. 集中量数\nD. 地位量数", "input": "", "output": "标准差可反映一个次数分布的离散程度, 而标准差属于差异量数, 则答案选 A。 \n \n\n因此答案为:A", "task_type": {"major": ["试题"], "minor": ["考研", "选择题"]}, "domain": ["心理学"], "metadata": "year:2022, score:2", "answer_from": "human", "human_verified": false, "copyright": "暂无版权及作者信息"} +{"instruction": "若自变量 X 与因变量 Y 的相关系数为 -0.5 , 则 Y 为 X 的一元线性回归方程的决定系数为\nA. 0.25\nB. 0.5\nC. 0.55\nD. 0.75", "input": "", "output": "决定系数为 $r^{2}$, 本题 $\\mathrm{r}=-0.5, r^{2}=0.25$ 。 \n \n\n因此答案为:A", "task_type": {"major": ["试题"], "minor": ["考研", "选择题"]}, "domain": ["心理学"], "metadata": "year:2022, score:2", "answer_from": "human", "human_verified": false, "copyright": "暂无版权及作者信息"} +{"instruction": "数据分布比较分散且有极端值时, 描述集中趋势的最佳统计量是\nA. 平均数\nB. 中数\nC. 全距\nD. 众数", "input": "", "output": "平均数易受极端值影响, 在出现数据比较分散且有极端值时, 可使用中数和众数,而众数损失了更多的数据情况, 因此中数更适合描述集中趋势。 \n \n\n因此答案为:B", "task_type": {"major": ["试题"], "minor": ["考研", "选择题"]}, "domain": ["心理学"], "metadata": "year:2022, score:2", "answer_from": "human", "human_verified": false, "copyright": "暂无版权及作者信息"} +{"instruction": "有 15 道四择一的单项选择题, 某考生随机猜答, 恰好答对 5 道题的概率为\nA. $C_{15}^{5}\\left(\\frac{1}{15}\\right)^{5}\\left(\\frac{4}{15}\\right)^{10}$\nB. $C_{15}^{5}\\left(\\frac{1}{5}\\right)^{5}\\left(\\frac{4}{5}\\right)^{10}$\nC. $C_{15}^{5}\\left(\\frac{1}{4}\\right)^{5}\\left(\\frac{4}{5}\\right)^{10}$\nD. $C_{15}^{5}\\left(\\frac{1}{4}\\right)^{5}\\left(\\frac{3}{4}\\right)^{10}$", "input": "", "output": "本题考查二项分布的计算应用, 使用公式: $\\mathrm{C}_{\\mathrm{n}}^{\\mathrm{m}}(\\mathrm{p})^{\\mathrm{m}}(\\mathrm{q})^{\\mathrm{n}-\\mathrm{m}}$, 其中 $\\mathrm{n}=15, \\mathrm{~m}=5, \\mathrm{p}=\\frac{1}{4}$, $\\mathrm{q}=\\frac{3}{4}$, 本题不需要计算, 只需要选择正确的公式, 因此答案选 D 。 \n \n\n因此答案为:D", "task_type": {"major": ["试题"], "minor": ["考研", "选择题"]}, "domain": ["心理学"], "metadata": "year:2022, score:2", "answer_from": "human", "human_verified": false, "copyright": "暂无版权及作者信息"} +{"instruction": "总体服从正态分布且方差已知, 对容量为 n 的样本的平均数而言, 其标准误计算公式为\nA. $\\frac{\\sigma}{\\sqrt{n}}$\nB. $\\frac{S_{n}}{n-1}$\nC. $\\frac{S_{n-1}}{\\sqrt{n}}$\nD. $\\frac{\\sigma}{n-1}$", "input": "", "output": "总体服从正态分布且方差已知, 则无需进行无偏估计, 标准误计算公式为: $\\frac{\\sigma}{\\sqrt{n}}$, 因\n此答案选 A。 \n \n\n因此答案为:A", "task_type": {"major": ["试题"], "minor": ["考研", "选择题"]}, "domain": ["心理学"], "metadata": "year:2022, score:2", "answer_from": "human", "human_verified": false, "copyright": "暂无版权及作者信息"} +{"instruction": "某测试满分为 20 分, 且测试结果服从正态分布, 要了解该测试结果与性别是否有关联, 最恰当的方法是\nA. 等级相关\nB.二列相关\nC. 点二列相关\nD. $\\chi^{2}$ 检验", "input": "", "output": "一列连续数据(正态分布的测试结果)和一列二分变量, 且二分变量 (性别) 为真正二分变量, 可使用点二列相关。 \n \n\n因此答案为:C", "task_type": {"major": ["试题"], "minor": ["考研", "选择题"]}, "domain": ["心理学"], "metadata": "year:2022, score:2", "answer_from": "human", "human_verified": false, "copyright": "暂无版权及作者信息"} +{"instruction": "已知两个正态总体独立且方差未知, 但两总体方差相等, 检验其均值的差异是否具有统计学意义,最适当的统计方法是\nA. 符号检验\nB. 秩和检验\nC.t 检验\nD.Z 检验", "input": "", "output": "已知两个正态总体独立且方差未知, 则数据可认为其呈 t 分布, 又知两总体方差相等, 因此方差齐性, 满足独立样本 t 检验的要求, 因此选 C 。", "task_type": {"major": ["试题"], "minor": ["考研", "选择题"]}, "domain": ["心理学"], "metadata": "year:2022, score:2", "answer_from": "human", "human_verified": false, "copyright": "暂无版权及作者信息"} +{"instruction": "区间估计依据的是\nA. 概率论原理\nB. 反证法原理\nC. 抽样分布原理\nD.小概率事件原理", "input": "", "output": "区间估计的原理是根据样本分布理论, 用样本分布的标准误 (SE), 计算区间长度,解释总体参数落入某置信区间可能的概率。 \n \n\n因此答案为:C", "task_type": {"major": ["试题"], "minor": ["考研", "选择题"]}, "domain": ["心理学"], "metadata": "year:2022, score:2", "answer_from": "human", "human_verified": false, "copyright": "暂无版权及作者信息"} +{"instruction": "依据项目反应理论, 下列表述中正确的是\nA. 项目的难度影响区分度\nB.项目的难度不影响信息函数\nC. 项目的参数具有跨样本不变性\nD. 项目的区分度不影响信息函数", "input": "", "output": "项目反应理论的优良性质之一是题目参数的跨群体不变性。\n\n因此答案为:C", "task_type": {"major": ["试题"], "minor": ["考研", "选择题"]}, "domain": ["心理学"], "metadata": "year:2022, score:2", "answer_from": "human", "human_verified": false, "copyright": "暂无版权及作者信息"} +{"instruction": "“阴中求阳” 的理论依据是\nA.阳得阴制而不亢盛\nB. 阴得阳助而不凝滞\nC. 阳得阴助而生化无穷\nD. 阴得阳升而泉源不嘳", "input": "", "output": "由于 “阳根于阴”, 故亦可滋阴以助阳。称为\"阴中求阳\", 即在补阳方剂中适当佐以滋阴药, \"昍得阴助而生化无穷\n\n因此答案为:C", "task_type": {"major": ["试题"], "minor": ["考研", "选择题"]}, "domain": ["中医学"], "metadata": "year:2023, score:1.5", "answer_from": "human", "human_verified": false, "copyright": "暂无版权及作者信息"} +{"instruction": "属于抑强扶弱、五行相克关系的是\nA.滋肾健脾\nB. 滋水涵木\nC.泻心清肝\nD. 疏肝健脾", "input": "", "output": "AB 体现了相生关系, C 不属于相生相克制定的治法; D 是根据相克指定的抑木扶土法\n\n因此答案为:D", "task_type": {"major": ["试题"], "minor": ["考研", "选择题"]}, "domain": ["中医学"], "metadata": "year:2023, score:1.5", "answer_from": "human", "human_verified": false, "copyright": "暂无版权及作者信息"} +{"instruction": "《上古天真论》, 男子五八\nA 筋骨隆盛\nB. 发堕齿槁\nC. 天癸蝎\nD. 面焦", "input": "", "output": ":《黄帝内经》云:丈夫儿岁, 肾气实, 发长齿更; 二八肾气盛, 天癸至, 精气溢泻, 阴阳和, 故能有子。三八肾气平均, 筋骨劲强, 故真牙生而长报。四儿筋骨隆盛, 肌肉洴壮;五八肾气衰, 发值齿槁。六八阳气衰施于上, 面焦, 发尝颁白; 七八肝气衰, 筋不能动,天癸蝎, 精少, 肾藏衰, 形体皆极, 儿则齿发去。\n\n因此答案为:B", "task_type": {"major": ["试题"], "minor": ["考研", "选择题"]}, "domain": ["中医学"], "metadata": "year:2023, score:1.5", "answer_from": "human", "human_verified": false, "copyright": "暂无版权及作者信息"} +{"instruction": "根据《素问兰灵秘典论》,三焦属于\nA.受盛之官\nB. 传导之官\nC. 决渎之官\nD. 州都之官", "input": "", "output": "A 是小肠, B 是大肠; D 是膀肤\n\n因此答案为:C", "task_type": {"major": ["试题"], "minor": ["考研", "选择题"]}, "domain": ["中医学"], "metadata": "year:2023, score:1.5", "answer_from": "human", "human_verified": false, "copyright": "暂无版权及作者信息"} +{"instruction": "最小的经络\nA. 孙络\nB. 经别\nC. 浮络\nD. 血络", "input": "", "output": "最小的经络是孙络\n\n因此答案为:A", "task_type": {"major": ["试题"], "minor": ["考研", "选择题"]}, "domain": ["中医学"], "metadata": "year:2023, score:1.5", "answer_from": "human", "human_verified": false, "copyright": "暂无版权及作者信息"} +{"instruction": "外感署邪, 症状见口渴喜饮, 气短之力病机是\nA. 署应夏季, 其性炎上\nB. 署为阳邪\nC. 署多夹湿\nD. 署性升散, 伤津耗气", "input": "", "output": "署邪伤津会出现口渴喜饮, 耗气会出现气哣亏力\n\n因此答案为:D", "task_type": {"major": ["试题"], "minor": ["考研", "选择题"]}, "domain": ["中医学"], "metadata": "year:2023, score:1.5", "answer_from": "human", "human_verified": false, "copyright": "暂无版权及作者信息"} +{"instruction": "至虚有盛侯是指\nA.正气虚, 邪气盛, 外现实象\nB.邪盛, 气血内闭, 外现虚象\nC. 脏腑精气极度虚衰, 气化不利, 外现实象\nD. 邪盛, 伤阴血, 外现虚象", "input": "", "output": "至虚有盛侯是指真虚假实证\n\n因此答案为:C", "task_type": {"major": ["试题"], "minor": ["考研", "选择题"]}, "domain": ["中医学"], "metadata": "year:2023, score:1.5", "answer_from": "human", "human_verified": false, "copyright": "暂无版权及作者信息"} +{"instruction": "崩漏下血, 色紫黯夹有血块者, 治宜\nA. 收涩止血\nB.益气摄血\nC. 温经止血\nD. 化㾌止血", "input": "", "output": "紫暗夹有血块是有床血的表现, 治当化麻止血\n\n因此答案为:D", "task_type": {"major": ["试题"], "minor": ["考研", "选择题"]}, "domain": ["中医学"], "metadata": "year:2023, score:1.5", "answer_from": "human", "human_verified": false, "copyright": "暂无版权及作者信息"} +{"instruction": "不属于面色青面色黑相同主症的是\nA. 血寒\nB. 惊风\nC. 疼痛\nD. 痗血", "input": "", "output": "青色主病: 气滞、痃血、寒证、疼痛、惊风\n黑色主病:肾病、㿑血、寒证、疼痛、水饮\n\n因此答案为:B", "task_type": {"major": ["试题"], "minor": ["考研", "选择题"]}, "domain": ["中医学"], "metadata": "year:2023, score:1.5", "answer_from": "human", "human_verified": false, "copyright": "暂无版权及作者信息"} +{"instruction": "下列证型中, 可见里急后重的是\nA.湿热蕴脾\nB. 寒湿困脾\nC. 肠热腑实\nD. 大肠湿热", "input": "", "output": "里急后重常见于㾭疾, 为湿热内助, 脆道气滞之故\n\n因此答案为:D", "task_type": {"major": ["试题"], "minor": ["考研", "选择题"]}, "domain": ["中医学"], "metadata": "year:2023, score:1.5", "answer_from": "human", "human_verified": false, "copyright": "暂无版权及作者信息"} +{"instruction": "下列不是燥苔的原因\nA. 热邪炽盛\nB. 津液耗伤\nC. 水湿内蕴\nD.气不化津", "input": "", "output": "煤苔往往见于津伤, 津旅不足等, 水湿多见于其苔、滑苔等\n\n因此答案为:C", "task_type": {"major": ["试题"], "minor": ["考研", "选择题"]}, "domain": ["中医学"], "metadata": "year:2023, score:1.5", "answer_from": "human", "human_verified": false, "copyright": "暂无版权及作者信息"} +{"instruction": "下列选项中, 属于署淫证与火淫证共同表现的是\nA. 发热恶寒, 咽喉肿痛\nB. 发热汗出, 口渴尿黄\nC. 发热恶热, 气短神疲\nD. 疘肿疮疡, 便秘尿黄", "input": "", "output": "$\\mathrm{AD}$ 为火淫证表现, C 为署滛证表现\n\n因此答案为:B", "task_type": {"major": ["试题"], "minor": ["考研", "选择题"]}, "domain": ["中医学"], "metadata": "year:2023, score:1.5", "answer_from": "human", "human_verified": false, "copyright": "暂无版权及作者信息"} +{"instruction": "下列不属于饮证的是\nA.胸闹心悸\nB. 带下清晰\nC. 头星目眩\nD. 脘腹痁满", "input": "", "output": "根据饮停部位的不同, 临床常分为饮停胃朌证、饮停䏛胁证、饮停心包证、饮牙䧋肺证等。\n\n因此答案为:B", "task_type": {"major": ["试题"], "minor": ["考研", "选择题"]}, "domain": ["中医学"], "metadata": "year:2023, score:1.5", "answer_from": "human", "human_verified": false, "copyright": "暂无版权及作者信息"} +{"instruction": "下列不属于血热临床表现的是\nA.斑疹紫黑\nB. 反复出血血块\nC. 经血鲜红质粘稠\nD. 月经先期量多", "input": "", "output": "A 为卫气营血辩证中血分证的表现, 血热证属于病性枋证中的气血辩证, 不属于同一站证体系。\n\n因此答案为:A", "task_type": {"major": ["试题"], "minor": ["考研", "选择题"]}, "domain": ["中医学"], "metadata": "year:2023, score:1.5", "answer_from": "human", "human_verified": false, "copyright": "暂无版权及作者信息"} +{"instruction": "心悸心烦, 失眠多梦, 潮热颧红, 舌红少津, 脉细数\nA心阴虚\nB.心火元盛\nC.小肠实热\nD.心肾不交", "input": "", "output": "症状为阴虚有热, 并未见到渡㴎砝软等肾虚证侯, D 错\n\n因此答案为:A", "task_type": {"major": ["试题"], "minor": ["考研", "选择题"]}, "domain": ["中医学"], "metadata": "year:2023, score:1.5", "answer_from": "human", "human_verified": false, "copyright": "暂无版权及作者信息"} +{"instruction": "在伤寒论中, 太阳经与少阳经同时发病, 可见于\nA. 并病\nB. 合病\nC.越经传\nD.循经传", "input": "", "output": "凡疾病发病之初, 两经或二经的病证同时出现, 称为合病。\n\n因此答案为:B", "task_type": {"major": ["试题"], "minor": ["考研", "选择题"]}, "domain": ["中医学"], "metadata": "year:2023, score:1.5", "answer_from": "human", "human_verified": false, "copyright": "暂无版权及作者信息"} +{"instruction": "见于肝胆湿热不见于湿热蕴脾的是\nA. 身目发黄\nB. 舌苔黄榪\nC. 口苦\nD. 阴痒湿疹", "input": "", "output": "湿热蕴脾不会见到阴单湿疹\n\n因此答案为:D", "task_type": {"major": ["试题"], "minor": ["考研", "选择题"]}, "domain": ["中医学"], "metadata": "year:2023, score:1.5", "answer_from": "human", "human_verified": false, "copyright": "暂无版权及作者信息"} +{"instruction": "水肿用的药的性味\nA.涩\nB.淡\nC. 苦\nD.咸", "input": "", "output": "淡㕲药可以利水渗湿\n\n因此答案为:B", "task_type": {"major": ["试题"], "minor": ["考研", "选择题"]}, "domain": ["中医学"], "metadata": "year:2023, score:1.5", "answer_from": "human", "human_verified": false, "copyright": "暂无版权及作者信息"} +{"instruction": "下列各项中, 治疗实热积滞, 大便燥结, 尤为适宜的是\nA. 决明子\nB.番浑叶\nC. 虎杖\nD. 芒硝", "input": "", "output": "芒硝污下通便, 汪捺软坚, 清火消肿。\n\n因此答案为:D", "task_type": {"major": ["试题"], "minor": ["考研", "选择题"]}, "domain": ["中医学"], "metadata": "year:2023, score:1.5", "answer_from": "human", "human_verified": false, "copyright": "暂无版权及作者信息"} +{"instruction": "附子和肉桂的共有药效\nA. 回阳通脉, 温中止呕\nB. 引火归元, 散寒通滞\nC. 回阳救逆, 散寒止痛\nD.温中散寒, 温肺化饮", "input": "", "output": "时子回阳救逆, 补火助阳, 散寒止痛。肉挂: 补火助阳, 散寒止痛, 温通经脉, 引火归元。\n\n因此答案为:C", "task_type": {"major": ["试题"], "minor": ["考研", "选择题"]}, "domain": ["中医学"], "metadata": "year:2023, score:1.5", "answer_from": "human", "human_verified": false, "copyright": "暂无版权及作者信息"} +{"instruction": "下列驱虫药中, 性味苦、辛, 温, 归胃、大肠经的是\nA. 南瓜子\nB. 鹤草芽\nC.槟楖\nD. 棑子", "input": "", "output": "槟枋苦、辛, 温, 归胃、大胅经\n\n因此答案为:C", "task_type": {"major": ["试题"], "minor": ["考研", "选择题"]}, "domain": ["中医学"], "metadata": "year:2023, score:1.5", "answer_from": "human", "human_verified": false, "copyright": "暂无版权及作者信息"} +{"instruction": "朱砂多入丸散服, 每次用量为\nA. $0.015-0.03 \\mathrm{~g}$\nB. $0.1-0.5 \\mathrm{~g}$\nC. $1-2 g$\nD. $3-5 \\mathrm{~g}$", "input": "", "output": "朱砂入丸敨 $0.1-0.5 \\mathrm{~g}$\n\n因此答案为:B", "task_type": {"major": ["试题"], "minor": ["考研", "选择题"]}, "domain": ["中医学"], "metadata": "year:2023, score:1.5", "answer_from": "human", "human_verified": false, "copyright": "暂无版权及作者信息"} +{"instruction": "升麻的功效是\nA. 透疹\nB. 升阳止泻\nC.疏肝解有\nD. 生津止渴", "input": "", "output": "升麻发表透疹, 清热解毒, 升举阳气, 功效中没有止泻, 主治中有\n\n因此答案为:A", "task_type": {"major": ["试题"], "minor": ["考研", "选择题"]}, "domain": ["中医学"], "metadata": "year:2023, score:1.5", "answer_from": "human", "human_verified": false, "copyright": "暂无版权及作者信息"} +{"instruction": "既能害痰行气, 又能止痛的药物是\nA.白前\nB. 细辛\nC. 芥子\nD. 葛根", "input": "", "output": "芥子温肺帮痸, 利气致结, 通络止痛。\n\n因此答案为:C", "task_type": {"major": ["试题"], "minor": ["考研", "选择题"]}, "domain": ["中医学"], "metadata": "year:2023, score:1.5", "answer_from": "human", "human_verified": false, "copyright": "暂无版权及作者信息"} +{"instruction": "既可以疏肝行气解郁, 又可以消食健脾开胃的药物是\nA.玫瑰花\nB. 佛手\nC. 麦芽\nD. 莱䓳子", "input": "", "output": "麦芽主治: 1.皊积不化, 脘腹胀满, 脾虚貪少; 2孚汁有积, 孚房胀痛, 妇女断乳; 3.肝有胁痛, 肝胃气痛\n\n因此答案为:C", "task_type": {"major": ["试题"], "minor": ["考研", "选择题"]}, "domain": ["中医学"], "metadata": "year:2023, score:1.5", "answer_from": "human", "human_verified": false, "copyright": "暂无版权及作者信息"} +{"instruction": "差活胜湿汤与大秦艽汤共同组成的药物有\nA.苍术, 独活\nB.川芎, 防风\nC.菜本, 白芷\nD.细辛, 甘草", "input": "", "output": "本影考察方剂组成\n\n因此答案为:B", "task_type": {"major": ["试题"], "minor": ["考研", "选择题"]}, "domain": ["中医学"], "metadata": "year:2023, score:1.5", "answer_from": "human", "human_verified": false, "copyright": "暂无版权及作者信息"} +{"instruction": "《杨氏家藏方》的草薢分清饮有而《医学心悟》的草薢分清饮不具有的药物是\nA.乌药、益智仁\nB. 白术、石菖蒲\nC. 获苓、车前子\nD. 肉桂、小茴香", "input": "", "output": "《扬氏家藏方》组成:益智化、川草花、石菖蒲、乌药\n《医学心悟》组成: 黄柏、川萆䒚、石昌蒲、获苓、白术、辇子心、丹参、车前子\n\n因此答案为:A", "task_type": {"major": ["试题"], "minor": ["考研", "选择题"]}, "domain": ["中医学"], "metadata": "year:2023, score:1.5", "answer_from": "human", "human_verified": false, "copyright": "暂无版权及作者信息"} +{"instruction": "吴茱英汤的功效是\nA.温中散寒, 缓急止痛\nB.温中补虚, 降逆止呕\nC. 温中补气, 和里��急\nD.温中散寒, 健脾止泻", "input": "", "output": "温中补虚, 降逆止呕。\n\n因此答案为:B", "task_type": {"major": ["试题"], "minor": ["考研", "选择题"]}, "domain": ["中医学"], "metadata": "year:2023, score:1.5", "answer_from": "human", "human_verified": false, "copyright": "暂无版权及作者信息"} +{"instruction": "紫金锭和仙方活命饮均有的功效是\nA. 清热解毒\nB. 消肿止痛\nC. 去腐生肌\nD. 软坚散结", "input": "", "output": "紫金锭: 辟移解毒, 化哓开空, 消肿止痛。\n仙方活命饮: 清热解毒, 消肿溃坚, 活血止痛\n\n因此答案为:B", "task_type": {"major": ["试题"], "minor": ["考研", "选择题"]}, "domain": ["中医学"], "metadata": "year:2023, score:1.5", "answer_from": "human", "human_verified": false, "copyright": "暂无版权及作者信息"} +{"instruction": "注重气血双补, 重在补气。以达到气旺血行, 养心安神的方剂是\nA.归脾汤\nB. 当归养血汤\nC. 补中益气汤\nD.天王补心丹", "input": "", "output": "归脾汤气血双补, 重在补气。以达到气旺血行, 养心安神\n\n因此答案为:A", "task_type": {"major": ["试题"], "minor": ["考研", "选择题"]}, "domain": ["中医学"], "metadata": "year:2023, score:1.5", "answer_from": "human", "human_verified": false, "copyright": "暂无版权及作者信息"} +{"instruction": "在清上与泻下的方剂中, 用泻下以清泻郁热的是\nA.大承气汤\nB. 黄连解毒汤\nC. 凉膈散\nD. 葛根芩连汤", "input": "", "output": "凉膈散 “以清代泻”\n\n因此答案为:C", "task_type": {"major": ["试题"], "minor": ["考研", "选择题"]}, "domain": ["中医学"], "metadata": "year:2023, score:1.5", "answer_from": "human", "human_verified": false, "copyright": "暂无版权及作者信息"} +{"instruction": "治疗妇人素有癔病而漏下不止, 或产后恶露不尽而疼痛拒按者, 方用\nA.生化汤\nB. 失笑散\nC.活络效灵丹\nD. 挂枝茯苓丸", "input": "", "output": "挂技获苓丸主治应阻胞宫证。\n\n因此答案为:D", "task_type": {"major": ["试题"], "minor": ["考研", "选择题"]}, "domain": ["中医学"], "metadata": "year:2023, score:1.5", "answer_from": "human", "human_verified": false, "copyright": "暂无版权及作者信息"} +{"instruction": "临床应用四逆汤, 症见四肢厥冷, 恶寒蜷卧, 脉微而复自下利, 利止而余症仍在, 应该加\nA. 䓤白\nB. 人参\nC. 生姜汁\nD. 猪胆汁", "input": "", "output": "四逆力人参汤《伤寒论》\n功用: 回阳救逆, 益气固脱。主治: 少阴病真阳衰德, 元气亦虚之证。症见四肢㞛逆, 恶寒埢卧, 脉敌而复自下利, 利虽止而余症仍在者。\n\n因此答案为:B", "task_type": {"major": ["试题"], "minor": ["考研", "选择题"]}, "domain": ["中医学"], "metadata": "year:2023, score:1.5", "answer_from": "human", "human_verified": false, "copyright": "暂无版权及作者信息"} +{"instruction": "猪采食后胃排空的过程受多种因素的影响。请简述影响猪胃排空的因素及其作用过程。", "input": "", "output": "(1)促进胃排空的因素: 食物对胃的扩张刺激通过迷走一迷走反射和壁内神经丛反射,促进胃排空;食物还可通过体液因素促进胃排空。\n(2)抑制胃排空的因素: 进人十二指肠的酸性食糜、脂肪、高渗溶液等均可引起肠一胃反射,抑制胃排空;酸和脂肪也可通过体液因素抑制胃排空。", "task_type": {"major": ["试题"], "minor": ["考研", "开放题"]}, "domain": ["动物生理学"], "metadata": "year:2022, score:6", "answer_from": "human", "human_verified": false, "copyright": "暂无版权及作者信息"} +{"instruction": "肾血流量不变而肾小球滤过率增加时, 近端小管对 $\\mathrm{Na}^{+}$和水的重吸收量有何变化? 请简述变化的原因及生理意义。", "input": "", "output": "(1) 近端小管对 $\\mathrm{Na}^{+}$和水的重吸收量增加。\n(2)原因: 当肾小球滤过率发生改变时,近端小管中 $\\mathrm{Na}^{+}$和水的重吸收量占滤过量的百分比保持不变, 因此滤过率增加时, 重吸收量也增加。\n(3) 意义: 保持尿量和尿 $\\mathrm{Na}^{+}$的相对稳定。", "task_type": {"major": ["试题"], "minor": ["考研", "开放题"]}, "domain": ["动物生理学"], "metadata": "year:2022, score:6", "answer_from": "human", "human_verified": false, "copyright": "暂无版权及作者信息"} +{"instruction": "哺乳动物的潮气量和呼吸频率可影响肺的气体交换效率。若潮气量加倍而呼吸频率减半,则动物的呼吸变深变慢; 反之则动物的呼吸变浅变快。请比较深而慢呼吸和浅而快呼吸气体交换效率的高低,并分析原因。", "input": "", "output": "(1)深而慢呼吸比浅而快呼吸的气体交换效率高。\n(2)深而慢呼吸时, 潮气量增加, 肺泡通气量增加,气体交换效率升高; 浅而快呼吸时, 潮气量减少,肺泡通气量减少,气体交换效率降低。", "task_type": {"major": ["试题"], "minor": ["��研", "开放题"]}, "domain": ["动物生理学"], "metadata": "year:2022, score:6", "answer_from": "human", "human_verified": false, "copyright": "暂无版权及作者信息"} +{"instruction": "哺乳动物大脑皮层运动区是调控躯体运动的最高级中枢。请阐述其对躯体运动的调控特征。", "input": "", "output": "(1)大脑皮层对躯体运动的调节是交叉性支配,但头面积是双侧性支配。\n(2)运动区定位呈前后 (上下) 倒置分布。\n(3)特定的皮层运动区精确支配特定部位的肌肉运动。\n(4)运动越精细复杂, 其对应的皮层代表区越大。", "task_type": {"major": ["试题"], "minor": ["考研", "开放题"]}, "domain": ["动物生理学"], "metadata": "year:2022, score:6", "answer_from": "human", "human_verified": false, "copyright": "暂无版权及作者信息"} +{"instruction": "心肌传导性是心肌的生理特性之一。请分析影响心肌传导性的主要因素。", "input": "", "output": "(1)结构因素: 心肌细胞的大小和细胞缝隙连接的数量均影响传导性。心肌细胞的直径愈大, 细胞之间的缝隙连接数量愈多, 则传导速度愈快; 心肌细胞的直径愈小, 细胞之间的缝隙连接数量愈少, 则传导速度愈慢。\n(2)电生理因素: 心肌细胞 0 期去极化的速度愈快、幅度愈高, 则兴奋传导愈快; 心肌细胞 0 期去极化的速度愈慢、幅度愈低, 则兴奋传导愈慢。邻近细胞的兴奋性愈高, 则兴奋传导愈快; 邻近细胞的兴奋性愈低, 则兴奋传导愈慢。", "task_type": {"major": ["试题"], "minor": ["考研", "开放题"]}, "domain": ["动物生理学"], "metadata": "year:2022, score:6", "answer_from": "human", "human_verified": false, "copyright": "暂无版权及作者信息"} +{"instruction": "在血液凝固及其影响因素的实验中, 某同学在分别含有适量氯化钻、柠檬酸钠和肝素溶液的试管中各加人 $1 \\mathrm{~mL}$ 新鲜血液样本, 发现这些物质均影响血液凝固过程。回答下列问题:\n(1) 分别说明氯化钙、柠檬酸钠和肝素对血液凝固的影响并分析原因。\n(2) 在本实验操作中应注意哪些事项? (答出 2 点即可)", "input": "", "output": "(1) $\\mathrm{Ca}^{2+}$ 是凝血因子之一, 血液凝固的多个过程均需要 $\\mathrm{Ca}^{2+}$ 参与, 因此氯化钻可促进血液凝固。\n柠檬酸钠可与 $\\mathrm{Ca}^{2+}$ 结合而除去血浆中游离的 $\\mathrm{Ca}^{2+}$, 从而起到抗凝作用。\n肝素可通过增强抗凝血酶活性等途径发挥间接凝抗作用。\n(2)注意事项:采集的血液应及时加人试管中;血液与试剂要充分混匀。", "task_type": {"major": ["试题"], "minor": ["考研", "开放题"]}, "domain": ["动物生理学"], "metadata": "year:2022, score:13", "answer_from": "human", "human_verified": false, "copyright": "暂无版权及作者信息"} +{"instruction": "现有 $n(n>100000)$ 个数保存在一维数组 $M$ 中, 需要在找 $M$ 中最小的 10 个数。请回答下列问题。\n(1)设计一个完成上述查找任务的算法, 要求平均情况下的比较次数尽可能少, 简述其算法思想 (不要程序实现)。\n(2) 说明你所设计的算法平均情况下的时间复杂度和空间复杂度。", "input": "", "output": "(1)算法思想\n【答案一】\n定义含 10 个元素的数组 $\\mathrm{A}$, 元素值均为该数组类型能表示的最大数 MAX。\nfor $M$ 中的每个元素 $s$\nif $(\\mathrm{s}<\\mathrm{A}[9])$ 丢弃 $\\mathrm{A}[9]$ 并将 $\\mathrm{s}$ 按升序插入 $\\mathrm{A}$ 中;\n当数据全部扫描完毕, 数组 $\\mathrm{A}$ 中保存的就是最小的 10 个数。\n【答案二】\n定义含 10 个元素的大根堆 $\\mathrm{H}$, 元素值均为该堆元素类型能表示的最大数 MAX。\nfor $\\mathrm{M}$ 中的每个元素 $\\mathrm{s}$\nif $(\\mathrm{s}<\\mathrm{H}$ 的堆顶元素) 删除堆顶元素并将 $\\mathrm{s}$ 插入 $\\mathrm{H}$ 中;\n当数据全部扫描完毕, 堆 $\\mathrm{H}$ 中保存的就是最小的 10 个数。\n(2)算法平均情况下的时间复杂度是 $O(n)$ ,空间复杂度是 $O(1)$ 。", "task_type": {"major": ["试题"], "minor": ["考研", "开放题"]}, "domain": ["计算机"], "metadata": "year:2022, score:10", "answer_from": "human", "human_verified": false, "copyright": "暂无版权及作者信息"} +{"instruction": "假设某磁盘驱动器中有 4 个双面盘片, 每个盘面有 20000 个磁道, 每个磁道有 500 个扇区, 每个扇区可记录 512 字节的数据, 盘片转速为 $7200 \\mathrm{r} / \\mathrm{m}$ (专/分), 平均寻道时间为 $5 \\mathrm{~ms}$. 请回答下列问题.\n(1)每个嘲区包含数据及其地址信息,地址信息分为 3 个字段. 这 3 个字段的名称各是什么?对于该磁盘,各字段至少占多少位?\n(3)若采用周期挪用 DMA 方式进行磁盘与主机之间的数据传送, 磁盘控制器中的数据缓冲区大小为 64 位, 则在一个扇区读写过程中, DMA 控制器向 CPU 发送了多少次总线请求?若 CPU 检测到 DMA ��制器的总线请求信号时也需要访问主存, 则 DMA 控制器是否可以获得总线使用权? 为什么", "input": "", "output": "(1) 3 个字段的名称为柱面号(或磁道号)、磁头号(或盘面号)、扇区号; 该磁盘的柱面号、磁头号、扇区号字段至少分别占 $\\left[\\log _{2} 20000\\right\\rceil=15$ 位、 $\\left[\\log _{2}(4 \\times 2) \\quad\\right]=3$ 位、 $\\left\\lceil\\log _{2} 500\\right\\rceil=9$ 位。\n(3) 在一个扇区读写过程中, DMA 控制器向 CPU 发送了 $512 \\mathrm{~B} / 64 \\mathrm{~b}=64$ 次总线请求。DMA 控制器可以获得总线使用权。因为一旦磁盘开始读写就必须按时完成数据传送, 否则会发生数据丢失", "task_type": {"major": ["试题"], "minor": ["考研", "开放题"]}, "domain": ["计算机"], "metadata": "year:2022, score:8", "answer_from": "human", "human_verified": false, "copyright": "暂无版权及作者信息"} +{"instruction": "Read the following text carefully and then translate the underlined segments into Chinese. Write your answers on the ANSWER SHEET. (10 points)\n The Man Who Broke Napoleon’s Code\n Between 1807 and 1814 the Iberian Peninsula (comprising Spain and Portugal) was the scene of a titanic and merciless struggle. It took place on many different planes: between Napoleon’s French army and the angry inhabitants; between the British, ever keen to exacerbate the emperor’ difficulties, and the marshals sent from Paris to try to keep them in check; between new forces of science and meritocracy and old ones of conservatism and birth. (46) It was also, and this is unknown even to many people well read about the period, a battle between those who made codes and those who broke them.\n I first discovered the Napoleonic cryptographic battle a few years ago when I was reading Sir Charles Oman’s epic History of the Peninsular War. In volume V he had attached an appendix, The Scovell Ciphers. (47) It listed many documents in code that had been captured from the French army of Spain, and whose secrets had been revealed by the work of George Scovell, an officer in British headquarters. Oman rated Scovell’s significance highly, but at the same time, the general nature of his History meant that (48) he could not analyze carefully what this obscure officer may or may not have contributed to that great struggle between nations or indeed tell us anything much about the man himself. I was keen to read more, but was surprised to find that Oman’s appendix, published in 1914, was the only considered thing that had been written about this secret war.\n I became convinced that this story was every bit as exciting and significant as that of Enigma and the breaking of German codes in the Second World War. The question was, could it be told? Studying Scovell’s papers at the Public Record Office (in Kew, west London) I found that he had left an extensive journal and copious notes about his work in the peninsula. What was more, many original French dispatches had been preserved in this collection. I realized at once that this was priceless. (49) There may have been many spies and intelligence officers during the Napoleonic Wars, but it is usually extremely difficult to find the material they actually provided or worked on. Furthermore, Scovell’s story involved much more than just intelligence work. His status in Lord Wellington’s headquarters and the recognition given to him for his work were all bound up with the class politics of the army at the time. His tale of self-improvement and hard work would make a fascinating biography in its own right, but represents something more than that. (50) Just as the code breaking has its wider relevance in the struggle for Spain, so his attempts to make his way up the promotion ladder speak volumes about British society.", "input": "", "output": "46.这场斗争也是一场编写密码者和破译密码者之间展开的战争,但是,这一点甚至对 那些熟知这一时期历史的人们来说都是未知的。\n47.这本书的附录中列出了从西班牙的法军那里缴获的许多密码文件,这些文件的秘密 由英国总部的一位军官乔治 ·斯科维尔(George Scovell)揭露了出来。\n48.阿曼(Oman) 无法仔细分析这位名不见经传的军官对那场国家间的伟大斗争有没有 做出贡献,也确实无法告诉我们任何有关此人的更多事情。\n49.在拿破仑战争时期,也许存在很多的间谍和情报官员,但通常难以找到他们实际提 供或者破译的情报资料。\n50.正如破解密码对于西班牙之战更为重要,斯科维尔加官进爵的尝试也将英国社会形 态体现得淋漓尽致。\n\n\n分析:\n\n46、It was also,and this is unknowneven to many people well read about the period,a battle between those who made codes and those who broke them.【结构分析】本句主干部分为It was also a battle,其后介词短语between...and... 中有两个定从,按照 段定从的处理思理可以前置,即 ... ... 的“人们” 。中间插入成分��并列内容,可以放在之后翻译。【译点分析】( 1) It:it 应该明确指代,是第一句最后的 struggle,可以翻译为“这场斗争”,如果翻译为“它”属于指代不明。(2) well: 此处为副词表程度,不是“好”的含义,类似于 well-prepared, 因此翻译方向为程度加 强 。结合read 含义,即“熟读” 。但如果翻译为“熟读”,与之后的 period“时期”含义并不是特 别搭配 。那么有两种处理: 1)熟读这一时期的历史; 2)熟“知”这一时期的历史。(3) codes:虽然是个典型的熟词僻义考点,但文中就是本意“密码”,因为在第四句提到了 spy“间 谍”和intelligence officer 情报人员,所以不应该翻译为“法则,准则” 。那 make codes 是“设置 /制定”密码,broke them 则是“破译”密码\n47、 It listed many documents in code that have been captured from the French army of Spain, and whose secrets had been revealed by the work of George Scovell, an officer in British headquarters.【结 构 解 析 】 句 子 主 干 是 Itlisted manydocuments, 后 面 incode 介 词 短 语 作 定 语 修 饰documents; ...that ... , and whose ... 为两个并列的定语从句, 均是对前方名词documents 的修饰限 定,且为被动结构; an officer in British headquarters是同位语修饰George Scovell。【译点分析】(1)代词it指代明确,往上指代the Scovell Ciphers, an appendix,可以译为“这一附录” 。(2)本句子的主干部分It listed many documents较短,占比0.5分。(3)that和whose两个并列的定语从句,句子比较长可以进行后置翻译,翻译时注意代词指代明 确即可。(4)两个定语从句占比较多,各自0.5分。(5)an officer in British headquarters作为句子中间的同位语,对前方的人进行解释说明,在整个 句子的分值中占比0.5分。\n48、he could not analyze carefully what this obscure officer may or may not have contributed tothat great struggle between nations or indeed tell us anything much about the man himself.【结构分析】本句主干为 he could not analyze what ... or tell us ...:what 引导宾语从句,or 引导 并列句, 并列两个动作 analyze carefully what...和 tell us anything much about the man himself。【译点分析】(1)句中 or 的并列容易误会成 tell us 与从句中的 have contributed 并列,造成理解及翻译的问题; 可以通过what 在从句中需要充当主语或宾语进行判断,后半句中tell sb sth双宾齐全,因此tell 只能与主干句中的analyze 并列。(2) contribute to:此处并非 contribute to sth 表“有助于,导致”,而需要将 what 做宾语的位置进行 还原,做contribute 的宾语,格式为 contribute A to B,意为“对 ...有...贡献” 。(3) obscure:考查了单词的一词多义,除了“模糊的,不明确的”以外,还有“名不见经传的”含义, 此处与officer 搭配,应选择后者。\n49、There may have been many spies and intelligence officers during the Napoleonic Wars but it isusually extremely difficult to find the material they actually provided or worked on.【结构分析】本 句 包 含 一 个 并 列 结 构, 并 列 的 成 分 为There may have been many spies andintelligenceofficersduring the Napoleonic Wars 以 及 itis usuallyextremelydifficult tofind the materialthey actually provided or worked on。其中,第一句为there be句型,第二句为主系表结构, it为形式主语,真正主语为to find...不定式短语,they actually provided or worked on是省略连词 的定语从句,修饰the material。【译点分析】(1) There may have been:为 there be句型,可直接译为“有”或者“存在” 。(2) during the Napoleonic Wars:介词短语做状语,在翻译时候,可置于“there be”前译。(3) it is usually extremely difficult to find ...:此部分为形式主语,在翻译时,可直接译为“形容词” 的是, 比如, 此部分可以翻译为“通常难以 ... ...” 。(4) they actually provided or worked on: 此部分为省略连词的定语从句, 可改写为 material(that/which) they actually provided or worked on 因定从部分较短,在翻译时可整体置于名词前翻译,遂可译为“他们实际提供或者破译的情报资料” 。\n 50、Just as the code breaking has its wider relevance in the struggle for Spain, so his attempts to make his way up the promotion ladder speak volumes about British society.【结构分析 】本 句 为 :比 较 结 构 justas ... , so ... , “ 正 如 ...,... 也 一 样 ”。 to make his way upthe promotion ladder为 to do不定式做后置定语修饰attempts 。about British Society为介词短语, 充当后置定语修饰volumes。【译点分析】(1) code breaking是前文提到的“破译密码” 。(2) relevance 是 relevant 的名词形式,同学们更多知道它“相关”的含义, 其实它还有“价值, 意义” 的意思。(3) in the struggle for Spain 为地点状语,作“对于西班牙之战”理解。(4) make one’s way“向 ... ...前进”,up 表示方向向上,promotion ladder“晋升阶梯”,整体可以理解为“加官进爵” 。(5) volume 本身有“音量”和“ ... ... 的量”的意思,speak volumes for/about sth.为固定搭配,字面意思“关于 ... ...说了很多的量”, 即:“充分说明”,后面加宾语“British society英国社会形态”,理 解为“将英国社会形态展现得淋漓尽致”更为通顺。", "task_type": {"major": ["试题"], "minor": ["考研", "开放题"]}, "domain": ["植物生理学"], "metadata": "year:2022, score:10", "answer_from": "human", "human_verified": false, "copyright": "暂无版权及作者信息"} +{"instruction": "Write an email to a professor at a British university, inviting him/her to organize a team for the international innovation contest to be held in our university. You should write about 100 words on the ANSWER SHEET. Do not use your own name in the email. Use “Li Ming” instead.", "input": "", "output": "Dear Professor Bruce,\nOn behalf of the “International Innovation Contest Committee”, I am writing this letter to invite you to organize a group of competitors for the purpose of participating in the upcoming global innovation competition.\n This contest is due to be held in the Hall of Innovation Center from 2:00 pm to 5:00 pm on May(ne xt Sunday). The arrangements are as follows. Firstly, we will hold an opening ceremony. Then, the competitors from different countries will display their works related to the updated andsophisticated science and technology. Lastly, we will confer awards and certificates on outstanding candidates. We believe you and your team’s participation will surly bring up newsideas and enlightening perspectives.\n Once again, we sincerely hope you and your team can take part in this contest. I am looking forward to your favorable reply.\nSincerely yours,\nLi Ming", "task_type": {"major": ["试题"], "minor": ["考研", "开放题"]}, "domain": ["植物生理学"], "metadata": "year:2022, score:10", "answer_from": "human", "human_verified": false, "copyright": "暂无版权及作者信息"} +{"instruction": "阅读下列材料, 并结合所学知识回答问题。\n\n\n材料一\n\n国初里甲之编, 均其户口, 可举纲以知目。首长之役, 择其望族, 如以臂而运指。意甚善也。然丁产之息耗渐殊, 而更乿之输充弗改, 则轻重之间, 规避自生矣。试按今之图籍, 有一甲之众, 足敌一社者, 尚可分里以役之乎?今之里胥, 有单贫已极者,尚可按籍以定之乎?窃考国典,其里不敷十甲之数,取邻图拨补之;甲不敷一百十户之数, 验丁产归并之, 排年里长设有消乏, 许于一百户内选丁粮近上者充补焉, 抑何其融通乎!——顾炎武《天下郡国利病书》\n\n材料二\n\n鸣呼, 人以户籍为定, 国家之兴, 十年大造, 稽其户之盛衰而升降之, 贼役斯平也, 今版籍之弊, 可为长太息矣! 富者, 丁联千百, 而籍之所人, 乃直数十; 贫者, 匹无, 而籍所载, 不免二三。里长永为里长, 有消无以苏其穷; 甲首永为甲首, 富豪得以避其重。户惟合而不分。且有冒相合者。户丁倍于国初, 繁庶埒于上郡, 而荒落犹夫旧额也, 自有郡至今, 百余年矣。而制度犹若是其草创焉, 何哉?-—嘉靖《思南府志》\n\n问题:\n\n(1)解释材料一加点句子的大意。\n\n(2)概括里甲制度在运行过程中产生的弊端, 并说明明朝政府的解决办法。", "input": "", "output": "(1)明朝的里甲制度规定: 以邻近 110 户为 1 里, 其中丁粮多的 10 户为里长, 其余 100 户分为 10 甲,每甲 10 户。 1 里不足 10 甲时,从邻近的里甲调拨补足;里甲不足 110 户时, 核查人丁、财产与邻里合并。里长中如果有财产贫乏, 允许在 100 户中挑选人丁、土地多的民户补充代替。\n\n(2)弊端:里甲之间贫富差距较大,赋役征收不公; 地主豪民长期占据里长和甲首之位, 得以避免重税; 地方人民相冒合户, 借机逃避和转嫁赋役, 影响国家财政收入。\n\n解决办法: 重新丈量全国土地; 实行\"一条鞭法\",规定:田赋和力役都折银征收;把过去按户按丁征收的力役改为折银征收, 称为户丁银; 归并和简化征收项目,统一编派; 赋役的征收解运由过去的民收民解改为官收官解。", "task_type": {"major": ["试题"], "minor": ["考研", "开放题"]}, "domain": ["历史"], "metadata": "year:2023, score:30", "answer_from": "human", "human_verified": false, "copyright": "暂无版权及作者信息"} +{"instruction": "阅读下列史料, 并结合所学知识回答问题。\n\n\n材料:\n\n棉纺织业最初源于印度, 公元 1200-1800 年, 棉纺织品的制造和消费在世界各地稳步扩大。英国东印度公司直接为印度棉织物开辟了巨大的新市场, 在 17 世纪末 18 世纪初, 东印度公司进口棉织品的数量不断激增, 平均比例占进口总额的 70\\%-80\\%,东方棉织品在英国��发的服饰新潮流却导致了国内毛纺织利益者\n的不满。1700 年, 英国议会颁布禁止法令, \"绝对禁止印度、波斯和中国的印花织物输入; 凡因违法而被扣押的货物, 应予没收、拍卖或再输出\"。虽然政府以法令的手段保护毛纺业, 但东印度公司进口棉织品的数量仍有增无减。18 世纪 30 年代, 化学领域带来的重大发现与机械发明同为英国棉纺织业发展的支柱, 契合了当时英国棉纺织工业的飞速发展。英国的棉制品从 1780 年的 35 万英镑增长到 1800 年的 580 万英镑。 20 年增长了 16 倍。英国的棉制品逐渐取代印度的棉制品, 成为世界市场上的主导产品。在随后一个多世纪里, 英国棉纺织得到持续飞速发展。——摘编自乔治.里耶罗罗《纺织的世界: 1200-1850 年间的全球棉纺织品史》\n问题:\n\n(1)概括工业革命前后英国纺织业兴起的原因。\n\n(2)分别说明英国棉纺织业崛起对英国和世界经济的影响。", "input": "", "output": "(1)原因: 英国拥有广阔的棉花等原料生产产地; 印度棉纺织品输入引起的市场竞争; 国际市场对棉织品的需求量激增, 英国占领了广大的殖民地, 拥有广阔的市场, 发展动力足; 科技的发展和机械发明, 使英国纺织业在较短时间内完成了机械化, 从而实现了产业升级。\n\n(2)对英国: 英国棉纺织业的崛起提高了棉纺织的生产效率, 促进了工业革命的深入发展, 推动了英国资本主义的发展; 促进了英国劳动组织形式和经营方式的改进, 棉纺织业的机械化促进现代工厂制度在英国形成; 进一步促进了英国的海外扩张与英国的崛起。\n\n对世界经济: 英国棉纺织业的崛起真正意义上推动了现代全球市场的形成, 棉纺织业的生产分工与销售等环节在客观上推进了世界各个地区的密切联系, 也带动了各地区之间商品的流通; 改变了世界经济格局, 英国棉纺织业的崛起标志着世界棉花生产中心的转移, 也进一步标志着世界经济重心的转移, 欧洲进一步崛起,亚洲加速衰落; 深刻影响了其他国家的经济发展, 其他欧美国家纷纷学习棉纺技术, 并以此走上工业化道路, 而对于亚非拉国家而言, 它们日益变成最低端的原料产地和棉织品倾销地, 民族工业发展受限。", "task_type": {"major": ["试题"], "minor": ["考研", "开放题"]}, "domain": ["历史"], "metadata": "year:2023, score:30", "answer_from": "human", "human_verified": false, "copyright": "暂无版权及作者信息"} +{"instruction": "概述春秋时期诸侯争霸的主要阶段, 并分析争霸战争的影响。", "input": "", "output": "平王东迁之后, 周王室的势力大为削弱, 一些诸侯国君励精图治, 积极改革内政,以尊王为旗号实际上从事争霸活动, 主要霸主有\"春秋五霸\"。\n\n(一)主要阶段\n\n(1)第一阶段: 王室衰微。平王东迁后, 天子直辖领地锐减, 财政困难, 逐渐丧失对诸侯的控制权,典型表现有周郑交质与周郑交恶。\n\n(2)第二阶段:齐桓公始霸。齐桓公在位时期,任用管仲为相实行内政改革,同时利用渔盐之利发展工商业, 使得国力增强。齐桓公以尊王攘夷为旗号存邢救卫,并于公元前 651 年的葵丘之会上确立了霸主地位。齐桓公死后, 齐国有所衰落,宋襄公谋霸中原, 但在泓之战中失败。\n\n(2)第三阶段: 晋楚争霸。晋文公在位时励精图治, 使晋国跻身大国行列。而楚国在泓之战后也赢得了一些中原诸侯的支持, 晋楚之间争霸不断。公元前 632 年, 晋楚决战城穏, 随后晋文公利用践土之盟确立了霸主地位。公元前 597 年,楚庄王在邲之战大败晋军, 公元前 575 年, 晋国在征陵之战大败楚军。这一时期,秦穆公也欲进挺中原,争夺霸权,但在崤之战中被晋国所败。\n\n(3)第四阶段: 弭兵运动与吴越争霸。公元前 546 年, 宋国第二次提出弭兵, 得到诸侯响应, 约定除齐、秦之外, 共尊晋楚为大国, 以小国利益的牺牲实现了和平, 此后中原诸国军权下移, 开始专注于内部事务, 东南的吴、越继续争霸, 吴王阖问、越王勾践先后图霸, 但时间都较为短暂。\n\n(二)影响\n\n(1)促进了社会发展。为了进行争霸, 各国进行了内政建设的改革, 如齐国管仲改革等, 改革不同程度地发展了生产, 促进了各国实力的进步。\n\n(2)加快了统一全国的步伐。在春秋三百年大国争霸的过程中, 大批小国被强国吞并。春秋初期有 120 余个国家,到春秋末期只剩下大约三分之一。\n\n(3)促进了奴隶制的瓦解。大国争霸过程中, 各国卿大夫的势力得到了加强, 对于奴隶制度的瓦解和新的封建制度的形成有重要作用。\n\n(4)加速了民族大融合。在春秋争霸过程中, \"华夏\"不再只是一个狭义的地理概念,不少少数民族因战争、通婚和迁徙往来而与华夏族融合, 而齐、楚等霸业也为华夏共同体的发展壮大起了重大作用。\n\n(5)争霸战争在一定程度上也给人民带来了灾难, 严重破坏了社会生产。", "task_type": {"major": ["试题"], "minor": ["考研", "开放题"]}, "domain": ["历史"], "metadata": "year:2023, score:40", "answer_from": "human", "human_verified": false, "copyright": "暂无版权及作者信息"} +{"instruction": "论述中世纪德国未能形成统一国家的原因。", "input": "", "output": "中世纪德国是欧洲地区不可忽视的政治力量, 但是在漫长的中世纪历史上, 德意志地区在周边民族与国家相继完成统一的情况下,却长期保持碎片化状态。\n\n(一)政治原因\n\n(1)德皇与教会长期争夺基督教世界领导权, 缺少有利的外部环境。罗马教会同德意志地区相邻, 不愿面对统一帝国的威胁, 积极干涉德意志统一。德皇同教会在叙任权上斗争不断, 且长期居于下风, 严重动摇了皇权的神圣性和权威性。 (2)德皇统治策略失误, 削弱了统一力量。德皇致力于对外扩张, 试图统一意大利, 再现罗马帝国的辉煌。又因罗马教会反对德国统一意大利, 德皇在入侵意大利和同罗马教会的斗争中消耗了自身实力。为取得国内力量的支持, 德皇不断赐予教俗封建主种种特权, 地方诸侯权力膨胀。\n\n(3)德国地方诸侯势力强大, 缺少统一的政治基础。德国封君封臣关系形成迟缓,土地由地方封建主直接占有而非通过国王分封所得。此外, 统一市场的缺乏促使诸侯间结成联盟保护自身经济利益,地方封建势力加强。1356 年颁布的《黄金诏书》,规定皇帝由七大选侯选举产生, 更是标志着德意志分裂割据的合法化。 (二)经济原因\n\n(1)德国封建化迟缓, 阻碍经济发展, 缺少统一的经济基础。德国农村公社长期存在, 阻碍工商业和城市发展, 各地间未建立起强有力的经济联系, 无法形成统一市场, 地区间经济发展不平衡加剧了分裂。\n\n(2)德国在中世纪后期经济逐渐衰落, 难以支撑一个统一国家。德国在中世纪后期经济逐渐落后于英法等国, 德国统治者哈布斯保家族更加重视其治下尼德兰和西班牙等发达地区,忽视了德意志的统一问题。\n\n(三)社会原因\n\n中世纪德国没有形成共同的民族意识, 缺少统一的精神文化基础。德意志国家由多个日耳曼部落组合而成, 部落以血缘关系为纽带, 发展水平不一, 没有共同的语言和文化,没有形成统一的德意志民族,地方势力的分裂倾向极强。", "task_type": {"major": ["试题"], "minor": ["考研", "开放题"]}, "domain": ["历史"], "metadata": "year:2023, score:40", "answer_from": "human", "human_verified": false, "copyright": "暂无版权及作者信息"} +{"instruction": "论述 20 世纪 50 年代中国优先发展重工业战略形成的历史背景及意义。", "input": "", "output": "新中国成立后, 面对严峻的外部形势。基于我国重工业落后的现实因素, 党中央在 50 年代初确立了优先发展重工业的战略。该战略适应了当时中国的国情, 为建立独立自主的工业体系打下了基础。\n\n(一)历史背景\n\n(1)中国旧有的重工业基础薄弱。中国长期是一个落后的农业国, 近代以来, 我国工业发展主要集中于轻工业, 重工业发展较为落后。而重工业在国民经济建设中具有先导性作用, 可以为国民经济各部门提供生产资料。\n\n(2)新生的人民政权面临严峻的外部形势。20世纪 50 年代初, 败退台湾的国民党军队及美国舰队不时袭扰我国沿海地区; 抗美援朝战争时中国军队装备鿒乏, 凸显了发展重工业的重要性和紧迫性。优先发展重工业战略是我国军事和国防发展的特殊需求。\n\n(3)苏联优先发展重工业取得了较大成功, 为中国的工业化建设提供了借鉴。苏联在社会主义建设中, 优先发展重工业, 在较短时间里实现了社会主义工业化。中国“一五计划”的制定就受到苏联的直接影响, 以重工业建设为重点, 且苏联的援华项目多为重工业。\n\n(4)新中国成立后, 经过三年恢复时期, 人民民主专政得到了巩固, 国家财政经济状况有了根本好转, 这为优先发展重工业创造了政治和经济条件。\n\n(二)意义\n\n(1)为社会主义工业化奠定了基础。通过大规模的重工业建设, 建成了当时门类\n\n较为齐全的重工业体系, 开始改变我国落后的工业面貌, 同时培养了一批工业技术人才, 积累了现代工业建设经验, 为社会主义工业化奠定了初步基础。\n\n(2)促进了“一五计划”和“三大改造”的顺利完成, 为我国建立独立自主的工业体系打下了基础, 也奠定了社会主义建设的经济基础。\n\n(3)保证了国家独立, 维护了新生政权, 提高了中国的国际地位。前所未有的重工业部门的创建, ���高了中国的国防和军事力量, 保证了国家的安全和主权完整,也提高了我国在国际社会的地位。\n\n(4)巩固了中国共产党的领导。\"一五\"期间优先发展重工业的成就使中国共产党的执政能力得到检验, 从而为新生政权筑牢了政治根基, 也为此后社会主义道路探索减轻了阻碍。", "task_type": {"major": ["试题"], "minor": ["考研", "开放题"]}, "domain": ["历史"], "metadata": "year:2023, score:40", "answer_from": "human", "human_verified": false, "copyright": "暂无版权及作者信息"} +{"instruction": "论述第二次世界大战后非洲独立国家经济发展道路的异同。", "input": "", "output": "第二次世界大战后, 在世界范围内的去殖民化冲击下, 非洲大部分国家获得了独立地位,\n开始大力发展经济,但非洲各国经济发展道路既有相同点也有不同点。 (一)相同\n\n(1)道路选择都面临着漫长的殖民历史与殖民包袱。二战后独立的非洲各国, 都面临着殖民包补, 他们经济基础薄弱, 大量依赖外资, 国家重要资源与经济基础设施多被英法美等外国资本控制。\n\n(2)都受到外部势力的影响。非洲独立国家经济发展面临着外部影响与大国干涉,例如, 冷战背景下英法美苏等大国将非洲作为各自的势力范围, 如何在大国夹缝下谋得生存是非洲各国经济发展面临的统一问题。\n\n(3)都面临着大量相同的困难。非洲落后的基础设施建设, 较低的识字率, 大量的贫困等因素都制约了非洲独立各国经济发展道路的选择、最终走向和结果。 (4)发展经历了相似的历程。非洲国家在独立之初到 20 世纪 80 年代经济发展普遍增速快, 经济年平均增长率达百分之五, 在 80 年代到 90 年代, 非洲经济缓慢增长, 甚至一些国家出现负增长, 整体而言非洲独立国家经济发展水平较低。 (二)不同\n\n(1)发展具体道路不一。西非和南非地区长期受殖民因素影响, 即使在独立后,依然难以摆脱英法等殖民国家的影响, 因此主要走上资本主义发展道路。而东非和北非地区受到苏联影响较大, 相继走上了社会主义道路。还有不少非洲国家选择结合资本主义和社会主义的\"第三条道路\"的发展模式,代表有埃及、多哥和几内亚等。\n\n(2)受传统因素的影响不同。部分非洲国家由于受传统影响较大, 依旧是以传统部落制生产方式为主, 经济发展长期滞后, 例如冈比亚。\n\n(3)与原殖民国之间的关系远近不同。个别国家仍深受殖民国的影响,如阿尔及利亚地区独立后各国, 在经济、政治和社会文化上深受法国影响, 其经济发展道路依旧是依附于法国发展, 走上了资本主义发展道路, 主要是作为法国的原材料产地和市场。\n\n(4)发展成果不一, 贫富差距较大。撒哈拉以南非洲长期和欧洲以及中东等发达国家经济联系密切, 经济发展水平较高, 而撒哈拉以南非洲除南非以外, 大部分国家经济发展水平较低,东非更长期是非洲最贫穷的地区。\n\n(5)经济发展基础不一。非洲各独立国家虽然都经历了漫长的殖民历史, 但是例如南非等国家, 白人殖民者占据主流, 在殖民时期经济发展水平较高, 独立后经济发展依旧维持高速增长, 而例如中南非洲的其他国家, 曾长期作为西方的原材料产地, 独立后经济基础差, 经济发展面临结构性问题。", "task_type": {"major": ["试题"], "minor": ["考研", "开放题"]}, "domain": ["历史"], "metadata": "year:2023, score:40", "answer_from": "human", "human_verified": false, "copyright": "暂无版权及作者信息"} +{"instruction": "论说文: 根据下述材料, 写一篇 700 字左右的论说文, 题目自拟。\n人们常说“领导艺术”,可见领导与艺术之间存在着某种相似点,如领导一个团队完成某项任务和指挥一个乐队演奏某首乐曲一样。", "input": "", "output": "题目:管理者要有领导艺术\n\n如同指挥一个乐队一样,管理者也应该有领导艺术。拥有领导艺术, 有助于激发工作热情。领导艺术很多时候是来源于领导者的威信、经验和才能的, 它能够潜移默化地作用于下属的工作和生活之中, 让下属产生一种敬佩感。这种敬佩感就像磁铁一样吸引着下属, 让他们自觉自愿地接受管理者的思想、行为方式, 从心底对管理者产生认同感和尊重, 从而使团队成员求同存异、心齐力合, 从而增强团队凝聚力。拥有领导艺术, 有助于产生协同效应。我们都知道, 乐队想要奏出美妙的乐曲, 需要钢琴、小提琴、大提琴等等不同乐配的配合。团队也是一样,越是庞大的团队、越是杂复的工作,越需要不同人之间的协调配合。但是, 这些人由于职位不同、专业不同、学识不同、性格不同、习惯不同, 对工作的认知也会产生不同, 此时, 就需要拥有���导艺术的管理者来协调这些不同的人之间的关系,让其各扬所长协同工作。当然,领导艺术的建立,并没有那么容易。因为,管理既有科学的一面,也有艺术的一面,这就对管理者的综合能力提出了较高的要求。管理者既要有足够的专业知识和管理才能, 又要懂得人与人的相处之道。那么,管理者应该如何形成自己的领导艺术呢? 我认为要做好以下两点:第一,要利用好权力领导力。管理者拥有对下属的奖惩权等职位权力,充分利用好这些权力,可以让下属的行为更加符合组织的预期,这是领导艺术的基础。第二, 要培养好非权力领导力。也就是说, 要通过管理者自身的过人人品、学识、能力、业绩, 潜移默化的影响下属, 形成对下属的感召力, 让下属从心底里认同和尊重自己。总之, 如果优秀的乐队需要优秀的指挥一样, 一个优秀的团队也必然需要一个管理艺术高超的管理者。管理者应该注重管理艺术。\n\n\n分析:\n\n您可以参考以下立意来写:\n\n(1)管理者要有领导艺术。\n(2)管理者应学会协调指挥。\n(3)管理要有艺术性。", "task_type": {"major": ["试题"], "minor": ["考研", "开放题"]}, "domain": ["管理学"], "metadata": "year:2023, score:35", "answer_from": "human", "human_verified": false, "copyright": "暂无版权及作者信息"} +{"instruction": "求极限 $\\lim _{x \\rightarrow 0}\\left(\\frac{1+\\int_{0}^{x} e^{t^{2}} d t}{e^{x}-1}-\\frac{1}{\\sin x}\\right)$.", "input": "", "output": "$\\frac{1}{2}$.\n\n\n分析:\n\n$\\lim _{x \\rightarrow 0}\\left(\\frac{1+\\int_0^x e^{t^2} d t}{e^x-1}-\\frac{1}{\\sin x}\\right)=\\lim _{x \\rightarrow 0} \\frac{\\sin x-1-\\int_0^x e^{t^2} d t}{\\left(e^x-1\\right) \\sin x}$\n又因为 $\\int_0^x e^{t^2} d t=\\int_0^x\\left(1+t^2+o\\left(t^2\\right)\\right) d t=x+\\frac{1}{3} x^3+o\\left(x^3\\right)$, 故\n$$\n\\begin{aligned}\n\\text { 原式 } & =\\lim _{x \\rightarrow 0} \\frac{\\left(x-\\frac{1}{3 !} x^3+o\\left(x^3\\right)\\right)\\left(1+x+\\frac{1}{3 !} x^3+o\\left(x^3\\right)\\right)-x-\\frac{1}{2} x^2+o\\left(x^2\\right)}{x^2} \\\\\n& =\\lim _{x \\rightarrow 0} \\frac{\\frac{1}{2} x^2+o\\left(x^2\\right)}{x^2}=\\frac{1}{2} .\n\\end{aligned}\n$$", "task_type": {"major": ["试题"], "minor": ["考研", "开放题"]}, "domain": ["数学"], "metadata": "year:2021, score:10", "answer_from": "human", "human_verified": false, "copyright": "暂无版权及作者信息"} +{"instruction": "设 $u_{n}(x)=e^{-n x}+\\frac{1}{n(n+1)} x^{n+1}(n=1,2, \\ldots)$, 求级数 $\\sum_{n=1}^{\\infty} u_{n}(x)$ 的收敛域及和函数.", "input": "", "output": "$S(x)=\\left\\{\\begin{array}{l}\\frac{e^{-x}}{1-e^{-x}}+(1-x) \\ln (1-x)+x, x \\in(0,1) \\\\ \\frac{e}{e-1}, x=1\\end{array}\\right.$.\n\n分析:\n\n$S(x)=\\sum_{n=1}^{\\infty} u_{n}(x)=\\sum_{n=1}^{\\infty}\\left[e^{-n x}+\\frac{1}{n(n+1)} x^{n+1}\\right]$, 收敛域 $(0,1], S_{1}(x)=\\sum_{n=1}^{\\infty} e^{-n x}=\\frac{e^{-x}}{1-e^{-x}}, x \\in(0,1]$\n$S_{2}(x)=\\sum_{n=1}^{\\infty} \\frac{1}{n(n+1)} x^{n+1}=\\sum_{n=1}^{\\infty} \\frac{x^{n+1}}{n}-\\sum_{n=1}^{\\infty} \\frac{x^{n+1}}{n+1}=-x \\ln (1-x)-[-\\ln (1-x)-x]$\n$=(1-x) \\ln (1-x)+x, \\quad x \\in(0,1)$\n$S_{2}(1)=\\lim _{x \\rightarrow 1^{-}} S_{2}(x)=1$\n$S(x)=\\left\\{\\begin{array}{l}\\frac{e^{-x}}{1-e^{-x}}+(1-x) \\ln (1-x)+x, x \\in(0,1) \\\\ \\frac{e}{e-1}, x=1\\end{array}\\right.$", "task_type": {"major": ["试题"], "minor": ["考研", "开放题"]}, "domain": ["数学"], "metadata": "year:2021, score:12", "answer_from": "human", "human_verified": false, "copyright": "暂无版权及作者信息"} +{"instruction": "已知曲线 $C:\\left\\{\\begin{array}{l}x^{2}+2 y^{2}-z=6 \\\\ 4 x+2 y+z=30\\end{array}\\right.$, 求 $C$ 上的点到 $x o y$ 坐标面距离的最大值.", "input": "", "output": "66\n\n分析:\n\n设拉格朗日函数 $L(x, y, z, \\lambda, \\mu)=z^2+\\lambda\\left(x^2+2 y^2-z-6\\right)+\\mu(4 x+2 y+z-30)$\n$$\n\\begin{aligned}\n& L_x^{\\prime}=2 x \\lambda+4 u=0 \\\\\n& L^{\\prime} y=4 y \\lambda+2 u=0 \\\\\n& L_z^{\\prime}=2 z-\\lambda+u=0 \\\\\n& x^2+2 y^2-z=6 \\\\\n& 4 x+2 y+z=30\n\\end{aligned}\n$$\n\n解得驻点: $(4,1,12),(-8,-2,66)$\n$\\mathrm{C}$ 上的点 $(-8,-2,66)$ 到 xoy 面距离最大为 66", "task_type": {"major": ["试题"], "minor": ["考研", "开放题"]}, "domain": ["数学"], "metadata": "year:2021, score:12", "answer_from": "human", "human_verified": false, "copyright": "暂无版权及作者信息"} +{"instruction": "设 $D \\subset R^{2}$ 是有界单连通闭区域, $I(D)=\\iint_{D}\\left(4-x^{2}-y^{2}\\right) d x d y$ 取得最大值的积分区域记为 $D_{1}$.\n(1)求 $I\\left(D_{1}\\right)$ 的值.\n(2)计算 $\\int_{\\partial D_{1}} \\frac{\\left(x e^{x^{2}+4 y^{2}}+y\\right) d x+\\left(4 y e^{x^{2}+4 y^{2}}-x\\right) d y}{x^{2}+4 y^{2}}$, 其中 $\\partial D_{1}$ 是 $D_{1}$ 的正向边界.", "input": "", "output": "$-\\pi$.\n\n分析:\n\n(1) 由二重积分的几何意义知: $I(\\mathrm{D})=\\iint_D\\left(4-x^2-y^2\\right) d \\sigma$, 当 且仅当 $4-x^2-y^2$ 在 $D$ 上大于 0 时, $I(\\mathrm{D})$ 达到最大, 故 $D_1: x^2+y^2 \\leq 4$ 且 $I\\left(\\mathrm{D}_1\\right)=\\int_0^{2 \\pi} d \\theta \\int_0^2\\left(4-r^2\\right) r d r=8 \\pi$.\n(2) 补 $D_2: x^2+4 y^2=r^2$ ( $r$ 很小), 取 $D_2$ 的方向为顺时针方向,\n$$\n\\begin{aligned}\n\\int_{\\partial D_1} \\frac{\\left(x e^{x^2+4 y^2}+y\\right) d x+\\left(4 y e^{x^2+4 y^2}-x\\right) d y}{x^2+4 y^2} \\\\\n& =\\int_{\\partial D_1+\\partial D_2} \\frac{\\left(x e^{x^2+4 y^2}+y\\right) d x+\\left(4 y e^{x^2+4 y^2}-x\\right) d y}{x^2+4 y^2}-\\int_{\\partial D_2} \\frac{\\left(x e^{x^2+4 y^2}+y\\right) d x+\\left(4 y e^{x^2+4 y^2}-x\\right) d y}{x^2+4 y^2} \\\\\n& =-\\frac{1}{r^2} e^{r^2} \\int_{\\partial D_2} x d x+4 y d y-\\frac{1}{r^2} e^{r^2} \\int_{\\partial D_2} y d x-x d y=\\frac{1}{r^2} \\iint_{D_2}-2 d \\sigma=-\\pi .\n\\end{aligned}\n$$", "task_type": {"major": ["试题"], "minor": ["考研", "开放题"]}, "domain": ["数学"], "metadata": "year:2021, score:12", "answer_from": "human", "human_verified": false, "copyright": "暂无版权及作者信息"} +{"instruction": "已知 $A=\\left(\\begin{array}{ccc}a & 1 & -1 \\\\ 1 & a & -1 \\\\ -1 & -1 & a\\end{array}\\right)$.\n(1)求正交矩阵 $P$, 使得 $P^{T} A P$ 为对角矩阵;\n(2)求正定矩阵 $C$, 使得 $C^{2}=(a+3) E-A$.", "input": "", "output": "(1) $P=\\left(\\begin{array}{ccc}\\frac{1}{\\sqrt{3}} & -\\frac{1}{\\sqrt{2}} & -\\frac{1}{\\sqrt{6}} \\\\ \\frac{1}{\\sqrt{3}} & \\frac{1}{\\sqrt{2}} & \\frac{1}{\\sqrt{6}} \\\\ -\\frac{1}{\\sqrt{3}} & 0 & \\frac{2}{\\sqrt{6}}\\end{array}\\right)$; (2) $C=\\left(\\begin{array}{ccc}\\frac{5}{3} & -1 & -1 \\\\ -1 & \\frac{5}{3} & \\frac{1}{3} \\\\ -1 & \\frac{1}{3} & \\frac{5}{3}\\end{array}\\right)$.\n\n分析:\n\n(1) 由 $|\\lambda E-A|=\\left|\\begin{array}{ccc}\\lambda-a & -1 & 1 \\\\ -1 & \\lambda-a & 1 \\\\ 1 & 1 & \\lambda-a\\end{array}\\right|=(\\lambda-a+1)^{2}(\\lambda-a-2)=0$\n得 $\\lambda_{1}=a+2, \\lambda_{2}=\\lambda_{3}=a-1$\n当 $\\lambda_{1}=a+2$ 时\n$((a+2) E-A)=\\left(\\begin{array}{ccc}2 & -1 & 1 \\\\ -1 & 2 & 1 \\\\ 1 & 1 & 2\\end{array}\\right) \\stackrel{r}{\\rightarrow}\\left(\\begin{array}{lll}1 & 0 & 1 \\\\ 0 & 1 & 1 \\\\ 0 & 0 & 0\\end{array}\\right)$ 的特征向量为 $\\alpha_{1}=\\left(\\begin{array}{c}1 \\\\ 1 \\\\ -1\\end{array}\\right)$,\n当 $\\lambda_{2}=\\lambda_{3}=a-1$ 所\n$((a-1) E-A)=\\left(\\begin{array}{ccc}-1 & -1 & 1 \\\\ -1 & -1 & 1 \\\\ 1 & 1 & -1\\end{array}\\right) \\stackrel{\\underset{\\sim}{1}}{r}\\left(\\begin{array}{ccc}1 & -1 \\\\ 0 & 0 & 0 \\\\ 0 & 0 & 0\\end{array}\\right)$ 的特征向量为 $\\alpha_{2}=\\left(\\begin{array}{c}-1 \\\\ 1 \\\\ 0\\end{array}\\right), \\alpha_{3}=\\left(\\begin{array}{c}-1 \\\\ 1 \\\\ 2\\end{array}\\right)$,\n令 $P=\\left(\\frac{\\alpha_{1}}{\\left|\\alpha_{1}\\right|}, \\frac{\\alpha_{2}}{\\left|\\alpha_{2}\\right|}, \\frac{\\alpha_{3}}{\\left|\\alpha_{3}\\right|}\\right)=\\left(\\begin{array}{ccc}\\frac{1}{\\sqrt{3}} & -\\frac{1}{\\sqrt{2}} & -\\frac{1}{\\sqrt{6}} \\\\ \\frac{1}{\\sqrt{3}} & \\frac{1}{\\sqrt{2}} & \\frac{1}{\\sqrt{6}} \\\\ -\\frac{1}{\\sqrt{3}} & 0 & \\frac{2}{\\sqrt{6}}\\end{array}\\right)$, 则 $P^{T} A P=\\Lambda=\\left(\\begin{array}{lll}a+2 & & \\\\ & a-1 & \\\\ & & a-1\\end{array}\\right)$,\n(2) $\\left.P^{T} C^{2} P=P^{T}(a+3) E-A\\right) P=\\left((a+3) E-\\Lambda=\\left(\\begin{array}{lll}1 & & \\\\ & 4 & \\\\ & & 4\\end{array}\\right)\\right.$\n$\\Rightarrow P^{T} C P P^{T} C P=\\left(\\begin{array}{lll}1 & & \\\\ & 4 & \\\\ & & 4\\end{array}\\right) \\Rightarrow P^{T} C P=\\left(\\begin{array}{lll}1 & & \\\\ & 2 & \\\\ & & 2\\end{array}\\right)$,\n故 $C=P\\left(\\begin{array}{lll}1 & & \\\\ & 2 & \\\\ & & 2\\end{array}\\right) P^{T}=\\left(\\begin{array}{ccc}\\frac{5}{3} & -1 & -1 \\\\ -1 & \\frac{5}{3} & \\frac{1}{3} \\\\ -1 & \\frac{1}{3} & \\frac{5}{3}\\end{array}\\right)$.", "task_type": {"major": ["试题"], "minor": ["考研", "开放题"]}, "domain": ["数学"], "metadata": "year:2021, score:12", "answer_from": "human", "human_verified": false, "copyright": "暂无版权及作者信息"} +{"instruction": "在区间 $(0,2)$ 上随机取一点, 将该区间分成两段, 较短的一段长度记为 $X$, 较长的一段长度记为 $Y$, 令 $Z=\\frac{Y}{X}$.\n(1)求 $X$ 的概率密度;\n(2)求 $Z$ 的概率密度.\n(3)求 $E\\left(\\frac{X}{Y}\\right)$.", "input": "", "output": "(1) $X \\sim f(x)=\\left\\{\\begin{array}{c}1,0a, F(x)<0$, 则 $F(b)<0$, 及 $f\\left(\\frac{a+b}{2}\\right) \\leq \\frac{1}{b-a} \\int_a^b f(x) d x$.", "task_type": {"major": ["试题"], "minor": ["考研", "开放题"]}, "domain": ["数学"], "metadata": "year:2022, score:12", "answer_from": "human", "human_verified": false, "copyright": "暂无版权及作者信息"} +{"instruction": "设二次型 $f\\left(x_{1}, x_{2}, x_{3}\\right)=\\sum_{i=1}^{3} \\sum_{j=1}^{3} i j x_{i} x_{j}$.\n(1) 求二次型矩阵\n(2) 求正交矩阵 $\\boldsymbol{Q}$, 使得二次型经正交变换 $\\boldsymbol{x}=\\boldsymbol{Q} \\boldsymbol{y}$ 化为标准形\n(3) 求 $f\\left(x_{1}, x_{2}, x_{3}\\right)=0$ 的解", "input": "", "output": "(1) 据题意, $f\\left(x_1, x_2, x_3\\right)=\\sum_{i=1}^3 \\sum_{j=1}^3 i j x_i x_j=x_1{ }^2+4 x_2{ }^2+9 x_3{ }^2+4 x_1 x_2+6 x_1 x_3+12 x_2 x_3$,\n\n故 $\\boldsymbol{A}=\\left(\\begin{array}{lll}1 & 2 & 3 \\\\ 2 & 4 & 6 \\\\ 3 & 6 & 9\\end{array}\\right)$.\n(2)易得 $\\boldsymbol{A}$ 的特征值 为 $14,0,0$.\n当 $\\lambda_1=14$ 时, ��� $(A-14 E) x=0$, 由\n$$\n\\boldsymbol{A}-14 \\boldsymbol{E}=\\left(\\begin{array}{ccc}\n-13 & 2 & 3 \\\\\n2 & -10 & 6 \\\\\n3 & 6 & -5\n\\end{array}\\right) \\rightarrow\\left(\\begin{array}{ccc}\n1 & -5 & 3 \\\\\n0 & -63 & 42 \\\\\n0 & 21 & -14\n\\end{array}\\right) \\rightarrow\\left(\\begin{array}{ccc}\n1 & -5 & 3 \\\\\n0 & 3 & -2 \\\\\n0 & 0 & 0\n\\end{array}\\right)\n$$\n\n得 $\\lambda_1=14$ 对应的特征值为 $\\alpha_1=(1,2,3)^{\\mathrm{T}}$.\n\n当 $\\lambda_2=\\lambda_3=0$ 时, 解 $\\boldsymbol{A x}=\\mathbf{0}$, 得 $\\lambda_2=\\lambda_3=0$ 对应的特征值为 $\\alpha_2=(-2,1,0)^{\\mathrm{T}}$ 和 $\\alpha_3=(-3,0,1)^{\\mathrm{T}}$.\n\n实对称矩阵不同特征值对应的特征向量正交, 故只需将 $\\alpha_2, \\alpha_3$ 正交化, 得\n$$\n\\xi_2=(-2,1,0)^{\\mathrm{T}}, \\quad \\xi_3=(-3,-6,5)^{\\mathrm{T}} .\n$$\n\n将 $\\alpha_1, \\xi_2, \\xi_3$ 单位化, 得\n$$\n\\gamma_1=\\frac{1}{\\sqrt{14}}(1,2,3)^{\\mathrm{T}} \\quad \\gamma_2=\\frac{1}{\\sqrt{5}}(-2,1,0)^{\\mathrm{T}} \\quad \\gamma_3=\\frac{1}{\\sqrt{70}}(-3,-6,5)^{\\mathrm{T}} .\n$$\n\n令 $\\boldsymbol{Q}=\\left(\\gamma_1, \\gamma_2, \\gamma_3\\right)$, 经正交变换 $\\boldsymbol{x}=\\boldsymbol{Q} \\boldsymbol{y}$, 将 $f$ 化为标 准形 $14 y_1{ }^2$.\n\n(3)在正交变换 $\\boldsymbol{x}=\\boldsymbol{Q} \\boldsymbol{y}$ 下, $f\\left(x_1, x_2, x_3\\right)$ 化为 $14 y_1{ }^2$. 由 $f\\left(x_1, x_2, x_3\\right)=0$, 得 $y_1=0$, 则\n\n$\\boldsymbol{x}=\\left(\\gamma_1, \\gamma_2, \\gamma_3\\right)\\left(\\begin{array}{c}0 \\\\ y_2 \\\\ y_3\\end{array}\\right)=y_2 \\gamma_2+y_3 \\gamma_3=k_1(-2,1,0)^{\\mathrm{T}}+k_2(-3,-6,5)^{\\mathrm{T}}$, 其中 $k_1, k_2$ 为任意常数.", "task_type": {"major": ["试题"], "minor": ["考研", "开放题"]}, "domain": ["数学"], "metadata": "year:2022, score:12", "answer_from": "human", "human_verified": false, "copyright": "暂无版权及作者信息"} +{"instruction": "设 $X_{1}, X_{2}, \\cdots, X_{n}$ 是来自期望为 $\\theta$ 的指数分布的简单随机样本, $Y_{1}, Y_{2}, \\cdots, Y_{m}$ 是来自期望为 $2 \\theta$ 的指数分布的简单随机样本, 且 $X_{1}, X_{2}, \\cdots, X_{n}, Y_{1}, Y_{2}, \\cdots, Y_{m}$ 相互独立, 求 $\\theta$ 的最大似然估计量 $\\hat{\\theta}$, 及 $D(\\hat{\\theta})$.", "input": "", "output": "由已知 $E(X)=\\theta=\\frac{1}{\\lambda_1} \\Rightarrow \\lambda_1=\\frac{1}{\\theta}, E(Y)=2 \\theta=\\frac{1}{\\lambda_2} \\Rightarrow \\lambda_2=\\frac{1}{2 \\theta}$,\n\n所以总体 $X \\sim E\\left(\\frac{1}{\\theta}\\right), Y \\sim E\\left(\\frac{1}{2 \\theta}\\right)$, 从而可得\n$$\nf_X(x)=\\left\\{\\begin{array}{ll}\n\\frac{1}{\\theta} \\mathrm{e}^{-\\frac{x}{\\theta}}, & x>0, \\\\\n0, & x \\leq 0 .\n\\end{array} \\quad f_Y(y)= \\begin{cases}\\frac{1}{2 \\theta} \\mathrm{e}^{-\\frac{y}{2 \\theta}}, & y>0, \\\\\n0, & y \\leq 0 .\\end{cases}\\right.\n$$\n\n设 $x_1, x_2, \\cdots, x_n, y_1, y_2, \\cdots, y_m$ 为样本 $X_1, X_2, \\cdots, X_n, Y_1, Y_2, \\cdots, Y_m$ 的观测值, 且样本相互独立, 则 似然函数为\n$$\nL(\\theta)= \\begin{cases}\\frac{1}{2^m} \\frac{1}{\\theta^{n+m}} \\mathrm{e}^{-\\frac{2 \\sum_{i=1}^n x_j+\\sum_{j=1}^n y_j}{2 \\theta}}, & x_i, y_j>0(i=1,2, \\cdots, n ; j=1,2, \\cdots, m), \\\\ 0, & \\text { 其它. }\\end{cases}\n$$\n\n当 $x_1, x_2, \\cdots, x_n, y_1, y_2, \\cdots, y_m>0$ 时, 似然函数两边取对数\n\n$$\n\\ln L(\\theta)=-m \\ln 2-(n+m) \\ln \\theta-\\frac{2 \\sum_{i=1}^n x_i+\\sum_{j=1}^m y_j}{2 \\theta},\n$$\n\n令 $\\frac{\\mathrm{d} \\ln L(\\theta)}{\\mathrm{d} \\theta}=-\\frac{n+m}{\\theta}+\\frac{2 \\sum_{i=1}^n x_i+\\sum_{j=1}^m y_j}{2 \\theta^2}=0$, 解得 $\\theta=\\frac{2 \\sum_{i=1}^n x_i+\\sum_{j=1}^m y_j}{2(n+m)}$,\n故 $\\theta$ 的最大似然估计量为 $\\hat{\\theta}=\\frac{2 \\sum_{i=1}^n X_i+\\sum_{j=1}^m Y_j}{2(n+m)}$.\n\n由 $X \\sim E\\left(\\frac{1}{\\theta}\\right), Y \\sim E\\left(\\frac{1}{2 \\theta}\\right)$, 则 $D(X)=\\theta^2, D(Y)=4 \\theta^2$,\n则 $D(\\hat{\\theta})=\\frac{1}{4(n+m)^2} D\\left(2 \\sum_{i=1}^n X_i+\\sum_{j=1}^m Y_j\\right)=\\frac{1}{4(n+m)^2}\\left(4 n \\cdot \\theta^2+m \\cdot 4 \\theta^2\\right)=\\frac{\\theta^2}{n+m}$.", "task_type": {"major": ["试题"], "minor": ["考研", "开放题"]}, "domain": ["数学"], "metadata": "year:2022, score:12", "answer_from": "human", "human_verified": false, "copyright": "暂无版权及作者信息"} +{"instruction": "设曲线 $y=y(x)(x>0)$ 经过点 $(1,2)$, 该曲线上任一点 $P(x, y)$ 到 $y$ 轴的距离等于该点处的切线在 $y$ 轴上的截距.\n(I) 求 $y(x)$;\n(II) 求函数 $f(x)=\\int_{1}^{x} y(t) d t$ 在 $(0,+\\infty)$ 上的最大值.", "input": "", "output": "(I) 设点 $(x, y)$ 处的切线方程为 $Y-y=y^{\\prime}(X-x)$, 故 $y$ 轴的截距为 $y-y^{\\prime} x$, 则 $x=y-y^{\\prime} x$,\n解得 $y=x(C-\\ln x)$, 其中 $C$ 为任意常数.\n由 $y(1)=C=2$, 故 $y(x)=x(2-\\ln x)$.\n(II) 由(I) 知 $f(x)=\\int_{1}^{x} t(2-\\ln t) d t$, 故 $f^{\\prime}(x)=x(2-\\ln x)=0$, 则驻点为 $x=e^{2}$.\n当 $00$; 当 $x>e^{2}$ 时, $f^{\\prime}(x)<0$, 故 $f(x)$ 在 $x=e^{2}$ 处取得极大值,\n同时也取得最大值, 且最大值为 $f\\left(e^{2}\\right)=\\int_{1}^{e^{2}} x(2-\\ln x) d x=\\frac{1}{4} e^{4}-\\frac{5}{4}$.", "task_type": {"major": ["试题"], "minor": ["考研", "开放题"]}, "domain": ["数学"], "metadata": "year:2023, score:10", "answer_from": "human", "human_verified": false, "copyright": "暂无版权及作者信息"} +{"instruction": "求函数 $f(x, y)=\\left(y-x^{2}\\right)\\left(y-x^{3}\\right)$ 的极值.", "input": "", "output": "$\\left\\{\\begin{array}{l}f_{x}^{\\prime}=-x\\left(2 y+3 x y-5 x^{3}\\right)=0 \\\\ f_{y}^{\\prime}=2 y-x^{2}-x^{3}=0\\end{array}\\right.$, 得驻点为 $(0,0),(1,1),\\left(\\frac{2}{3}, \\frac{10}{27}\\right)$.\n$f_{x x}^{\\prime \\prime}=-\\left(2 y+3 x y-5 x^{3}\\right)-x\\left(3 y-15 x^{2}\\right), f_{x y}^{\\prime \\prime}=-x(2+3 x), \\quad f_{y y}^{\\prime \\prime}=2$.\n代入 $(0,0),\\left\\{\\begin{array}{l}A=f_{x x}^{\\prime \\prime}=0 \\\\ B=f_{x y}^{\\prime \\prime}=0, \\\\ C=f_{y y}^{\\prime \\prime}=2\\end{array}\\right.$ 则 $A C-B^{2}=0$, 故充分条件失效, 当 $x \\rightarrow 0$ 时, 取\n$y=x^{2}+k x^{3}(k>0), \\quad f(x, y)=\\left(y-x^{2}\\right)\\left(y-x^{3}\\right)=k x^{3}\\left[x^{2}+(k-1) x^{3}\\right]=k x^{5}+o\\left(x^{5}\\right)$,\n则 $\\lim _{x \\rightarrow 0} \\frac{f(x, y)}{x^{5}}=\\lim _{x \\rightarrow 0} \\frac{k x^{5}+o\\left(x^{5}\\right)}{x^{5}}=k>0$, 由极限的局部保号性: 存在 $\\delta>0$, 当 $x \\in(-\\delta, 0)$ 时, $\\frac{f(x, y)}{x^{5}}>0, f(x, y)<0=f(0,0)$, 当 $x \\in(0, \\delta)$ 时, $\\frac{f(x, y)}{x^{5}}>0$, $f(x, y)>0=f(0,0)$, 故 $(0,0)$ 不是极值点;\n代入 $(1,1),\\left\\{\\begin{array}{l}A=f_{x x}^{\\prime \\prime}=12 \\\\ B=f_{x y}^{\\prime \\prime}=-5, \\\\ C=f_{y y}^{\\prime \\prime}=2\\end{array}\\right.$ 则 $A C-B^{2}<0$, 故 $(1,1)$ 不是极值点;\n代入 $\\left(\\frac{2}{3}, \\frac{10}{27}\\right)$ 的 $\\left\\{\\begin{array}{l}A=f_{x x}^{\\prime \\prime}=\\frac{100}{27} \\\\ B=f_{x y}^{\\prime \\prime}=-\\frac{8}{3}, \\\\ C=f_{y y}^{\\prime \\prime}=2\\end{array}\\right.$ 则 $A C-B^{2}>0$ 且 $A>0$, 故 $\\left(\\frac{2}{3}, \\frac{10}{27}\\right)$ 是极小值点;故 $f\\left(\\frac{2}{3}, \\frac{10}{27}\\right)=-\\frac{4}{729}$ 为极小值.", "task_type": {"major": ["试题"], "minor": ["考研", "开放题"]}, "domain": ["数学"], "metadata": "year:2023, score:12", "answer_from": "human", "human_verified": false, "copyright": "暂无版权及作者信息"} +{"instruction": "设空间有界区域 $\\Omega$ 中, 柱面 $x^2+y^2=1$ 与平面 $z=0$ 和 $x+z=1$ 围成, $\\Sigma$ 为 $\\Omega$ 边界的外侧, 计算曲面积分\n$$\nI=\\oiiint_{\\Sigma} 2 x z d y d z+x z \\cos y d z d y+3 y z \\sin x d x d y .\n$$", "input": "", "output": "由高斯公式可得:\n$$\n\\begin{aligned}\n& =\\iiint_{\\Omega}(2 z-x z \\sin y+3 y \\sin x) d V \\\\\n& =\\iiint_{\\Omega} 2 z d V=\\iint_{D_{x y}} d x d y \\int_0^{1-x} 2 z d z=\\iint_{D_{x y}}(1-x)^2 d x d y \\quad\\left(D_{x y} ; x^2+y^2 \\leq 1\\right) \\\\\n& =\\iint_{D_{x y}}\\left(1-2 x+x^2\\right) d x d y=\\pi+\\frac{1}{2} \\iint_{D_{x y}}\\left(x^2+y^2\\right) d x d y \\\\\n& =\\pi+\\frac{1}{2} \\int_0^{2 \\pi} d \\theta \\int_0^1 r^3 d r=\\pi+\\frac{\\pi}{4}=\\frac{5 \\pi}{4} .\n\\end{aligned}\n$$", "task_type": {"major": ["试题"], "minor": ["考研", "开放题"]}, "domain": ["数学"], "metadata": "year:2023, score:12", "answer_from": "human", "human_verified": false, "copyright": "暂无版权及作者信息"} +{"instruction": "设函数 $f(x)$ 在 $[-a, a]$ 上具有二阶连续导数, 证明:\n(I) 若 $f(x)=0$, 则存在 $\\xi \\in(-a, a)$, 使得 $f^{\\prime \\prime}(\\xi)=\\frac{1}{a^2}[f(a)+f(-a)]$;\n(II) 若 $f(x)$ 在 $(-a, a)$ 内取得极值, 则存在 $\\eta \\in(-a, a)$ 使得\n$$\n\\left|f^{\\prime \\prime}(\\eta)\\right| \\geq \\frac{1}{2 a^2}|f(a)-f(-a)| .\n$$", "input": "", "output": "(I) 证明: $f(x)=f(0)+f^{\\prime}(0) x+\\frac{f^{\\prime \\prime}(\\eta)}{2 !} x^2=f^{\\prime}(0) x+\\frac{f^{\\prime \\prime}(\\eta)}{2 !} x^2, \\eta$ 介于 0 与 $x$ 之间,则 $f(a)=f^{\\prime}(0) a+\\frac{f^{\\prime \\prime}\\left(\\eta_1\\right)}{2 !} a^2, 0<\\eta_1